You are on page 1of 199

AP PHYSICS 1

Course Syllabus

COURSE INTRODUCTION

AP Physics 1 is an algebra based course in general physics. General physics topics


presented during the course closely follow those outlined by the College Board and also
mirrors an introductory level university physics course.

AP Physics 1 is organized around seven big ideas that bring together the fundamental
science principles and theories of general physics. These big ideas are intended to
encourage students to think about physics concepts as interconnected pieces of a puzzle.
The solution to the puzzle is how the real world around them actually works. The
students will participate in inquiry based explorations of these topics to gain a more
conceptual understanding of these physics concepts. Students will spend less of their
time in traditional formula based learning and more of their effort will be directed to
developing critical thinking and reasoning skills.

TEXT
Physics Principles with Applications 7th Edition, Douglas C. Giancoli, Pearson Prentice
Hall (2014)

BIG IDEAS FOR AP PHYSICS 1

Big Idea 1: Objects and systems have properties such as mass and charge. Systems may
have internal structure.
Big Idea 2: Fields existing in space can be used to explain interactions.
Big Idea 3: The interactions of an object with other objects can be described by forces.
Big Idea 4: Interactions between systems can result in changes in those systems.
Big Idea 5: Changes that occur as a result of interactions are constrained by conservation
laws.
Big Idea 6: Waves can transfer energy and momentum from one location to another
without the permanent transfer of mass and serve as a mathematical model for the
description of other phenomena.
Big Idea 7: The mathematics of probability can be used to describe the behavior of
complex systems and to interpret the behavior of quantum mechanical systems.

TOPICS
• Kinematics
• Dynamics: Newton’s laws
• Circular motion and universal law of gravitation
• Simple harmonic motion: simple pendulum and mass-spring systems
• Impulse, linear momentum, and conservation of linear momentum: collisions
• Work, energy, and conservation of energy
• Rotational motion: torque, rotational kinematics and energy, rotational dynamics, and
conservation of angular momentum
• Electrostatics: electric charge and electric force
• DC circuits: resistors only
• Mechanical waves and sound

GRADING
Grades will be assigned based on the percentage of points accumulated through exams,
quizzes, labs, homework, projects, and various other exercises during the semester.

EXAMS AND QUIZZES (60%)


Exams will follow each chapter with a comprehensive final at the end of each semester.
Each exam will be worth 100 points and each final will be worth 200 points. Every test
will be formatted to simulate the format of AP Physics 1 test given at the end of the class.
Quizzes will be given on a regular basis. Quizzes will be formatted to simulate the free
response portion of the AP Physics 1 test given at the end of the class. Quizzes will be
both announced and unannounced.

LABORATORY (25%)
There is an extensive laboratory component in AP Physics 1. All labs will involve the
manipulation of give equipment to explore a concept in a real world setting. Many labs
will involve the use of sensors and computer simulations. There will be labs that will
involve either and opened questions or problem. These labs will require the need to
design an experiment with given equipment to collect the data need to address the
question at hand. All labs will be recorded in a laboratory notebook for future reference.

1. Graph matching (motion sensor)


2. Accelerated motion (motion sensor)
3. Determine g on an incline (motion sensor)
4. Picket fence free fall (photo gate)
5. Projectile motion (photo gate)
6. Bull’s eye (projectile motion)
7. Force and acceleration
8. Force and equilibrium
9. Atwood’s machine (smart pulley system)
10. Static and kinetic friction (force sensor)
11. Momentum, energy, and collisions (force sensor)
12. Impulse and momentum (force sensor)
13. Elastic collisions
14. Work and energy relationships
15. Centripetal force
16. Rotation equilibrium
17. Simple harmonic motion (motion sensor)
18. Pendulum periods (photo gate)
19. Ohm’s Law (current and voltage probe)
20. Series and Parallel Circuits (virtual lab)
21. Speed of sound (microphone sensor)
HOMEWORK (15%)
Homework will be assigned on a regular basis to reinforce classroom learning. All
problems will be in preparation for the AP Physics B exam. Each problem has been
selected to reinforce a specific concept and increase the student’s problem solving skills.

MAKE UP WORK
1. No make-up work will be allowed.
2. All assignments are due at the beginning of class on the posted due date.
3. In the case of excused absences, all make up work must be completed and turned
in according to school policy.
4. Exams will be completed on the first day you return unless other arrangements are
made in advance.
5. If a laboratory exercise is missed it must be made up outside of regular class time.
Unit 5 (Exam 5 12/13/2018)

TOPIC READING ASSESSMENT

• Forces • Chp 4 • Incline plane practice problems


• Incline plane homework
WEEK 1

• Dynamics practice problems I


• Dynamics homework I
• Lab: Atwood Machine

• Forces • Chp 4 • Dynamics practice problems I


• Dynamics homework II
WEEK 2

• Lab: Newton’s 2nd law


• Quiz

• Work/Energy • Chp 5 • Work Practice problems


• Work homework
• Conservation of Energy Practice problems
WEEK 3

• Conservation of Energy homework


• Exam 4 Review
• Exam 4
Chapter 2 Kinematics in One Dimension

Chapter 2

KINEMATICS IN ONE DIMENSION

PREVIEW

Kinematics is the study of how things move – how far (distance and displacement), how
fast (speed and velocity), and how fast that how fast changes (acceleration). We say that
an object moving in a straight line is moving in one dimension, and an object which is
moving in a curved path (like a projectile) is moving in two dimensions. We relate all
these quantities with a set of equations called the kinematic equations.

The content contained in all sections of chapter 2 of the textbook is included on the AP
Physics B exam.

QUICK REFERENCE

Important Terms

acceleration
the rate of change in velocity
acceleration due to gravity
the acceleration of a freely falling object in the absence of air resistance, which
near the earth’s surface is approximately 10 m/s2 .
acceleration-time graph
plot of the acceleration of an object as a function of time
average acceleration
the acceleration of an object measured over a time interval
average velocity
the velocity of an object measured over a time interval; the displacement of an
object divided by the change in time during the motion
constant (or uniform) acceleration
acceleration which does not change during a time interval
constant (or uniform) velocity
velocity which does not change during a time interval
displacement
change in position in a particular direction (vector)
distance
the length moved between two points (scalar)
free fall
motion under the influence of gravity
initial velocity
the velocity at which an object starts at the beginning of a time interval

19
Chapter 2 Kinematics in One Dimension

instantaneous
the value of a quantity at a particular instant of time, such as instantaneous
position, velocity, or acceleration
kinematics
the study of how motion occurs, including distance, displacement, speed, velocity,
acceleration, and time.
position-time graph
the graph of the motion of an object that shows how its position varies with time
speed
the ratio of distance to time
velocity
ratio of the displacement of an object to a time interval
velocity-time graph
plot of the velocity of an object as a function of time, the slope of which is
acceleration, and the area under which is displacement

Equations and Symbols


v − vo
a=
t
v = vo + at
1
∆x = (vo + v)t
2
1
∆x = vo t + at 2
2
v = vo + 2a∆x
2 2

where

Δx = displacement (final position – initial


position)
v = velocity or speed at any time
v o = initial velocity or speed
t = time
a = acceleration

Ten Homework Problems


Chapter 2 Problems 2, 10, 13, 30, 40, 43, 58, 59, 73, 78

20
Chapter 2 Kinematics in One Dimension

DISCUSSION OF SELECTED SECTIONS

2.1 Displacement
Distance d can be defined as total length moved. If you run around a circular track, you
have covered a distance equal to the circumference of the track. Distance is a scalar,
which means it has no direction associated with it. Displacement Δx, however, is a
vector. Displacement is defined as the straight-line distance between two points, and is a
vector which points from an object’s initial position x o toward its final position x f . In our
previous example, if you run around a circular track and end up at the same place you
started, your displacement is zero, since there is no distance between your starting point
and your ending point. Displacement is often written in its scalar form as simply Δx or x.

2.2 Speed and Velocity


Average speed is defined as the amount of distance a moving object covers divided by
the amount of time it takes to cover that distance:

distance d
average speed = v = =
elapsed time t

where v stands for speed, d is for distance, and t is time.

Average velocity is defined a little differently than average speed. While average speed is
the total change in distance divided by the total change in time, average velocity is the
displacement divided by the change in time. Since velocity is a vector, we must define it
in terms of another vector, displacement. Oftentimes average speed and average velocity
are interchangeable for the purposes of the AP Physics B exam. Speed is the magnitude
of velocity, that is, speed is a scalar and velocity is a vector. For example, if you are
driving west at 50 miles per hour, we say that your speed is 50 mph, and your velocity is
50 mph west. We will use the letter v for both speed and velocity in our calculations, and
will take the direction of velocity into account when necessary.
2.3 Acceleration
Acceleration tells us how fast velocity is changing. For example, if you start from rest on
the goal line of a football field, and begin walking up to a speed of 1 m/s for the first
second, then up to 2 m/s, for the second second, then up to 3 m/s for the third second, you
are speeding up with an average acceleration of 1 m/s for each second you are walking.
We write

∆v 1 m / s m
a= = = 1m / s / s = 1 2
∆t 1s s

In other words, you are changing your speed by 1 m/s for each second you walk. If you
start with a high velocity and slow down, you are still accelerating, but your acceleration
would be considered negative, compared to the positive acceleration discussed above.

21
Chapter 2 Kinematics in One Dimension

Usually, the change in speed ∆v is calculated by the final speed v f minus the initial speed
v o . The initial and final speeds are called instantaneous speeds, since they each occur at a
particular instant in time and are not average speeds.

2.5 Applications of the Equations of Kinematics for Constant

Acceleration
Kinematics is the study of the relationships between distance and displacement, speed and
velocity, acceleration, and time. The kinematic equations are the equations of motion
which relate these quantities to each other. These equations assume that the acceleration
of an object is uniform, that is, constant for the time interval we are interested in. The
kinematic equations listed below would not work for calculating velocities and
displacements for an object which is accelerating erratically. Fortunately, the AP Physics
B exam generally deals with uniform acceleration, so the kinematic equations listed
above will be very helpful in solving problems on the test.

2.6 Freely Falling Bodies


An object is in free fall if it is falling freely under the influence of gravity. Any object,
regardless of its mass, falls near the surface of the Earth with an acceleration of 9.8 m/s2,
which we will denote with the letter g. We will round the free fall acceleration g to 10
m/s2 for the purpose of the AP Physics B exam. This free fall acceleration assumes that
there is no air resistance to impede the motion of the falling object, and this is a safe
assumption on the AP Physics B test unless you are told differently for a particular
question on the exam.
Since the free fall acceleration is constant, we may use the kinematic equations to solve
problems involving free fall. We simply need to replace the acceleration a with the
specific free fall acceleration g in each equation. Remember, anytime a velocity and
acceleration are in opposite directions (like when a ball is rising after being thrown
upward), you must give one of them a negative sign.

22
Chapter 2 Kinematics in One Dimension

Example 1 A girl is holding a ball as she steps onto a tall elevator on the ground floor of
a building. The girl holds the ball at a height of 1 meter above the elevator floor. The
elevator begins accelerating upward from rest at 3 m/s2. After the elevator accelerates for
5 seconds, find

(a) the speed of the elevator


(b) the height of the floor of the
elevator above the ground.

At the end of 5 s, the girl lets go of the ball from a height of 1 meter above the floor of
the elevator. If the elevator continues to accelerate upward at 3 m/s2, describe the motion
of the ball
(c) relative to the girl’s hand,
(d) relative to the ground.

(e) Determine the time after the ball is released that it will make contact with the floor.

(f) What is the height above the ground of the ball and floor when they first make
contact?

Solution:
(a) v = vo + at = 0 + (3 m / s 2 )(5s ) = 15 m / s upward

(b) y = vo t + at 2 = 0 + (3 m / s 2 )(5 s ) = 37.5 m


1 1 2

2 2
(c) When the girl releases the ball, both she and the ball are moving with a speed of 15
m/s upward. However, the girl continues to accelerate upward at 3 m/s2, but the ball
ceases to accelerate upward, and the ball’s acceleration is directed downward at g = 10
m/s2, that is, it is in free fall with an initial upward velocity of 15 m/s. Therefore the ball
will appear to the girl to fall downward with an acceleration of 3 m/s2 – (- 10 m/s2) = 13
m/s2 downward, and will quickly fall below her hand.

(d) Someone watching the ball from the ground would simply see the ball rising upward
with an initial velocity of 15 m/s, and would watch it rise to a maximum height, at which
point it would be instantaneously at rest (provided it doesn’t strike the floor of the
elevator before it reaches its maximum height).

(e) When the ball is released, it is traveling upward with a speed of 15 m/s, has a
downward acceleration of 13 m/s2 relative to the floor, and is at a height y = 1 m above
the floor. The time it takes to “fall” to the floor is

23
Chapter 2 Kinematics in One Dimension

1 2
y= at
2
1
( )
1 m = 13 m / s 2 t 2
2
t = 0.4 s

(f) In this time of 0.4 s, the elevator floor has moved up a distance of
∆y = a e t 2 = (3 m / s 2 )(0.4 s ) = 0.24 m
1 1 2

2 2

Thus, the ball and elevator floor collide at a height above the ground of
37.5 m + 0.24 m = 37.74 m.

2.7 Graphical Analysis of Velocity and Acceleration


Let’s take some time to review how we interpret the motion of an object when we are
given the information about it in graphical form. On the AP Physics B exam, you will
need to be able to interpret three types of graphs: position vs.time, velocity vs. time, and
acceleration vs. time.

Position vs. time

Consider the position vs. time graph below:

x (m) x (m)
Δx P
Δx

Δt
Δt

t (s) t (s)

∆x
The slope of the graph on the left is , and is therefore velocity. The curved graph on
∆t
the right indicates that the slope is changing. The slope of the curved graph is still

velocity, even though the velocity is changing, indicating the object is accelerating. The
instantaneous velocity at any point on the graph (such as point P) can be found by
drawing a tangent line at the point and finding the slope of the tangent line.

24
Chapter 2 Kinematics in One Dimension

Velocity vs. time

Consider the velocity vs. time graph below:

v (m/s) v (m/s)
Δv

Area
Δt
t (s)
t (s)

∆v
As shown in the figure on the left, the slope of a velocity vs. time graph is , and is
∆t
therefore acceleration. As shown on the figure on the right, the area under a velocity vs.
time graph would have units of (s ) = m , and is therefore displacement.
m
s

Acceleration vs. time

Since the AP Physics B exam generally deals with constant acceleration, any graph of
acceleration vs. time on the exam would likely be a straight horizontal line:

a (m/s2)
a (m/s2)
+5 m/s2

0
0
t(s)
t(s)
-5 m/s2

This graph on the left tells us that the acceleration of this object is positive. If the object
were accelerating negatively, the horizontal line would be below the time axis, as shown
in the graph on the right.

25
Chapter 2 Kinematics in One Dimension

Example 2 Consider the position vs. time graph below representing the motion of a car.

Assume that all accelerations of the car are constant.


H I
G J

x(m) D E
F
C

B K
0
t(s)

On the axes below, sketch the velocity vs. time and acceleration vs. time graphs for this
car.

v(m/s)

t(s)

a(m/s2)

t(s)

26
Chapter 2 Kinematics in One Dimension

Solution:
The car starts out at a distance behind our reference point of zero, indicated on the graph
as a negative displacement. The velocity (slope) of the car is initially positive and
constant from points A to C, with the car crossing the reference point at B. Between
points C and D, the car goes from a high positive velocity (slope) to a low velocity,
eventually coming to rest (v = 0) at point D. At point E the car accelerates positively from
rest up to a positive constant velocity from points F to G. Then the velocity (slope)
decreases from points G to H, indicating the car is slowing down. It is between these two
points that the car’s velocity is positive, but its acceleration is negative, since the car’s
velocity and acceleration are in opposite directions. The car once again comes to rest at
point H, and then begins gaining a negative velocity (moving backward) from rest at
point I, increasing its speed negatively to a constant negative velocity between points J
and K. At K, the car has returned to its original starting position.

The velocity vs. time graph for this car would look like this:

v(m/s)

B C F G
A
D E I
H
0

t(s)
J K

The acceleration vs. time graph for this car would look like this:

a(m/s2)

E F

A B C D G H I J K
0

t(s)

27
Chapter 2 Kinematics in One Dimension

CHAPTER 2 REVIEW QUESTIONS

For each of the multiple choice questions below, choose the best answer.
Unless otherwise noted, use g = 10 m/s2 and neglect air resistance.

1. Which of the following statements is 5. A bus starting from a speed of +24


true? m/s slows to 6 m/s in a time of 3 s. The
(A) Displacement is a scalar and average acceleration of the bus is
distance is a vector. (A) 2 m/s2
(B) Displacement is a vector and (B) 4 m/s2
distance is a scalar. (C) 6 m/s2
(C) Both displacement and distance are (D) – 2 m/s2
vectors. (E) – 6 m/s2
(D) Neither displacement nor distance
are vectors. 6. A train accelerates from rest with an
(E) Displacement and distance are acceleration of 4 m/s2 for a time of 20 s.
always equal. What is the train’s speed at the end of 20
s?
2. Which of the following is the best (A) 0.25 m/s
statement for a velocity? (B) 4 m/s
(A) 60 miles per hour (C) 2.5 m/s
(B) 30 meters per second (D) 0.8 m/s
(C) 30 km at 45° north of east (E) 80 m/s
(D) 40 km/hr
(E) 50 km/hr southwest 7. A football player starts from rest 10
meters from the goal line and accelerates
3. A jogger runs 4 km in 0.4 hr, then 8 away from the goal line at 5 m/s2. How
km in 0.8 hr. What is the average speed far from the goal line is the player after 4
of the jogger? s?
(A) 10 km/hr (A) 6 m
(B) 3 km/hr (B) 30 m
(C) 1 km/hr (C) 40 m
(D) 0.1 km/hr (D) 50 m
(E) 100 km/hr (E) 60 m

4. A motorcycle starts from rest and


accelerates to a speed of 20 m/s in a time
of 8 s. What is the motorcycle’s average
acceleration?
(A) 160 m/s2
(B) 80 m/s2
(C) 8 m/s2
(D) 2.5 m/s2
(E) 0.4 m/s2

28
Chapter 2 Kinematics in One Dimension

8. A ball is dropped from rest. What is


the acceleration of the ball immediately
after it is dropped?
(A) zero
(B) 5 m/s2
(C) 10 m/s2
(D) 20 m/s2
(E) 30 m/s2

Questions 9 – 11: 12. Which two of the following pairs of


A ball is thrown straight upward with a graphs are equivalent?
speed of +12 m/s. (A) x v

9. What is the ball’s acceleration just 0


after it is thrown? t t
(A) zero
(B) 10 m/s2 upward (B)
x v
(C) 10 m/s2 downward
(D) 12 m/s2 upward
0
(E) 12 m/s2 downward t t

10. How much time does it take for the (C) x v


ball to rise to its maximum height?
(A) 24 s 0
(B) 12 s t t
(C) 10 s
(D) 2 s (D) x v
(E) 1.2 s
0
11. What is the approximate maximum t t
height the ball reaches?
(A) 24 m (E) x v
(B) 17 m
(C) 12 m 0
t t
(D) 7 m
(E) 5 m

29
Chapter 2 Kinematics in One Dimension

Questions 13 – 14:
Consider the velocity vs time graph
below:

13. A which time(s) is the object at rest?


(A) zero
(B) 1 s
(C) 3 s to 4 s
(D) 4 s only
(E) 8 s

14. During which interval is the speed of


the object decreasing?
(A) 0 to 1 s
(B) 1 s to 3 s
(C) 3 s to 4 s
(D) 4 s to 8 s
(E) the speed of the object is never
decreasing in this graph

30
Chapter 2 Kinematics in One Dimension

Free Response Question


Directions: Show all work in working the following question. The question is worth 15 points,
and the suggested time for answering the question is about 15 minutes. The parts within a
question may not have equal weight.

1. (15 points)

A cart
on a long horizontal track can move with negligible friction to the left or to the right. During the
time intervals when the cart is accelerating, the acceleration is constant. The acceleration during
other time intervals is also constant, but may have a different value. Data is taken on the motion
of the cart, and recorded in the table below.

Displacement Velocity time


x(m) v(m/s) t(s)
2 -4 0
-2 1
-2 2
-2 3
1 6
1 7
0 9
0 10

31
Chapter 2 Kinematics in One Dimension

(a) Plot these data points on the v vs t graph below, and draw the best-fit straight lines between
each data point, that is, connect each data point to the one before it. The acceleration is constant
or zero during each interval listed in the data table.

(b) List all of the times between t = 0 and t = 10 s at which the cart is at rest.

(c) i. During which time interval is the magnitude of the acceleration of the cart the
greatest?
ii. What is the value of this maximum acceleration?

(d) Find the displacement of the cart from x = 0 at a time of 10 s.

(e) On the following graph, sketch the acceleration vs. time graph for the motion of this cart
from t = 0 to t = 10 s.

32
Chapter 2 Kinematics in One Dimension

ANSWERS AND EXPLANATIONS TO CHAPTER 2 REVIEW QUESTIONS

Multiple Choice

1. B
Displacement is the straight-line length from an origin to a final position and includes direction,
whereas distance is simply length moved.

2. E
Velocity is a vector and therefore direction should be included.

3. A
Average speed is total distance divided by total time. The total distance covered by the jogger is
12 km and the total time is 1.2 hours, so the average speed is 10 km/hr.

4. D
∆v 20 m / s m
a= = = 2.5 2
∆t 8s s

5. E
v f − vo 6 m / s − 24 m / s m
a= = =−6 2
t 3s s

6. E
( )
v f = vi + at = 0 + 4 m / s 2 (20 s ) = 80 m / s

33
Chapter 2 Kinematics in One Dimension

7. D
1 m 
at = (10 m) + 0 +  5 2 (4 s ) = 50 m
1 2
x f = xo + vo t +
2

2 2 s 

8. C
The acceleration due to gravity is 10 m/s2 at all points during the ball’s fall.

9. C
After the ball is thrown, the only acceleration it has is the acceleration due to gravity, 10 m/s2.

10. E
At the ball’s maximum height, v f = 0. Thus,
v f = vo − gt = 0
12 m / s
t= = 1 .2 s
10 m / s 2

11. D
1 m 
y = gt 2 = 10 2 (1.2 s ) = 7.2 m ≈ 7 m
1 2

2 2 s 

12. B
Both of these graphs represent motion that begins at a high positive velocity, and slows down to
zero velocity.

13. B
The line crosses the axis (v = 0) at a time of 1 second.

14. A
The object begins with a high negative (backward) velocity at t = 0, then its speed decreases to
zero by a time of 1 s.

34
Chapter 2 Kinematics in One Dimension

Free Response Question Solution


(a) 4 points

(b) 2 points
The cart is at rest when the velocity is zero, that is, when the graph crosses the time axis. Thus, v
= 0 at 5 s, 9 s, and 10 s, as well as all points between 9 and 10 s.

(c) i. 1 point
The acceleration can be found by finding the slope of the v vs t graph in a particular interval. The
slope (acceleration) is maximum (steepest) in the time interval from 0 to 1 s.

ii. 2 points
− 2 m / s − (− 4 m / s )
Acceleration = slope of v vs t graph = = 2m / s2
1s − 0 s

(d) 3 points
The displacement of the cart from x = 0 can be found by determining the area under the graph.
Note that the area is negative from 0 to 5 s, and positive from 5 s to 9 s. Don’t forget the initial
displacement of 2 m at t = 0.

Area from 0 to 5 s = 10 squares = - 10 m.


Area from 5 to 10 s = 2.5 squares = +2.5 m

Total displacement from x = 0 is 2 m – 10 m + 2.5 m = - 5.5 m.

35
Chapter 2 Kinematics in One Dimension

(e) 3 points

36
Chapter 2 Kinematics in One Dimension

Chapter 2

KINEMATICS IN ONE DIMENSION

PREVIEW

Kinematics is the study of how things move – how far (distance and displacement), how
fast (speed and velocity), and how fast that how fast changes (acceleration). We say that
an object moving in a straight line is moving in one dimension, and an object which is
moving in a curved path (like a projectile) is moving in two dimensions. We relate all
these quantities with a set of equations called the kinematic equations.

The content contained in all sections of chapter 2 of the textbook is included on the AP
Physics B exam.

QUICK REFERENCE

Important Terms

acceleration
the rate of change in velocity
acceleration due to gravity
the acceleration of a freely falling object in the absence of air resistance, which
near the earth’s surface is approximately 10 m/s2 .
acceleration-time graph
plot of the acceleration of an object as a function of time
average acceleration
the acceleration of an object measured over a time interval
average velocity
the velocity of an object measured over a time interval; the displacement of an
object divided by the change in time during the motion
constant (or uniform) acceleration
acceleration which does not change during a time interval
constant (or uniform) velocity
velocity which does not change during a time interval
displacement
change in position in a particular direction (vector)
distance
the length moved between two points (scalar)
free fall
motion under the influence of gravity
initial velocity
the velocity at which an object starts at the beginning of a time interval

19
Chapter 2 Kinematics in One Dimension

instantaneous
the value of a quantity at a particular instant of time, such as instantaneous
position, velocity, or acceleration
kinematics
the study of how motion occurs, including distance, displacement, speed, velocity,
acceleration, and time.
position-time graph
the graph of the motion of an object that shows how its position varies with time
speed
the ratio of distance to time
velocity
ratio of the displacement of an object to a time interval
velocity-time graph
plot of the velocity of an object as a function of time, the slope of which is
acceleration, and the area under which is displacement

Equations and Symbols


v − vo
a=
t
v = vo + at
1
∆x = (vo + v)t
2
1
∆x = vo t + at 2
2
v = vo + 2a∆x
2 2

where

Δx = displacement (final position – initial


position)
v = velocity or speed at any time
v o = initial velocity or speed
t = time
a = acceleration

Ten Homework Problems


Chapter 2 Problems 2, 10, 13, 30, 40, 43, 58, 59, 73, 78

20
Chapter 2 Kinematics in One Dimension

DISCUSSION OF SELECTED SECTIONS

2.1 Displacement
Distance d can be defined as total length moved. If you run around a circular track, you
have covered a distance equal to the circumference of the track. Distance is a scalar,
which means it has no direction associated with it. Displacement Δx, however, is a
vector. Displacement is defined as the straight-line distance between two points, and is a
vector which points from an object’s initial position x o toward its final position x f . In our
previous example, if you run around a circular track and end up at the same place you
started, your displacement is zero, since there is no distance between your starting point
and your ending point. Displacement is often written in its scalar form as simply Δx or x.

2.2 Speed and Velocity


Average speed is defined as the amount of distance a moving object covers divided by
the amount of time it takes to cover that distance:

distance d
average speed = v = =
elapsed time t

where v stands for speed, d is for distance, and t is time.

Average velocity is defined a little differently than average speed. While average speed is
the total change in distance divided by the total change in time, average velocity is the
displacement divided by the change in time. Since velocity is a vector, we must define it
in terms of another vector, displacement. Oftentimes average speed and average velocity
are interchangeable for the purposes of the AP Physics B exam. Speed is the magnitude
of velocity, that is, speed is a scalar and velocity is a vector. For example, if you are
driving west at 50 miles per hour, we say that your speed is 50 mph, and your velocity is
50 mph west. We will use the letter v for both speed and velocity in our calculations, and
will take the direction of velocity into account when necessary.
2.3 Acceleration
Acceleration tells us how fast velocity is changing. For example, if you start from rest on
the goal line of a football field, and begin walking up to a speed of 1 m/s for the first
second, then up to 2 m/s, for the second second, then up to 3 m/s for the third second, you
are speeding up with an average acceleration of 1 m/s for each second you are walking.
We write

∆v 1 m / s m
a= = = 1m / s / s = 1 2
∆t 1s s

In other words, you are changing your speed by 1 m/s for each second you walk. If you
start with a high velocity and slow down, you are still accelerating, but your acceleration
would be considered negative, compared to the positive acceleration discussed above.

21
Chapter 2 Kinematics in One Dimension

Usually, the change in speed ∆v is calculated by the final speed v f minus the initial speed
v o . The initial and final speeds are called instantaneous speeds, since they each occur at a
particular instant in time and are not average speeds.

2.5 Applications of the Equations of Kinematics for Constant

Acceleration
Kinematics is the study of the relationships between distance and displacement, speed and
velocity, acceleration, and time. The kinematic equations are the equations of motion
which relate these quantities to each other. These equations assume that the acceleration
of an object is uniform, that is, constant for the time interval we are interested in. The
kinematic equations listed below would not work for calculating velocities and
displacements for an object which is accelerating erratically. Fortunately, the AP Physics
B exam generally deals with uniform acceleration, so the kinematic equations listed
above will be very helpful in solving problems on the test.

2.6 Freely Falling Bodies


An object is in free fall if it is falling freely under the influence of gravity. Any object,
regardless of its mass, falls near the surface of the Earth with an acceleration of 9.8 m/s2,
which we will denote with the letter g. We will round the free fall acceleration g to 10
m/s2 for the purpose of the AP Physics B exam. This free fall acceleration assumes that
there is no air resistance to impede the motion of the falling object, and this is a safe
assumption on the AP Physics B test unless you are told differently for a particular
question on the exam.
Since the free fall acceleration is constant, we may use the kinematic equations to solve
problems involving free fall. We simply need to replace the acceleration a with the
specific free fall acceleration g in each equation. Remember, anytime a velocity and
acceleration are in opposite directions (like when a ball is rising after being thrown
upward), you must give one of them a negative sign.

22
Chapter 2 Kinematics in One Dimension

Example 1 A girl is holding a ball as she steps onto a tall elevator on the ground floor of
a building. The girl holds the ball at a height of 1 meter above the elevator floor. The
elevator begins accelerating upward from rest at 3 m/s2. After the elevator accelerates for
5 seconds, find

(a) the speed of the elevator


(b) the height of the floor of the
elevator above the ground.

At the end of 5 s, the girl lets go of the ball from a height of 1 meter above the floor of
the elevator. If the elevator continues to accelerate upward at 3 m/s2, describe the motion
of the ball
(c) relative to the girl’s hand,
(d) relative to the ground.

(e) Determine the time after the ball is released that it will make contact with the floor.

(f) What is the height above the ground of the ball and floor when they first make
contact?

Solution:
(a) v = vo + at = 0 + (3 m / s 2 )(5s ) = 15 m / s upward

(b) y = vo t + at 2 = 0 + (3 m / s 2 )(5 s ) = 37.5 m


1 1 2

2 2
(c) When the girl releases the ball, both she and the ball are moving with a speed of 15
m/s upward. However, the girl continues to accelerate upward at 3 m/s2, but the ball
ceases to accelerate upward, and the ball’s acceleration is directed downward at g = 10
m/s2, that is, it is in free fall with an initial upward velocity of 15 m/s. Therefore the ball
will appear to the girl to fall downward with an acceleration of 3 m/s2 – (- 10 m/s2) = 13
m/s2 downward, and will quickly fall below her hand.

(d) Someone watching the ball from the ground would simply see the ball rising upward
with an initial velocity of 15 m/s, and would watch it rise to a maximum height, at which
point it would be instantaneously at rest (provided it doesn’t strike the floor of the
elevator before it reaches its maximum height).

(e) When the ball is released, it is traveling upward with a speed of 15 m/s, has a
downward acceleration of 13 m/s2 relative to the floor, and is at a height y = 1 m above
the floor. The time it takes to “fall” to the floor is

23
Chapter 2 Kinematics in One Dimension

1 2
y= at
2
1
( )
1 m = 13 m / s 2 t 2
2
t = 0.4 s

(f) In this time of 0.4 s, the elevator floor has moved up a distance of
∆y = a e t 2 = (3 m / s 2 )(0.4 s ) = 0.24 m
1 1 2

2 2

Thus, the ball and elevator floor collide at a height above the ground of
37.5 m + 0.24 m = 37.74 m.

2.7 Graphical Analysis of Velocity and Acceleration


Let’s take some time to review how we interpret the motion of an object when we are
given the information about it in graphical form. On the AP Physics B exam, you will
need to be able to interpret three types of graphs: position vs.time, velocity vs. time, and
acceleration vs. time.

Position vs. time

Consider the position vs. time graph below:

x (m) x (m)
Δx P
Δx

Δt
Δt

t (s) t (s)

∆x
The slope of the graph on the left is , and is therefore velocity. The curved graph on
∆t
the right indicates that the slope is changing. The slope of the curved graph is still

velocity, even though the velocity is changing, indicating the object is accelerating. The
instantaneous velocity at any point on the graph (such as point P) can be found by
drawing a tangent line at the point and finding the slope of the tangent line.

24
Chapter 2 Kinematics in One Dimension

Velocity vs. time

Consider the velocity vs. time graph below:

v (m/s) v (m/s)
Δv

Area
Δt
t (s)
t (s)

∆v
As shown in the figure on the left, the slope of a velocity vs. time graph is , and is
∆t
therefore acceleration. As shown on the figure on the right, the area under a velocity vs.
time graph would have units of (s ) = m , and is therefore displacement.
m
s

Acceleration vs. time

Since the AP Physics B exam generally deals with constant acceleration, any graph of
acceleration vs. time on the exam would likely be a straight horizontal line:

a (m/s2)
a (m/s2)
+5 m/s2

0
0
t(s)
t(s)
-5 m/s2

This graph on the left tells us that the acceleration of this object is positive. If the object
were accelerating negatively, the horizontal line would be below the time axis, as shown
in the graph on the right.

25
Chapter 2 Kinematics in One Dimension

Example 2 Consider the position vs. time graph below representing the motion of a car.

Assume that all accelerations of the car are constant.


H I
G J

x(m) D E
F
C

B K
0
t(s)

On the axes below, sketch the velocity vs. time and acceleration vs. time graphs for this
car.

v(m/s)

t(s)

a(m/s2)

t(s)

26
Chapter 2 Kinematics in One Dimension

Solution:
The car starts out at a distance behind our reference point of zero, indicated on the graph
as a negative displacement. The velocity (slope) of the car is initially positive and
constant from points A to C, with the car crossing the reference point at B. Between
points C and D, the car goes from a high positive velocity (slope) to a low velocity,
eventually coming to rest (v = 0) at point D. At point E the car accelerates positively from
rest up to a positive constant velocity from points F to G. Then the velocity (slope)
decreases from points G to H, indicating the car is slowing down. It is between these two
points that the car’s velocity is positive, but its acceleration is negative, since the car’s
velocity and acceleration are in opposite directions. The car once again comes to rest at
point H, and then begins gaining a negative velocity (moving backward) from rest at
point I, increasing its speed negatively to a constant negative velocity between points J
and K. At K, the car has returned to its original starting position.

The velocity vs. time graph for this car would look like this:

v(m/s)

B C F G
A
D E I
H
0

t(s)
J K

The acceleration vs. time graph for this car would look like this:

a(m/s2)

E F

A B C D G H I J K
0

t(s)

27
Chapter 2 Kinematics in One Dimension

CHAPTER 2 REVIEW QUESTIONS

For each of the multiple choice questions below, choose the best answer.
Unless otherwise noted, use g = 10 m/s2 and neglect air resistance.

1. Which of the following statements is 5. A bus starting from a speed of +24


true? m/s slows to 6 m/s in a time of 3 s. The
(A) Displacement is a scalar and average acceleration of the bus is
distance is a vector. (A) 2 m/s2
(B) Displacement is a vector and (B) 4 m/s2
distance is a scalar. (C) 6 m/s2
(C) Both displacement and distance are (D) – 2 m/s2
vectors. (E) – 6 m/s2
(D) Neither displacement nor distance
are vectors. 6. A train accelerates from rest with an
(E) Displacement and distance are acceleration of 4 m/s2 for a time of 20 s.
always equal. What is the train’s speed at the end of 20
s?
2. Which of the following is the best (A) 0.25 m/s
statement for a velocity? (B) 4 m/s
(A) 60 miles per hour (C) 2.5 m/s
(B) 30 meters per second (D) 0.8 m/s
(C) 30 km at 45° north of east (E) 80 m/s
(D) 40 km/hr
(E) 50 km/hr southwest 7. A football player starts from rest 10
meters from the goal line and accelerates
3. A jogger runs 4 km in 0.4 hr, then 8 away from the goal line at 5 m/s2. How
km in 0.8 hr. What is the average speed far from the goal line is the player after 4
of the jogger? s?
(A) 10 km/hr (A) 6 m
(B) 3 km/hr (B) 30 m
(C) 1 km/hr (C) 40 m
(D) 0.1 km/hr (D) 50 m
(E) 100 km/hr (E) 60 m

4. A motorcycle starts from rest and


accelerates to a speed of 20 m/s in a time
of 8 s. What is the motorcycle’s average
acceleration?
(A) 160 m/s2
(B) 80 m/s2
(C) 8 m/s2
(D) 2.5 m/s2
(E) 0.4 m/s2

28
Chapter 2 Kinematics in One Dimension

8. A ball is dropped from rest. What is


the acceleration of the ball immediately
after it is dropped?
(A) zero
(B) 5 m/s2
(C) 10 m/s2
(D) 20 m/s2
(E) 30 m/s2

Questions 9 – 11: 12. Which two of the following pairs of


A ball is thrown straight upward with a graphs are equivalent?
speed of +12 m/s. (A) x v

9. What is the ball’s acceleration just 0


after it is thrown? t t
(A) zero
(B) 10 m/s2 upward (B)
x v
(C) 10 m/s2 downward
(D) 12 m/s2 upward
0
(E) 12 m/s2 downward t t

10. How much time does it take for the (C) x v


ball to rise to its maximum height?
(A) 24 s 0
(B) 12 s t t
(C) 10 s
(D) 2 s (D) x v
(E) 1.2 s
0
11. What is the approximate maximum t t
height the ball reaches?
(A) 24 m (E) x v
(B) 17 m
(C) 12 m 0
t t
(D) 7 m
(E) 5 m

29
Chapter 2 Kinematics in One Dimension

Questions 13 – 14:
Consider the velocity vs time graph
below:

13. A which time(s) is the object at rest?


(A) zero
(B) 1 s
(C) 3 s to 4 s
(D) 4 s only
(E) 8 s

14. During which interval is the speed of


the object decreasing?
(A) 0 to 1 s
(B) 1 s to 3 s
(C) 3 s to 4 s
(D) 4 s to 8 s
(E) the speed of the object is never
decreasing in this graph

30
Chapter 2 Kinematics in One Dimension

Free Response Question


Directions: Show all work in working the following question. The question is worth 15 points,
and the suggested time for answering the question is about 15 minutes. The parts within a
question may not have equal weight.

1. (15 points)

A cart
on a long horizontal track can move with negligible friction to the left or to the right. During the
time intervals when the cart is accelerating, the acceleration is constant. The acceleration during
other time intervals is also constant, but may have a different value. Data is taken on the motion
of the cart, and recorded in the table below.

Displacement Velocity time


x(m) v(m/s) t(s)
2 -4 0
-2 1
-2 2
-2 3
1 6
1 7
0 9
0 10

31
Chapter 2 Kinematics in One Dimension

(a) Plot these data points on the v vs t graph below, and draw the best-fit straight lines between
each data point, that is, connect each data point to the one before it. The acceleration is constant
or zero during each interval listed in the data table.

(b) List all of the times between t = 0 and t = 10 s at which the cart is at rest.

(c) i. During which time interval is the magnitude of the acceleration of the cart the
greatest?
ii. What is the value of this maximum acceleration?

(d) Find the displacement of the cart from x = 0 at a time of 10 s.

(e) On the following graph, sketch the acceleration vs. time graph for the motion of this cart
from t = 0 to t = 10 s.

32
Chapter 2 Kinematics in One Dimension

ANSWERS AND EXPLANATIONS TO CHAPTER 2 REVIEW QUESTIONS

Multiple Choice

1. B
Displacement is the straight-line length from an origin to a final position and includes direction,
whereas distance is simply length moved.

2. E
Velocity is a vector and therefore direction should be included.

3. A
Average speed is total distance divided by total time. The total distance covered by the jogger is
12 km and the total time is 1.2 hours, so the average speed is 10 km/hr.

4. D
∆v 20 m / s m
a= = = 2.5 2
∆t 8s s

5. E
v f − vo 6 m / s − 24 m / s m
a= = =−6 2
t 3s s

6. E
( )
v f = vi + at = 0 + 4 m / s 2 (20 s ) = 80 m / s

33
Chapter 2 Kinematics in One Dimension

7. D
1 m 
at = (10 m) + 0 +  5 2 (4 s ) = 50 m
1 2
x f = xo + vo t +
2

2 2 s 

8. C
The acceleration due to gravity is 10 m/s2 at all points during the ball’s fall.

9. C
After the ball is thrown, the only acceleration it has is the acceleration due to gravity, 10 m/s2.

10. E
At the ball’s maximum height, v f = 0. Thus,
v f = vo − gt = 0
12 m / s
t= = 1 .2 s
10 m / s 2

11. D
1 m 
y = gt 2 = 10 2 (1.2 s ) = 7.2 m ≈ 7 m
1 2

2 2 s 

12. B
Both of these graphs represent motion that begins at a high positive velocity, and slows down to
zero velocity.

13. B
The line crosses the axis (v = 0) at a time of 1 second.

14. A
The object begins with a high negative (backward) velocity at t = 0, then its speed decreases to
zero by a time of 1 s.

34
Chapter 2 Kinematics in One Dimension

Free Response Question Solution


(a) 4 points

(b) 2 points
The cart is at rest when the velocity is zero, that is, when the graph crosses the time axis. Thus, v
= 0 at 5 s, 9 s, and 10 s, as well as all points between 9 and 10 s.

(c) i. 1 point
The acceleration can be found by finding the slope of the v vs t graph in a particular interval. The
slope (acceleration) is maximum (steepest) in the time interval from 0 to 1 s.

ii. 2 points
− 2 m / s − (− 4 m / s )
Acceleration = slope of v vs t graph = = 2m / s2
1s − 0 s

(d) 3 points
The displacement of the cart from x = 0 can be found by determining the area under the graph.
Note that the area is negative from 0 to 5 s, and positive from 5 s to 9 s. Don’t forget the initial
displacement of 2 m at t = 0.

Area from 0 to 5 s = 10 squares = - 10 m.


Area from 5 to 10 s = 2.5 squares = +2.5 m

Total displacement from x = 0 is 2 m – 10 m + 2.5 m = - 5.5 m.

35
Chapter 2 Kinematics in One Dimension

(e) 3 points

36
Chapter 3 Kinematics in Two Dimensions

Chapter 3

KINEMATICS IN TWO DIMENSIONS

PREVIEW

Two-dimensional motion includes objects which are moving in two directions at the same
time, such as a projectile, which has both horizontal and vertical motion. These two
motions of a projectile are completely independent of one another, and can be described
by constant velocity in the horizontal direction, and free fall in the vertical direction.
Since the two-dimensional motion described in this chapter involves only constant
accelerations, we may use the kinematic equations.

The content contained in sections 1, 2, 3, and 5 of chapter 3 of the textbook is included


on the AP Physics B exam.

QUICK REFERENCE

Important Terms

projectile
any object that is projected by a force and continues to move by its own inertia
range of a projectile
the horizontal distance between the launch point of a projectile and where it
returns to its launch height
trajectory
the path followed by a projectile

Equations and Symbols


Horizontal direction: Vertical direction:

v y = voy + a y t
v x = vox + a x t
1
1 y= (vo + v y )t
x = (vo x + v x )t 2 y
2
1
1 y = voy t + a y t 2
x = vox t + a x t 2 2
2
v y = voy + 2a y y
2 2
v x = vox + 2a x x
2 2

For a projectile near the surface of the earth:

a x = 0, v x is constant, and a y = g = 10 m/s2.

37
Chapter 3 Kinematics in Two Dimensions

Ten Homework Problems


Chapter 3 Problems 12, 13, 16, 22, 25, 28, 39, 43, 64, 71

DISCUSSION OF SELECTED SECTIONS

3.2 Equations of Kinematics in Two Dimensions

Chapter 2 dealt with displacement, velocity, and acceleration in one dimension. But if an
object moves in the horizontal and vertical direction at the same time, we say that the
object is is moving in two dimensions. We subscript any quantity which is horizontal with
an x (such as v x and a x ), and we subscript any quantity which is vertical with a y (such as
v y and a y .)

Example 1 A helicopter moves in such a way that its position at any time is described by
the horizontal and vertical equations

x = 5t + 12t2 and y = 10 + 2t + 6t2 ,

where x and y are in meters and t is in seconds.


(a) What is the initial position of the helicopter at time t = 0?
(b) What are the x and y components of the helicopter’s acceleration at 3 seconds?
(c) What is the speed of the helicopter at 4 seconds?

Solution:
(a) For the initial position, we simply substitute zero for time:

x = 5(0) + 12(0)2 and y = 10 + 2(0) + 6(0)2


yielding x = 0 and y = 10 m at t = 0.
1 2
(b) Notice that both equations are of the familiar form s = s 0 + vo t + at . This means
2
that the acceleration in the equation for x must be 24 m/s2 (that is, ½ (24)t2), and the
acceleration in the equation for y must be 12 m/s2. Thus, a x = 24 m/s2, and a y = 12 m/s2.

(c) The velocity in the x – direction v x would take the form

v x = v ox + a x t = 5 + 24t = 5 + 24(4s) = 101 m/s.

The velocity in the y – direction would take the form

v y = v oy + a y t = 2 + 12t = 2 + 12(4s) = 50 m/s.


Thus, the speed of the helicopter can be found by Pythagoras’ theorem:

v = vx + v y =
2 2
(101 m / s )2 + (50 m / s )2 = 112.7 m/s

38
Chapter 3 Kinematics in Two Dimensions

3.3 Projectile Motion


Projectile motion results when an object is thrown either horizontally through the air or at
an angle relative to the ground. In both cases, the object moves through the air with a
constant horizontal velocity, and at the same time is falling freely under the influence of
gravity. In other words, the projected object is moving horizontally and vertically at the
same time, and the resulting path of the projectile, called the trajectory, has a parabolic
shape. For this reason, projectile motion is considered to be two-dimensional motion.
The motion of a projectile can be broken down into constant velocity and zero
acceleration in the horizontal direction, and a changing vertical velocity due to the
acceleration of gravity. Let’s label any quantity in the horizontal direction with the
subscript x, and any quantity in the vertical direction with the subscript y. If we fire a
cannonball from a cannon on the ground pointing up at an angle θ, the ball will follow a
parabolic path and we can draw the vectors associated with the motion at each point
along the path:

v
vy v
vy v
vx vx
v
vx vy
vx

vy v

At each point, we can draw the horizontal velocity vector v x , the vertical velocity vector
v y , and the vertical acceleration vector g, which is simply the acceleration due to gravity.
Notice that the length of the horizontal velocity and the acceleration due to gravity
vectors do not change, since they are constant. The vertical velocity decreases as the ball
rises and increases as the ball falls. The motion of the ball is symmetric, that is, the
velocities and acceleration of the ball on the way up is the same as on the way down, with
the vertical velocity being zero at the top of the path and reversing its direction at this
point.

39
Chapter 3 Kinematics in Two Dimensions

At any point along the trajectory, the velocity vector is the vector sum of the horizontal
and vertical velocity vectors, that is, v = v x + v y.

vy
v

θ
vx

By the Pythagorean theorem,

v = vx + v y
2 2

and
v x = v cosθ
v y = v sin θ
 vy 
θ = tan −1  
 vx 

In both the horizontal and vertical cases, the acceleration is constant, being zero in the
horizontal direction and 10 m/s2 downward in the vertical direction, and therefore we can
use the kinematic equations to describe the motion of a projectile.

Kinematic Equations for a Projectile

Horizontal motion Vertical motion

ax = 0 a y = g = - 10 m/s2

x v y = v oy + g t
vx =
t

x = vxt 1 2
y = voy t + gt
2

Notice the minus sign in the equations in the right column. Since the acceleration g and
the initial vertical velocity v oy are in opposite directions, we must give one of them a
negative sign, and here we’ve chosen to make g negative. Remember, the horizontal
velocity of a projectile is constant, but the vertical velocity is changed by gravity.

40
Chapter 3 Kinematics in Two Dimensions

Example 2 A golf ball resting on the ground is struck by a golf club and given an initial
velocity of 50 m/s at an angle of 30º above the horizontal. The ball heads toward a fence
12 meters high at the end of the golf course, which is 200 meters away from the point at
which the golf ball was struck. Neglect any air resistance that may be acting on the golf
ball.

50 m/s
12 m
30º

200 m

(a) Calculate the time it takes for the ball to reach the plane of the fence.
(b) Will the ball hit the fence or pass over it? Justify your answer by showing your
calculations.
(c) On the axes below, sketch a graph of the vertical velocity v y of the golf ball vs. time t.
Be sure to label all significant points on each axis.

vy (m/s)

t(s)

41
Chapter 3 Kinematics in Two Dimensions

Solution:
(a) The time it takes for the ball to reach the plane of the fence can be found by

x x 200 m
t= = = = 4.6 s
v x v cos θ (50 m / s ) cos 30°

(b) To determine whether or not the golf ball will strike the fence we need to find the
ball’s vertical position y at the time when the ball is at x = 200 m, that is, at 4.6 seconds.
y = voy t + gt 2 = y = v sin 30t + gt 2 = (50 m / s )sin 30(4.6 s ) + (−10 m / s 2 )(4.6 s )
1 1 1 2

2 2 2
y = 9.7 m

Thus, the ball will strike the fence, since the ball is at a height of less than 12 m when it
reaches the plane of the fence.

(c) The y-component of the ball’s velocity is initially vsin 30 = (50 m/s) sin 30 = 25 m/s.
So the vertical speed would begin at 25 m/s on the vertical axis, and decrease with a
negative slope of 10 m/s2, crossing the time axis when the vertical velocity is zero, that
is, when the ball has reached its maximum height. We can find this time by using the
equation

v y = 0 = v0 y + gt = v sin 30 + gt
v sin 30 (50 m / s )sin 30
t= = = 2.5 s
g 10 m / s 2

The ball’s vertical velocity is negative (downward) after 2.5 s, until it strikes the fence at
4.6 s.

vy (m/s)
25 m/s

2.5 s 4.6 s
t(s)

42
Chapter 3 Kinematics in Two Dimensions

CHAPTER 3 REVIEW QUESTIONS


For each of the multiple choice questions below, choose the best answer.
Unless otherwise noted, use g = 10 m/s2 and neglect air resistance.

1. Which of the following is NOT true of


a projectile launched from the ground at 2. A projectile is launched horizontally
an angle? from the edge of a cliff 20 m high with
(A) The horizontal velocity is constant an initial speed of 10 m/s. What is the
(B) The vertical acceleration is upward horizontal distance the projectile travels
during the first half of the flight, and before striking the level ground below
downward during the second half of the cliff?
the flight. (A) 5 m
(C) The horizontal acceleration is zero. (B) 10 m
(D) The vertical acceleration is 10 m/s2 (C) 20 m
(E) The time of flight can be found by (D) 40 m
horizontal distance divided by (E) 60 m
horizontal velocity.
3. A projectile is launched from level
ground with a velocity of 40 m/s at an
angle of 30° from the ground. What will
be the vertical component of the
projectile’s velocity just before it strikes
the ground? (sin 30° = 0.5, cos 30 =
0.87)
(A) 10 m/s
(B) 20 m/s
(C) 30 m/s
(D) 35 m/s
(E) 40 m/s

43
Chapter 3 Kinematics in Two Dimensions

Questions 4 – 6 5. The acceleration in the x – direction


A toy rocket moves in the horizontal and the y – direction, respectively, are
direction according to the equation x = (A) zero, 3 m/s2
5t, and in the vertical direction according (B) zero, 6 m/s2
to the equation y = 3t2, where x and y are (C) 5 m/s2, 3 m/s2
in meters and t is in seconds. (D) 5 m/s2, 6 m/s2
(E) 5 m/s2, 12 m/s2
4. The length of the displacement vector
of the rocket from the origin (t = 0) at a 6. The horizontal velocity after 10
time of 2 s is most nearly seconds is most nearly
(A) 22 m (A) zero
(B) 2 m (B) 5 m/s
(C) – 2 m (C) 10 m/s
(D) 250 m (D) 50 m/s
(E) 16 m (E) 300 m/s

Free Response Problem

Directions: Show all work in working the following question. The question is worth 10
points, and the suggested time for answering the question is about 10 minutes. The parts
within a question may not have equal weight.

1. (10 points)
Two planetary explorers land on an uncharted planet and decide to test the range of
cannon they brought along. When they fire a cannonball with a speed of 100 m/s at an
angle of 25˚ from the horizontal ground, they find that the cannonball follows a parabolic
path and takes 10 seconds to return to the ground.

(a) Determine the acceleration due to gravity on this uncharted planet.


(b) Determine the maximum height above the level ground the cannonball reaches.
(c) One of the astronauts exclaims that the cannonball “must have landed over a mile
away!” Is the astronaut right? Justify your answer (1 mile = 1600 m).
(d) The astronauts then fire another identical cannonball at 100 m/s at an angle of 75˚ to
the horizontal ground. Will the cannonball travel a horizontal range x′ which is less than,
greater than, or equal to the horizontal range for a 25˚ launch angle?

_____ less than _____ greater than _____ equal to

Justify your answer.

44
Chapter 3 Kinematics in Two Dimensions

ANSWERS AND EXPLANATIONS TO CHAPTER 3 REVIEW QUESTIONS

Multiple Choice

1. B
Since the vertical acceleration is due to gravity, it is always downward.

2. C
First we find the time of flight, which can be calculated from the height:
1 2y 2(20 m )
y = gt 2 , so t = = = 2s
2 g 10 m / s 2
Then, x = v x t = (10 m / s )(2 s ) = 20 m

3. B
Neglecting air resistance, the y – component of the velocity of the projectile just before it
lands is equal to the y – component of the velocity when it is first fired:
v y = (40 m / s )sin 30° = 20 m / s

4. E
At a time of t = 2 s, x = 5(2 s ) = 10 m and y = 3(2 s )2 = 12 m . Then the length of the
displacement vector can e found by Pythagoras’s Theorem:
r = x2 + y2 = (10)2 + (12)2 = 244 ≈ 16 m

5. B
Both the horizontal and vertical components of the displacement of the rocket at any time
1
can be found by the general equation s = s0 + vot + at 2 . Since the equation for x has no
2
t2 term, the horizontal acceleration must be zero. The vertical acceleration can be found
1
by y = at 2 , and since y = 3t2, a = 6 m/s2.
2

6. D
Since x = v x t = 5t , then v x = 5 m/s, which remains constant.

45
Chapter 3 Kinematics in Two Dimensions

Free Response Question Solution

(a) 3 points
Since it takes 10 s to return to the ground, it takes 5s to reach maximum height, at which
point the vertical velocity v y = 0. Thus,
v y = voy − gt
voy vo sin 25 (100 m / s )sin 25
g= = = = 8.5 m / s 2
t t 5s

(b) 3 points
y max = gt 2 = (8.5 m / s 2 )(5s ) = 105 m
1 1 2

2 2
(c) 2 points
x = vox t = v cos 25 t = (100 m / s ) cos 25(10 s ) = 906 m , and so it lands less than a mile away
from where it was launched.

(d) 2 points
Two launch angles which are complementary, i.e., whose sum is 90˚, will produce the
same horizontal range x for a particular initial velocity. The complement of 25˚ is 65˚.
Since the new launch angle is greater than 65˚, the horizontal component of the velocity
for the 75˚ launch angle will be less than that of a 65˚ (and 25˚) launch angle, and
therefore the horizontal range x′ will be less for the 75˚ launch angle.

46
Chapter 4 Forces and Newton’s Laws of Motion

Chapter 4

FORCES AND NEWTON’S LAWS OF MOTION

PREVIEW

Dynamics is the study of the causes of motion, in particular, forces. A force is a push or a
pull. We arrange our knowledge of forces into three laws formulated by Isaac Newton:
the law of inertia, the law of force and acceleration (F net = ma), and the law of action
and reaction. Friction is the force applied by two surfaces parallel to each other, and the
normal force is the force applied by two surfaces perpendicular to each other. Newton’s
law of universal gravitation states that all masses attract each other with a gravitational
force which is proportional to the product of the masses and inversely proportional to the
square of the distance between them. The gravitational force holds satellites in orbit
around a planet or star.
The content contained in all sections of chapter 4 of the textbook is included on the AP
Physics B exam.

QUICK REFERENCE

Important Terms

coefficient of friction
the ratio of the frictional force acting on an object to the normal force exerted by
the surface in which the object is in contact; can be static or kinetic
dynamics
the study of the causes of motion (forces)
equilibrium
the condition in which there is no unbalanced force acting on a system, that is, the
vector sum of the forces acting on the system is zero.
force
any influence that tends to accelerate an object; a push or a pull
free body diagram
a vector diagram that represents all of the forces acting on an object
friction
the force that acts to resist the relative motion between two rough surfaces which
are in contact with each other
gravitational field
space around a mass in which another mass will experience a force
gravitational force
the force of attraction between two objects due to their masses

47
Chapter 4 Forces and Newton’s Laws of Motion

inertia
the property of an object which causes it to remain in its state of rest or motion at
a constant velocity; mass is a measure of inertia
inertial reference frame
a reference frame which is at rest or moving with a constant velocity; Newton’s
laws are valid within any inertial reference frame
kinetic friction
the frictional force acting between two surfaces which are in contact and moving
relative to each other
law of universal gravitation
the gravitational force between two masses is proportional to the product of the
masses and inversely proportional to the square of the distance between them.
mass
a measure of the amount of substance in an object and thus its inertia; the ratio of
the net force acting on an accelerating object to its acceleration
net force
the vector sum of the forces acting on an object
newton
the SI unit for force equal to the force needed to accelerate one kilogram of mass
by one meter per second squared
non-inertial reference frame
a reference frame which is accelerating; Newton’s laws are not valid within a
non-inertial reference frame.
normal force
the reaction force of a surface acting on an object
static friction
the resistive force that opposes the start of motion between two surfaces in
contact
weight
the gravitational force acting on a mass

Equations and Symbols

ΣF = Fnet = ma where
W = mg
F = force
f s max ≤ µ s FN ( static ) m = mass
f k = µ k FN (kinetic) a = acceleration
W = weight
Gm1 m2
FG = g = acceleration due to gravity
r2 f s max = maximum static frictional force
f k = kinetic frictional force
F N = normal force
F G = gravitational force
r = distance between the centers of two
masses

48
Chapter 4 Forces and Newton’s Laws of Motion

Ten Homework Problems


Chapter 4 Problems 23, 24, 36, 44, 52, 58, 67, 68, 70, 80

DISCUSSION OF SELECTED SECTIONS

Newton’s Laws of Motion

The first law of motion states that an object in a state of constant velocity (including zero
velocity) will continue in that state unless acted upon by an unbalanced force. The
property of the book which causes it to follow Newton’s first law of motion is its inertia.
Inertia is the sluggishness of an object to changing its state of motion or state of rest. We
measure inertia by measuring the mass of an object, or the amount of material it contains.
Thus, the SI unit for inertia is the kilogram. We often refer to Newton’s first law as the
law of inertia.

The law of inertia tells us what happens to an object when there are no unbalanced forces
acting on it. Newton’s second law tells us what happens to an object which does have an
unbalanced force acting on it: it accelerates in the direction of the unbalanced force.
Another name for an unbalanced force is a net force, meaning a force which is not
canceled by any other force acting on the object. Sometimes the net force acting on an
object is called an external force.

Newton’s second law can be stated like this: A net force acting on a mass causes that
mass to accelerate in the direction of the net force. The acceleration is proportional to
the force (if you double the force, you double the amount of acceleration), and inversely
proportional to the mass of the object being accelerated (twice as big a mass will only be
accelerated half as much by the same force). In equation form, we write Newton’s second
law as

F net = ma

where F net and a are vectors pointing in the same direction. We see from this equation
that the newton is defined as a
(kg )m .
s2

The weight of an object is defined as the amount of gravitational force acting on its mass.
Since weight is a force, we can calculate it using Newton’s second law:

F net = ma becomes Weight = mg,

where the specific acceleration associated with weight is, not surprisingly, the
acceleration due to gravity. Like any force, the SI unit for weight is the newton.

49
Chapter 4 Forces and Newton’s Laws of Motion

Newton’s third law is sometimes called the law of action and reaction. It states that for
every action force, there is an equal and opposite reaction force. For example, let’s say
your calculator weighs 1 N. If you set it on a level table, the calculator exerts 1 N of force
on the table. By Newton’s third law, the table must exert 1 N back up on the calculator. If
the table could not return the 1 N of force on the calculator, the calculator would sink into
the table. We call the force the table exerts on the calculator the normal force. Normal is
another word for perpendicular, because the normal force always acts perpendicularly to
the surface which is applying the force, in this case, the table. The force the calculator
exerts on the table, and the force the table exerts on the calculator are called an action-
reaction pair.

4.3 – 4.4 Newton’s Second Law of Motion and the Vector Nature of
Newton’s Second Law of Motion

Since force is a vector quantity, we may break forces into their x and y components. The
horizontal component of a force can cause a horizontal acceleration, and the vertical
component of a force can cause a vertical acceleration. These horizontal and vertical
components are independent of each other.

Example 1 A forklift lifts a 20-kg box with an upward vertical acceleration of 2.0 m/s2,
while pushing it forward with a horizontal acceleration of 1.5 m/s2.

(a) Draw a free-body diagram for the box on the diagram below.

(b) What is the magnitude of the horizontal force F x acting on the box?
(c) What is the magnitude of the upward normal force F N the platform exerts on the box?
(d) If the box starts from rest at ground level (x = 0, y = 0, and v = 0) at time t = 0, write
an expression for its vertical position y as a function of horizontal distance x.
(e) On the axes below, sketch a y vs x graph of the path which the box follows. Label all
significant points on the axes of the graph.

50
Chapter 4 Forces and Newton’s Laws of Motion

y(m)

Solution:
FN

ay = 2.0 m/s2

Fx
ax = 1.5 m/s2
(a)

W
(b) The horizontal force F x exerted by the wall causes the horizontal acceleration a x = 1.5
m/s2. Thus, the magnitude of the horizontal force is

F x = ma x = (200 kg)(1.5 m/s2) = 300 N

(c) In order to accelerate the box upward at 2.0 m/s2, the normal force F N must first
overcome the downward weight of the box. Writing Newton’s second law in the vertical
direction gives

F net y = ma y

(F N – W) = ma y

(F N – mg) = ma y

F N = ma y + mg = (200 kg)(2.0 m/s2) + (200 kg)(9.8 m/s2) = 2360 N

51
Chapter 4 Forces and Newton’s Laws of Motion

(d) Since the box starts from rest on the ground, we can write

1 1
x= a x t 2 and y = a y t 2
2 2

Substituting for a x and a y , we get

x=
1
2
( ) 1
(
1.5 m / s 2 t 2 and y = 2.0 m / s 2 t 2
2
)
Solving both sides for t and setting the equations equal to each other yields

2.0 4
y= x= x
1.5 3

(e) The graph of y vs x would be linear beginning at the origin of the graph and having a
4
positive slope of :
3

y(m)

3 x(m)

4.7 The Gravitational Force

Newton’s law of universal gravitation states that all masses attract each other with a
gravitational force which is proportional to the product of the masses and inversely
proportional to the square of the distance between them. The gravitational force holds
satellites in orbit around a planet or star.

The equation describing the gravitational force is

Gm1 m2
FG =
r2
where F G is the gravitational force, m 1 and m 2 are the masses in kilograms, and r is the
distance between their centers. The constant G simply links the units for gravitational
force to the other quantities, and in the metric system happens to be equal to 6.67 x 10-11
Nm2/kg2. Like several other laws in physics, Newton’s law of universal gravitation is an
inverse square law, where the force decreases with the square of the distance from the
centers of the masses.

52
Chapter 4 Forces and Newton’s Laws of Motion

Example 2 An artificial satellite of mass m 1 = 400 kg orbits the earth at a distance


r = 6.45 x 106 m above the center of the earth. The mass of the earth is m 2 = 5.98 x 1024
kg. Find (a) the weight of the satellite and (b) the acceleration due to gravity at this
orbital radius.

Solution (a) The weight of the satellite is equal to the gravitational force that the earth
exerts on the satellite:

Gm1 m2 (6.67 x10 −11 kg )(400 kg )(5.98 x10 24 kg )


FG = = = 3835 N
r2 (6.45 x10 6 m) 2

(b) The acceleration due to gravity is

W FG 3835 N m
g= = = = 9.59 2
m1 m1 400 kg s

Note that even high above the surface of the earth, the acceleration due to gravity is not
zero, but only slightly less than at the surface of the earth.

4.8 – 4.9 The Normal Force, Static and Kinetic Frictional Forces
The normal force F N is the perpendicular force that a surface exerts on an object. If a box
sits on a level table, the normal force is simply equal to the weight of the box:

FN

If the box were on an inclined plane, the normal force would be equal to the component
of the weight of the box which is equal and opposite to the normal force:
FN
y
mgsinθ
x
mgcosθ
θ

mg θ

53
Chapter 4 Forces and Newton’s Laws of Motion

In this case, the component of the weight which is equal and opposite to the normal force
is mgcos θ.

Friction is a resistive force between two surfaces which are in contact with each other.
There are two types of friction: static friction and kinetic friction. Static friction is the
resistive force between two surfaces which are not moving relative to each other, but
would be moving if there were no friction. A block at rest on an inclined board would be
an example of static friction acting between the block and the board. If the block began to
slide down the board, the friction between the surfaces would no longer be static, but
would be kinetic, or sliding, friction. Kinetic friction is typically less than static friction
for the same two surfaces in contact.

The ratio of the frictional force between the surfaces divided by the normal force acting
on the surfaces is called the coefficient of friction. The coefficient of friction is
represented by the Greek letter µ (mu). Equations for the coefficients of static and kinetic
friction are

f s max fk
µs = and µ k = , where f s is the static frictional force and f k is the kinetic
FN FN
frictional force. Note that the coefficient of static friction is equal to ratio of the maximum
frictional force and the normal force. The static frictional force will only be as high as it
has to be to keep a system in equilibrium.

When you draw a free body diagram of forces acting on an object or system of objects,
you would want to include the frictional force as opposing the relative motion (or
potential for relative motion) of the two surfaces in contact.

Example 3 A block of wood rests on a board. One end of the board is slowly lifted until
the block just begins to slide down. At the instant the block begins to slide, the angle of
the board is θ. What is the relationship between the angle θ and the coefficient of static
friction μ s ?

Solution Let’s draw the free-body diagram for the block on the inclined plane:

FN
f
y
mgsinθ
x
mgcosθ
θ

mg θ

54
Chapter 4 Forces and Newton’s Laws of Motion

At the instant the block is just about to move, the maximum frictional force directed up
the incline is equal and opposite to the +x-component of the weight down the incline, and
the normal force is equal and opposite to the y-component of the weight.

f s max = mg sin θ
FN = mg cos θ
Then
f s max mg sin θ
µs = = = tan θ
FN mg cos θ

This expression is only valid for the case in which the static frictional force is maximum.

Example 4 After the block in Example 3 just begins to move, should the board be
lowered or raised to keep the block moving with a constant velocity down the incline?
Explain your answer.

Solution Since the coefficient of kinetic friction is generally less than the coefficient of
static friction for the same two surfaces in contact, the block would require less force
directed down the incline (mgsin θ) to keep it sliding at a constant speed. Thus, the board
should be lowered to a smaller θ just after the block begins to slide to keep the block
moving with a constant velocity.

4.10 – 4.11 The Tension Force, Equilibrium Applications of Newton’s


Laws of Motion

The force in a rope or cable that pulls on an object is called the tension force. Like any
other force, tension can accelerate or contribute to the acceleration of an object or system
of objects.

Example 5 An elevator cable supports an empty elevator car of mass 300 kg. Determine
the tension in the cable when the elevator car is (a) at rest and (b) the car has a downward
acceleration of 2.0 m/s2.

55
Chapter 4 Forces and Newton’s Laws of Motion

Solution (a) The free-body diagram for the car would look like this:

FT

When the elevator car is at rest, the tension in the cable F T is equal to the weight W of the
car:

F T = W = mg = (300 kg)(10 m/s2) = 3000 N.

(b) When the elevator car is being lowered with an acceleration of 2.0 m/s2, the
downward weight force is greater than the upward tension force. We can use Newton’s
second law to find the tension in the cable. Choosing the downward direction as positive,

ΣF = ma
(W − FT ) = ma
m
FT = W − ma = (3000 N ) − (300 kg )(2.0 ) = 2400 N upward.
s2

A system is said to be static if it has no velocity and no acceleration. According to


Newton’s first law, if an object is in static equilibrium, the net force on the object must be
zero.

56
Chapter 4 Forces and Newton’s Laws of Motion

Example 6 Three ropes are attached as shown below. The tension forces in the ropes are
T 1 , T 2 , and T 3 , and the mass of the hanging ball is m= 3.0 kg.

30º 40º
T2
T3

T1

Since the system is in equilibrium, the net force on the system must be zero. We can find
the tension in each of the three ropes by finding the vector sum of the tensions and setting
this sum equal to zero:

T2 T2y
T3y
T3
T3x

T2x
T1
T1

T1 + T2 + T3 = 0

Since T 1 is in the negative y-direction, T 1 must be equal to the weight mg of the mass. As
we resolve each tension force into its x- and y-components, we see that

T 1 = mg = (3.0 kg)(10 m/s2) = 30 N

T 2x = T 2 cos40 and T 2y = T 2 sin40

T 3x = T 3 cos30 and T 3y = T 3 sin30

Since the forces are in equilibrium, the vector sum of the forces in the x-direction must
equal zero:

ΣF x = 0

57
Chapter 4 Forces and Newton’s Laws of Motion

T 2x = T 3x

T 2 cos40 = T 3 cos30

T3 cos 30
T2 = = 1.13T3
cos 40

The sum of the forces in the y-direction must also be zero:

ΣF y = 0

T 2y + T 3y = W

T 2 sin40 + T 3 sin30 = W

Knowing that T2 = 1.13T3 , we can solve the equations above for each of the tensions in
the ropes:

(1.13T3 )sin 40 + T3 sin 30 = mg


(3.0kg )10 m2 
T3 =
mg
=  s  = 24.4 N
1.13 sin 40 + sin 30 1.23

T2 = 1.13T3 = 1.13(24.4 N ) = 27.6 N

4.12 Nonequilibrium Applications of Newton’s Laws of Motion


If the net force acting on a system is not zero, the system must accelerate. A common
example used to illustrate Newton’s second law is a system of blocks and pulley.

Example 7 In the diagram below, two blocks of mass m 1 = 2 kg and m 2 = 6 kg are


connected by a string which passes over a pulley of negligible mass and friction. What is
the acceleration of the system?

2 kg
6 kg

58
Chapter 4 Forces and Newton’s Laws of Motion

Solution Let’s draw a free-body force diagram for each block. There are two forces
acting on each of the masses: weight downward and the tension in the string upward. Our
free-body force diagrams should look like this:

T T

W1

W2

Writing Newton’s second law for each of the blocks:

ΣF1 = m1a
T − m1 g = m1a
T = m1 g + m1a
and
ΣF2 = m2a
m2 g − T = m2 a
T = m2 g − m2 a

Notice that the tension T acting on the 2 kg block is greater than block its weight, but the
6 kg block has a greater weight than the tension T. This is, of course, the reason the 6 kg
block accelerates downward and the 2 kg block accelerates upward. The tension acting on
each block is the same, and their accelerations are the same. Setting their tensions equal
to each other, we get

m1 g + m1a = m2 g − m2 a

Solving for a, we get

m
a = 5.0
s2

59
Chapter 4 Forces and Newton’s Laws of Motion

Example 8 A block of mass m 1 = 2 kg rests on a horizontal table. A string is tied to the


block, passed over a pulley, and another block of mass m 2 = 4 kg is hung on the other end
of the string, as shown in the figure below. The coefficient of kinetic friction between the
2 kg block and the table is 0.2. Find the acceleration of the system.
2 kg

μk = 0.2

4 kg

Solution Once again, let’s draw a free-body force diagram for each of the blocks, and
then apply Newton’s second law.

FN
T

fk

W1
W2

60
Chapter 4 Forces and Newton’s Laws of Motion

Block 1:

ΣF1 = m1a Block 2:


T − f k = m1 a
ΣF2 = m2a
T − µ k FN = m1 a
W2 − T = m2 a
Since FN = m1 g ,
m2 g − T = m2 a
T − µ k m1 g = m1 a
T = m2 g − m2 a
T = µ k m1 g + m1 a
Setting the two equations for T equal to each other:

µ k m1 g + m1 a = m2 g − m2 a

Solving for a and substituting the values into the equation, we get

(4kg )10 m2  − (0.2)(2kg )10 m2 


m2 g − µ k m1 g  s   s  m
a= = =6 2
m1 + m2 2kg + 4kg s

Example 9 Three blocks of mass m 1 , m 2 , and m 3 are connected by a string passing over
pulley attached to a plane inclined at an angle θ as shown below.

m2

m1

m3 θ
θ

The friction between each block on the inclined plane and the surface of the plane has a
magnitude f.

(a) Draw the forces acting on each block.


(b) Assuming that m 3 is large enough to descend and cause the system to accelerate,
determine the acceleration of the system in terms of the given quantities and fundamental
constants.

61
Chapter 4 Forces and Newton’s Laws of Motion

(c) As m 2 reaches the top of the inclined plane, it comes to rest up against the pulley. The
vertical string supporting m 3 is cut, and m 1 and m 2 begin to slide down the incline.
Assuming m 1 = m 2 , what is the speed of the blocks after they have slid a distance d
down the plane?

Solution (a)

FN2

T2 FN1
T1
T2 T1

f
W2

W1

W3

(b) Since the pulley serves only to change the direction of the tension in the string that
passes over it, we may find the acceleration of the system by treating the masses as if
they lie in a straight line and are accelerated by the weight of the hanging block m 3 :
a

m3
m2 m1 2W1 sinθ
W3
2f

Newton’s second law:

ΣF = ma
W3 − 2W1 sin θ − 2 f = (m1 + m2 + m3 )a
W3 − 2W1 sin θ − 2 f
a=
(m1 + m2 + m3 )

62
Chapter 4 Forces and Newton’s Laws of Motion

(c) Since m 1 = m 2 , and friction f acts equally on both, they slide down the plane with
equal acceleration and speed. Thus, the two blocks slide down the incline as if they were
one block of mass m 1 + m 2 accelerated down the plane by a force of (m 1 + m 2 )gsin θ
against a frictional force of 2f directed up the plane.

(m1+m2)gsinθ

2f

Newton’s second law for the system is

ΣF = ma
(m1 + m2 )g sin θ − 2 f = (m1 + m2 )a
(m1 + m2 )g sin θ − 2 f
a=
(m1 + m2 )
The speed can be found by the kinematic equation
v f = vi + 2ad
2 2

Since the initial velocity is zero,


1
  (m + m2 )g sin θ − 2 f  2
v f = 2ad = 2  1 d 
  (m1 + m2 )  

63
Chapter 4 Forces and Newton’s Laws of Motion

CHAPTER 4 REVIEW QUESTIONS


For each of the multiple choice questions below, choose the best answer.
Unless otherwise noted, use g = 10 m/s2 and neglect air resistance.

1. The amount of force needed to keep a


30 45
0.2 kg hockey puck moving at a constant
speed of 7 m/s on frictionless ice is T2
T3
(A) zero
(B) 0.2 N
(C) 0.7 N T1
(D) 7 N
(E) 70 N W=30N

2. A force of 26 N is needed to 4. Which of the following is true of the


overcome a frictional force of 5 N to magnitudes of tensions T 1 , T 2 , and T 3
accelerate a 3 kg mass across a floor. in the ropes in the diagram shown
What is the acceleration of the mass? above?
(A) 4 m/s2 (A) T 1 must be greater than 30 N.
(B) 5 m/s2 (B) The tension T 2 is greater than T 1 .
(C) 7 m/s2 (C) The y-component of T 2 and T 3 is
(D) 20 m/s2 equal to 30 N.
(E) 60 m/s2 (D) The sum of the magnitudes of T 2
and T 3 is equal to T 1 .
3. A force of 100 N directed at an angle (E) The sum of the magnitudes of T 1
of 45° from the horizontal pulls a 70 kg and T 2 is equal to T 3 .
sled across a frozen frictionless pond.
The acceleration of the sled is most 5. Two blocks of mass m and 5m are
nearly (sin 45° = cos 45° = 0.7) connected by a light string which passes
(A) 1.0 m/s2 over a pulley of negligible mass and
(B) 0.7 m/s2 friction. What is the acceleration of the
(C) 7 m/s2 masses in terms of the acceleration due
(D) 35 m/s2 to gravity, g?
(E) 50 m/s2 (A) 4g
(B) 5g
(C) 6g
(D) 4/5 g
(E) 2/3 g

64
Chapter 4 Forces and Newton’s Laws of Motion

6. A 1-kg block rests on a frictionless 9. Which of the following diagrams of


table and is connected by a light string to two planets would represent the largest
another block of mass 2 kg. The string is gravitational force between the masses?
passed over a pulley of negligible mass
and friction, with the 2 kg mass hanging (A)
vertically. What is the acceleration of m m
the masses?
(A) 5 m/s2 r
(B) 6.7 m/s2 c
(C) 10 m/s2
(D) 20 m/s2 (B) m m
(E) 30 m/s2
2r
7. Friction
(A) can only occur between two surfaces
2m
which are moving relative to one (C) m
another.
(B) is equal to the normal force divided 2r
by the coefficient of friction.
(C) opposes the relative motion between
the two surfaces in contact. 2m
(D) m
(D) only depends on one of the surfaces
in contact. r
(E) is always equal to the applied force.

(E) 2m 2m

2r

30˚
10. A satellite is in orbit around the
earth. Consider the following quantities:
8. A 2-kg wooden block rests on an I. distance from the center
inclined plane as shown above. The of the earth
frictional force between the block and II. mass of the earth
the plane is most nearly (sin 30 = 0.5, III. mass of the satellite
cos 30 = 0.87, tan 30 = 0.58)
(A) 2 N The gravitational acceleration g depends
(B) 10 N on which of the above?
(C) 12 N (A) I only
(D) 17 N (B) I and II only
(E) 20 N (C) III only
(D) I and III only
(E) I, II, and III

65
Chapter 4 Forces and Newton’s Laws of Motion

Free Response Question


Directions: Show all work in working the following question. The question is worth 15 points,
and the suggested time for answering the question is about 15 minutes. The parts within a
question may not have equal weight.

1. (15 points)
A block of mass m rests on an air table (no friction), and is pulled with a force probe, producing
the Force vs. acceleration graph shown below.

(a) Determine the mass of the block.

The block is now placed on a rough horizontal surface having a coefficient of static friction μ s =
0.2, and a coefficient of kinetic (sliding) friction μ k = 0.1.

(b) What is the minimum value of the force F which will cause the block to just begin to move?

F
L

F
θ

μs=0.2 ; μk=0.1

66
Chapter 4 Forces and Newton’s Laws of Motion

(c) After the block begins to move, the same force determined in part (b) continues to act
on the block. What is the acceleration of the block?

(d) The force F is now tripled to 3F, which then pulls the block up an incline of angle
θ = 20˚ and having a coefficient of kinetic friction μ k = 0.1.

i. Draw the free-body diagram for the block as it is being pulled up the incline.

ii. Determine the magnitude of the frictional force f k acting on the block as it
slides up the incline.

iii. Determine the acceleration of the block as it is pulled up the incline.

ANSWERS AND EXPLANATIONS TO CHAPTER 4 REVIEW QUESTIONS

Multiple Choice

1. A
The law of inertia states that no net force is needed to keep an object moving at a constant
velocity.

2. C
The net force is 26 N – 5 N = 21 N, and the acceleration is
F 21 N
a = net = = 7 m / s2
m 3 kg
3. A
Only the x-component of the force accelerates the sled horizontally: F x =(100 N)cos45˚= 70 N.
F 70 N
Then a = x = = 1m / s 2
m 70 kg

67
Chapter 4 Forces and Newton’s Laws of Motion

4. C
T 2y + T 3y must equal the weight of the block (30 N), since the system is in equilibrium.

5. E
The net force acting on the system is 5mg – mg = 4 mg. Then
Fnet = 4mg = (5 + 1)ma , and a = g .
2
3

6. B
The weight of the 2 kg block is the net force accelerating the entire system.
2
2mg = 3ma , so a = g = 6.7 m/s2.
3

7. C
Friction acts on each of the surfaces in contact that are moving or have the potential for moving
relative to each other.

8. B
Since the block is in static equilibrium, the frictional force must be equal and opposite to the
component of the weight pointing down the incline:
( )
f = mg sin θ = (2kg ) 10 m / s 2 sin 30 = 10 N

9. D
The greater the mass, and the smaller the separation distance r, the greater the force according to
Newton’s law of universal gravitation.

10. B
GM E
Since the acceleration due to gravity g = , it does not depend on the mass of the satellite.
r2

Free Response Problem Solution

(a) 2 points
Fnet
Newton’s 2nd law states that m = . This ratio is the slope of the F vs. a graph. So,
a
∆F 15 N − 0
m = slope = = = 3kg
∆a 5 m / s 2 − 0
(b) 2 points
The block will just begin to move when the force F overcomes the maximum static frictional
force:
Fmin = f s = µ s FN = µ s mg = (0.2 )(3kg )(10 m / s 2 ) = 6 N

68
Chapter 4 Forces and Newton’s Laws of Motion

(c) 3 points
Once the block begins to move we must use the coefficient of kinetic friction to determine the
frictional force.
f k = µ k FN = µ k mg = (0.1)(3kg )(10 m / s 2 ) = 3 N
Then the net force acting on the block is F – f k = 6 N – 3 N = 3 N to the right.
The acceleration of the block is
F 3N
a = net = = 1m / s 2
m 3 kg
FN
(d) 3F

i. 3 points
mgsinθ

fk
mgcosθ

mg

ii. 2 points
As the block sides up the incline, the normal force F N is no longer equal to mg, but mgcosθ.

( )
f k = µ k FN = µ k mg cos θ = (0.1)(3kg ) 10 m / s 2 cos 20 = 2.8 N

iii. 3 points
The net force is now

( )
3F − f k − mg sin θ = 3(6 N ) − 2.8 N − (3kg ) 10 m / s 2 sin 20 = 5 N
Fnet 5 N
a= = = 1.67 m / s 2
m 3 kg

69
Chapter 5 Dynamics of Uniform Circular Motion

Chapter 5

DYNAMICS OF UNIFORM CIRCULAR MOTION

PREVIEW

An object which is moving in a circular path with a constant speed is said to be in


uniform circular motion. For an object to move in a circular path, there must be a force
exerted on the object which is directed toward the center of the circular path called the
centripetal force. This centripetal force gives rise to centripetal acceleration. This
centripetal force can be provided by tension in string, friction acting between tires and the
road, or the gravitational force holding a satellite in orbit around a planet or star.

The content contained in sections 1 – 4 and 5 – 8 of chapter 5 of the textbook is included


on the AP Physics B exam.

QUICK REFERENCE

Important Terms

centripetal acceleration
the acceleration of an object moving in circular motion which is directed toward
the center of the circular path
centripetal force
the central force causing an object to move in a circular path
period
the time for one complete cycle or revolution
satellite motion
the motion of a satellite orbiting under the influence of gravity in a circular or
elliptical path around a larger mass
tangential velocity
the velocity tangent to the path of an object moving in a curved path
uniform circular motion
motion in a circular path of constant radius at a constant speed

70
Chapter 5 Dynamics of Uniform Circular Motion

Equations and Symbols

2πr where
v=
T
v = velocity or speed of an object in
v2
ac = circular motion or orbiting satellite
r r = radius of circular motion or orbit
mv 2 T = period of revolution
Fc =
r a c = centripetal acceleration
F c = centripetal force
GM E
v= m = mass
r G = gravitational constant = 6.67 x 10-11
N m2 / kg2
M E = mass of the earth = 5.98 x 1024 kg

Ten Homework Problems


Chapter 5 Problems 7, 8, 13, 18, 30, 36, 38, 40, 49, 54

DISCUSSION OF SELECTED SECTIONS

5.1 – 5.3 Uniform Circular Motion, Centripetal Acceleration and


Centripetal Force

The law of inertia states that if an object is moving it will continue moving in a straight
line at a constant velocity until a net force causes it to speed up, slow down, or change
direction. If a force on an object is constantly applied toward a central point, the object
will be pulled away from its straight-line path, and will follow a circular path.
As long as the string is pulled toward this central point, the ball will continue moving in a
circle at a constant speed. An object moving in a circle at a constant speed is said to be in
uniform circular motion (UCM). Notice that even though the speed is constant, the
velocity vector is not constant, since it is always changing direction due to the central
force that the string applies to the ball, which we will call the centripetal force F c .
Centripetal means “center-seeking”. The three vectors associated with uniform circular
motion are velocity v, centripetal force F c , and centripetal acceleration a c . These vectors
are drawn in the diagram of a ball on the end of a string shown below.

Fc ac

71
Chapter 5 Dynamics of Uniform Circular Motion

Notice that the velocity vector is tangent to the path of the ball, and points in the direction
the ball would move if the string were to break at that instant. The centripetal force and
acceleration are both pointing toward the center. Although the centripetal force does not
change the speed of the ball, it accelerates the ball toward the center of the circle,
constantly changing its direction. The time it takes for the ball to complete one revolution
is called the period T. Since period is a time, we will measure it in seconds, minutes,
hours, or even years. On the other hand, frequency f is the number of revolutions the ball
makes per unit time. Units for frequency would include
revolutions revolutions
, , or any time unit divided into revolutions or cycles. Another
sec ond hour
name for rev/s is hertz. The relationships between period, frequency, speed, centripetal
force, and centripetal acceleration are listed in the Equations and Symbols list above.

Some other examples of centripetal force is the gravitational force keeping a satellite in
orbit, and friction between a car’s tires and the road which causes the car to turn in a
circle.

5.5 Satellites in Circular Orbits


As a satellite orbits the earth, it is pulled toward the earth with a gravitational force which
is acting as a centripetal force. The inertia of the satellite causes it to tend to follow a
straight-line path, but the centripetal gravitational force pulls it toward the center of the
orbit.
m
v
FG= FC

72
Chapter 5 Dynamics of Uniform Circular Motion

If a satellite of mass m moves in a circular orbit around a planet of mass M, we can set
the centripetal force equal to the gravitational force and solve for the speed of the satellite
orbiting at a particular distance r:

Fc = FG
mv 2 GmM
=
r r2
GM
v=
r
Example 1 Venus orbits the sun at a radius of 1.08 x 1011 m with an orbital period of
228 days. Find the mass of the sun in kg.

Solution:
For consistency, let’s convert days to seconds:
(228 days ) 24 hr  3600 s  = 1.97 x 10 7 s
 day  hr 

GM 2π r
v= =
r T
GM 4π r 2 2
=
r T2

M =
4π 2 r 3
=
(
4π 2 1.08 x1011 m )3

= 1.92 x 10 30 kg
 Nm 2 
GT 2
 6.67 x10 −11 2 
(
 1.97 x10 7 s )
2

 kg 

The table of constants in the front of your textbook lists the mass of the sun as
1.99 x 1030 kg.

73
Chapter 5 Dynamics of Uniform Circular Motion

5.7 Vertical Circular Motion


Consider a ball attached to the end of a string and swung in a vertical circle. Because of
the gravitational force acting on the ball, it will increase its speed slightly as it falls from
point A to point B, and decreases its speed slightly as it rises from point B to point A.
This also implies that the tension in the string increases as the ball falls and decreases as
the ball rises.
A

In this case, the centripetal force is not merely the tension force in the string. Since
centripetal force is always the net force acting on an object following a circular path, the
centripetal force in this case is a combination of the tension force in the string and the
weight of the ball. Notice that the tension force and weight vectors at point A (top) are in
the same direction and their magnitudes can be added to find the centripetal force at point
A, and the tension force and weight vectors at point B (bottom) are in the opposite
direction and their magnitudes can be subtracted to find the centripetal force at point B.
This implies that the tension force must be greater at the bottom of the circle than at the
top. Try working the free response question below, and check your answer for an
illustration of the forces involved in a vertical circle.

74
Chapter 5 Dynamics of Uniform Circular Motion

CHAPTER 5 REVIEW QUESTIONS


For each of the multiple choice questions below, choose the best answer.
Unless otherwise noted, use g = 10 m/s2 and neglect air resistance.

1. A ball on the end of a string is swung


in a horizontal circle, rotating clockwise 2. If the string were suddenly cut when
as shown. A, When the ball is at a the ball is at the position shown in
particular point in the circle, the answer (E) in the previous question, the
direction of the velocity, centripetal subsequent motion of the ball would be
force, and centripetal acceleration (A) to move to the right.
vectors respectively are shown below: (B) to move to the left.
(C) to move to the top of the page.
(D) to move to down and to the right
(E) to move up and to the left.

v Questions 3 - 5:
F a A 40 kg child sits on the edge of a
Which of the following best represents carnival ride at a radius of 4 m.
the position of the ball as it rotates The ride makes 3 revolutions in 6 s.
clockwise?
Top View 3. The period of revolution for this ride
(A) is
(A) ½ rev/s
(B) ½ s
(C) 2 rev/s
(B) (D) 2 s
(E) 4 s

4. The speed of the child is most nearly


(A) 4 m/s
(B) 12 m/s
(C) (C) 24 m/s
(D) 120 m/s
(E) 360 m/s

5. The force which is holding the child


(D) on the ride is most nearly
(A) 30 N
(B) 160 N
(C) 320 N
(D) 1440 N
(E) (E) 2880 N

75
Chapter 5 Dynamics of Uniform Circular Motion

Questions 6 – 7: 7. The car has a mass m and a speed v as


it moves around the track of radius R.
The figure below shows the top view of Which of the following expressions can
a car going around a horizontal circular be used to find the value of the
track at a constant speed in the coefficient of friction between the tires
counterclockwise direction. Assume the and the road?
frictional force between the tires and the
road is at its maximum value. gR
(A) µ =
v

gv
(B) µ =
R

(C) µ = vgR

v2
6. Which of the following vectors (D) µ =
represents the frictional force acting on gR
the tires of the car?
mv 2
(E) µ =
(A) R

(B) 8. The acceleration due to gravity g at a


distance r from the center of a planet of
(C) mass M is 9 m/s2. In terms of the orbital
distance r, what would the speed of this
satellite have to be to remain in a
(D) circular orbit around this planet at this
distance?

(E) (A) v = 3 r

(B) v = 3r

(C) v = 9 r

(D) v = 9r

(E) v = 33 r

76
Chapter 5 Dynamics of Uniform Circular Motion

9. In general, (C) the larger the orbital radius of a


satellite, the longer its orbital period.
(A) the smaller the orbital radius of a (D) the smaller the orbital radius of a
satellite, the longer its orbital period. satellite, the smaller its acceleration.
(B) the larger the orbital radius of a (E) the larger the orbital radius of a
satellite, the shorter its orbital period. satellite, the greater the gravitational
force acting on it.

Free Response Question


Directions: Show all work in working the following question. The question is worth 15
points, and the suggested time for answering the question is about 15 minutes. The parts
within a question may not have equal weight.

1. (15 points)

A ball on the end of a string is swung in a vertical circle. The mass m of the ball is 0.25
kg and the radius of the circle R = 0.75 m. The position of the ball is marked every
quarter of a revolution in the diagram below.

R
IV II

III

77
Chapter 5 Dynamics of Uniform Circular Motion

(a) On the diagram below, draw all of the forces acting on the ball when it is at position
IV. Be sure and label each force.

(b) When the ball is at position IV, the tension force in the string is twice as great as the
weight of the ball. Determine the speed of the ball at position IV.

(c) On the diagrams below, draw and label the forces acting on the ball
i. at the top of the circle (position I) and
ii. at the bottom of the circle (position III).

Position I Position III

(d) The speed of the ball at position III (the bottom of the circle) is 5 m/s. Determine the
tension force in the string as the ball passes through position III.

(e) Suppose the string breaks just as he ball is at position IV.


i. Describe the subsequent motion of the ball.
ii. Use the speed you calculated in part (b) to determine the maximum height the
ball reaches after the string breaks.

78
Chapter 5 Dynamics of Uniform Circular Motion

ANSWERS AND EXPLANATIONS TO CHAPTER 4 REVIEW QUESTIONS

Multiple Choice

1. B
The velocity vector is always tangent to the path and in the direction of motion, and the
centripetal force and acceleration are toward the center of the circle.

a
v

2. D
At the instant the ball is in the position shown in answer (E) in the previous question, it is
moving tangent to the path, down and to the right.

3. C v
6s
Period T = = 2s
3 rev
4. B
2πr 2π (4 m )
v= = ≈ 12 m / s
T 2s

5. D
mv 2 (40 kg )(12 m / s )
2
F= = = 1440 N
r 4m
6. B
The frictional force provides the centripetal force that causes the car to move in a circle.

7. D
mv 2 mv 2 v2
f = µFN = , and FN = mg , so µmg = , and µ =
R R gR
8. A
From section 5.6 in the textbook, v = gr = 9r = 3 r

9. C
Kepler’s 3rd law (section 5.5 in the textbook) states that the square of the orbital period is
proportional to the cube of the orbital radius. Thus, the farther away a satellite orbits, the
longer orbital period it has.

79
Chapter 5 Dynamics of Uniform Circular Motion

Free Response Question Solution

(a) 2 points

FT

mg

(b) 2 points
mv 2
FT = 2mg =
R
Solving for v we get
( )
v = 2 gR = 2 10 m / s 2 (0.75 m ) = 3.9 m / s

(c) 4 points
Position I Position III

i. ii.
FT

FT
mg
mg
(The lengths of the vectors are not
necessarily drawn to scale.)

(d) 3 points
Choosing up as the positive direction:

ΣF = ma
FT − mg = ma

The net force and the acceleration are directed toward the center of the circle.

 v2 
FT − mg = m 
R
 v2 
(
FT = mg + m  = (0.25 kg ) 10 m / s +
2 (0.25 kg )(5 m / s )
)
2
= 10.8 N
R 0.75 m

80
Chapter 5 Dynamics of Uniform Circular Motion

(e) 2 points
i. At the instant the ball is at position IV, its velocity vector points upward, and therefore
when the string breaks, the ball will rise upward against gravity. When we describe the
motion of an object, we discuss its velocity (speed and direction), its acceleration, and
any path it would follow. In this case, the ball would simply move straight upward,
accelerating at – 10 m/s2 and decreasing its speed, reach a maximum height where its
velocity is instantaneously zero, then accelerate downward and increase its speed as it
falls to the ground.

ii. 2 points
The ball is projected straight upward a 3.9 m/s, and at maximum height h, the speed of
the ball is zero.

v 2 = vo + 2 gh = 0
2

h=
vo
2

=
(3.9 m / s )2 = 0.76 m
(
2 g 2 10 m / s 2 )

81
Chapter 6 Work and Energy

Chapter 6

WORK AND ENERGY

PREVIEW

Work is the scalar product of the force acting on an object and the displacement through
which it acts. When work is done on or by a system, the energy of that system is always
changed. If work is done slowly, we say that the power level is low. If work is done
quickly, the power level is high. Kinetic energy is the energy an object has because of its
motion, and potential energy is the energy an object has because of its location or
configuration. If the energy of a system remains constant throughout a process, we say
that energy is conserved.

The content contained in all sections of chapter 6 of the textbook is included on the AP
Physics B exam.

QUICK REFERENCE

Important Terms
conservative force
a force which does work on an object which is independent of the path taken by
the object between its starting point and its ending point
conserved properties
any properties which remain constant during a process
energy
the non-material quantity which is the ability to do work on a system
joule
the unit for energy equal to one Newton-meter
kinetic energy
the energy a mass has by virtue of its motion
law of conservation of energy
the total energy of a system remains constant during a process
mechanical energy
the sum of the potential and kinetic energies in a system
potential energy
the energy an object has because of its position
power
the rate at which work is done or energy is dissipated
watt
the SI unit for power equal to one joule of energy per second
work
the scalar product of force and displacement

82
Chapter 6 Work and Energy

Equations and Symbols


W = F • s = (F cos θ )s where
1 W = work
KE = mv 2
2 F = force
1 1 s = displacement
W = KE f − KE 0 = mv f − mv0
2 2
F • s = scalar product of force and
2 2
displacement
PE = mgh
KE = kinetic energy
W g = ∆PE g = mg (h0 − h f ) v = velocity or speed
PE 0 + KE 0 = PE f + KE f m = mass
PE = potential energy (denoted as U on
W
P= the AP Physics exam)
t g = acceleration due to gravity
P = F • v = (F cos θ )v h = height above some reference point
P = power
t = time

Ten Homework Problems


Chapter 6 Problems 8, 10, 16, 20, 61, 64, 67, 73, 77, 78

DISCUSSION OF SELECTED SECTIONS

6.1 Work Done by a Constant Force


As you know by now, there are many words in physics that have meanings which are
well-defined but not necessarily used in the way they are normally used outside of the
context of a physics course, and work is certainly one of these words. In physics, work is
defined as the scalar product of force and displacement, that is,
Fsinθ
W = F·s = Fscosθ F

θ
Fcosθ

where θ is the angle between the applied force and the displacement. Here, the force and
displacement vectors are multiplied together in such a way that the product yields a
scalar. Thus, work is not a vector, and has no direction associated with it. Since work is
the product of force and displacement, it has units of newton-meters, or joules (J). A joule
is the work done by applying a force of one newton through a displacement of one meter.
One joule is about the amount of work you do in lifting your calculator to a height of one

83
Chapter 6 Work and Energy

meter. For work to be done on an object, the force must have a component in the same
direction as the displacement.

6.2 The Work – Energy Theorem and Kinetic Energy


Energy is the ability to do work, and when work is done, there is always a transfer of
energy. Energy can take on many forms, such as potential energy, kinetic energy, and
heat energy. The unit for energy is the same as the unit for work, the joule. This is
because the amount of work done on a system is exactly equal to the change in energy of
the system. This is called the work-energy theorem.

Kinetic energy is the energy an object has because it is moving. In order for a mass to
gain kinetic energy, work must be done on the mass to push it up to a certain speed, or to
slow it down. The work-energy theorem states that the change in kinetic energy of an
object is exactly equal to the work done on it, assuming there is no change in the object’s
potential energy.

1 1
W = ∆KE = mv f − mvo
2 2

2 2

The work done on a system can also be converted into heat energy, and usually some of
the work is.

6.3 Gravitational Potential Energy


Potential energy is the energy a system has because of its position or configuration.
When you stretch a rubber band, you store energy in the rubber band as elastic potential
energy. When you lift a mass upward against gravity, you do work on the mass and
therefore change its energy. The work you do on the mass gives it potential energy
relative to the ground. To lift it, you must apply a force equal to the weight mg of the
mass through a displacement height h, and thus the work done in lifting the mass is

W = Fs = (mg)h

which must also equal its potential energy

PE = mgh

84
Chapter 6 Work and Energy

6.4 Conservative Forces versus Nonconservative Forces, and the Work –


Energy Theorem
A force is said to be conservative if the work done by the force does not depend on the
path taken between any two points. The gravitational force and the spring force are two
examples of conservative forces. These two forces conserve energy during a round trip. A
force is said to be nonconservative if the work done by the force depends on the path
taken. Friction is the most common example of a nonconservative force on the AP
Physics B exam, since a taking longer path will dissipate more heat energy. Work done
by a nonconservative force generally cannot be recovered as usable energy.

6.5 The Conservation of Mechanical Energy


When work is done on a system, the energy of that system changes from one form to
another, but the total amount of energy remains the same. We say that total energy is
conserved, that is, remains constant during any process. This is also called the law of
conservation of energy.

We can use the fact that the sum of the potential and kinetic energies remains constant
during free fall to solve for quantities such as speed or initial height.

U top + K top = U bottom + K bottom


mgh top + ½ mv top 2 = mgh bottom + ½ mv bottom 2

The sum of the kinetic and potential energies of a system is called the total mechanical
energy of the system. These same principles can be applied to a block sliding down a
frictionless ramp, a pendulum swinging from a height, and many other situations. We
could use Newton’s laws and kinematics to solve these types of problems, but usually
conservation of energy is easier to apply.

h
2R

x
B D E

Example 1
A small block of mass m begins from rest at the top of a curved track at a height h and
travels around a circular loop of radius R. There is negligible friction between the block
and the track between points A and D, but the coefficient of kinetic friction on the
horizontal surface between points D and E is μ. The distance between points D and E is x.

85
Chapter 6 Work and Energy

Answer all of the following questions in terms of the given quantities and fundamental
constants.

(a) Determine the speed of the block at point B, at the bottom of the loop.
(b) Determine the kinetic energy of the block at point C, at the top of the loop.

After the block slides down the loop from point C to point D, it enters the rough portion
of the track. The speed of the block at point E is half the speed of the block at point D.

(c) Determine the speed of the block at point D, just before it enters the rough portion of
the track.
(d) Determine the amount of work done by friction between points D and E.
(e) Find an expression for the coefficient of kinetic friction μ.

Solution
Since there is no friction on the track between points A and B, there is no loss of energy.
Thus,

U A = KB
1
mgh =
2
mv B
2
Solving for v B ,

v B = 2 gh

(b) Conservation of energy:

U A = UC + KC
mgh = mg (2 R ) + K C
K C = mgh − 2mgR

(c) There is no energy lost on the track between points A and D, so the speed of the block
at point D is the same as the speed at point B:
v D = 2 gh

(d) The work done by friction is the product of the frictional force and the displacement
through which it acts.
W f = fx = µFN x = µmgx

(e) The frictional force causes the block to have a negative acceleration according to
Newton’s second law:
f − µmg
a= = = − µg
m m

86
Chapter 6 Work and Energy

Using a kinematic equation,

v E = v D + 2ax
2 2

0= ( 2 gh )
2
+ 2(− µg )x
2 gh h
µ= =
2 gx x

6.7 Power
Work can be done slowly or quickly, but the time taken to perform the work doesn’t
affect the amount of work which is done, since there is no element of time in the
definition for work. However, if you do the work quickly, you are operating at a higher
power level than if you do the work slowly. Power is defined as the rate at which work is
done. Oftentimes we think of electricity when we think of power, but it can be applied to
mechanical work and energy as easily as it is applied to electrical energy. The equation
for power is

Work
P=
time

and has units of joules/second or watts (W). A machine is producing one watt of power if
it is doing one joule of work every second. A 75-watt light bulb uses 75 joules of energy
each second.

87
Chapter 6 Work and Energy

Example 2

Motor

h
v

A motor raises a mass of 3 kg to a height h at a constant speed of 0.05 m/s. The battery
(not shown) which provides energy to the motor originally stores 4 J of energy, all of
which can be used to lift the mass.

(a) What is the power developed in the motor?


(b) To what maximum height can the motor lift the mass using its stored energy?

Solution
W Fh (mg )h
(a) P = = = = (mg )v = (3kg )(10 m / s 2 )(0.05 m / s ) = 1.5 Watts
t t t
(b)
W = mgh
W 4J
h= = = 0.13 m
(
mg (3kg ) 10 m / s 2 )

88
Chapter 6 Work and Energy

6.9 Work Done by a Variable Force


Graphically, the work done on an object or system is equal to the area under a Force vs.
displacement graph:

The area under the graph from zero to 20 meters is 1450 N m. Thus, the force represented
by the graph does 1450 J of work. This work is also a measure of the energy which was
transferred while the force was being applied.

Supplemental Section: Potential Energy vs. Displacement Graphs

Since work done by a force is equal to the product of the force and the displacement
through which it acts, the area under a force versus displacement graph is equal to the
work done. Doing work on a system changes the energy of that system. Consider the
potential energy versus displacement graph below:

89
Chapter 6 Work and Energy

This graph could represent the potential energy of a mass oscillating on a spring, as the
work done on the spring is equal to the potential energy at any point on the graph. That
potential energy in turn is converted into kinetic energy as the mass is accelerated by the
spring. The total energy of the system remains constant.

Example 3 Let’s say the total energy of the object represented by the U vs. x graph above
is 20 J. This means that the sum of the potential and kinetic energy of the object at any
displacement is 20 J, even though the potential energy on the graph goes higher than 20 J.

(a) What is the relationship between the potential energy and the displacement of this
object?
(b) What is the potential energy of the object when it is at a displacement of – 2 m?
(c) What is the maximum displacement of the object?
(d) What is the kinetic energy of the object when it is at a displacement of 3 m?
(e) What is the speed of the object when it is at a displacement of 3 m?

Solution
(a) Since the U vs. x graph is a parabola, the potential energy must be directly
proportional to the square of the displacement, that is, U ∝ x 2 .
(b) If we go over to x = - 2, and go up to meet the graph, the potential energy is 4 J.
(c) The maximum displacement of the object corresponds to the maximum potential
energy, which is 20 J. If we look up 20 J on the graph, we find that the corresponding
displacement is approximately ± 4.4 m.
(d) The potential energy that corresponds to x = 3 m is 9 J. Since the total energy is 20 J,
the kinetic energy at this displacement must be 20 J – 9 J = 11 J.

90
Chapter 6 Work and Energy

(e)
1 2
K= mv
2
2K 2(11 J )
v= = = 2.7 m / s
m 3 kg

CHAPTER 6 REVIEW QUESTIONS


For each of the multiple choice questions below, choose the best answer.
Unless otherwise noted, use g = 10 m/s2 and neglect air resistance.

1. Which of the following is NOT true of 4. A ball falls from a height h from a
work? tower. Which of the following
(A) It is the scalar product of force and statements is true?
displacement. (A) The potential energy of the ball is
(B) It is measured in joules. conserved as it falls.
(C) It has the same units as energy. (B) The kinetic energy of the ball is
(D) It is a vector which is always in the conserved as it falls.
same direction as the displacement. (C) The difference between the potential
(E) It takes energy to perform work energy and kinetic energy is a
constant as the ball falls.
2. A 4-kg box is pushed across a level (D) The sum of the kinetic and potential
floor with a force of 60 N for a energies of the ball is a constant.
displacement of 20 m, then lifted to a (E) The velocity of the ball is constant
height of 2 m. What is the total work as the ball falls.
done on the box?
(A) 200 J 5. A 0.5-kg ball is dropped from a third
(B) 400 J story window which is 20 m above the
(C) 1120 J sidewalk. What is the speed of the ball
(D) 1200 J just before it strikes the sidewalk?
(E) 1280 J (A) 5 m/s
(B) 10 m/s
3. A 20-kg cart is pushed up the inclined (C) 14 m/s
plane shown by a force F to a height of (D) 20 m/s
3 m. What is the potential energy of the (E) 200 m/s
cart when it reaches the top of the
inclined plane?
(A) 1500 J F
(B) 630 J
(C) 600 J 3m
(D) 300 J
(E) 150 J

91
Chapter 6 Work and Energy

6. Drew lifts a 60-kg crate onto a truck (E) Drew and Connor do the same
bed 1 meter high in 3 seconds. Connor amount of work and operate at the
lifts sixty 1-kg boxes onto the same same power level.
truck in a time of 2 minutes. Which of
the following statements is true?
(A) Drew does more work than Connor 7. A crane can lift a 500 kg mass to
does. height of 30 m in 2 minutes. The power
(B) Connor does more work than Drew at which the crane is operating is
does. (A) 125 Watts
(C) They do the same amount of work, (B) 1250 Watts
but Drew operates at a higher power (C) 7500 Watts
level. (D) 250 000 Watts
(D) They do the same amount of work, (E) 500 000 Watts
but Connor operates at a higher
power level.

92
Chapter 6 Work and Energy

Free Response Question

Directions: Show all work in working the following question. The question is worth 15
points, and the suggested time for answering the question is about 15 minutes. The parts
within a question may not have equal weight.

1. (15 points)

The potential energy U of 2.0 kg object as a function of distance x from its equilibrium
position is shown above. The object is acted upon by a force F. This particular object has
a total energy E of 30 J.
(a) What is the magnitude of the force F acting on the object between the displacements
x 1 = - 2 m and x 2 = - 6 m?
(b) What is the magnitude of the acceleration of this object between the displacements
x 1 = - 2 m and x 2 = - 6 m?
(c) Can the object reach a displacement of x = - 8 m? Explain.
(d) The object first reaches its maximum kinetic energy at which displacement?
(e) Determine the change in the speed of the object between x = 3 m and x = 5 m.

93
Chapter 6 Work and Energy

ANSWERS AND EXPLANATIONS TO CHAPTER 6 REVIEW QUESTIONS

Multiple Choice

1. D
Work is not a vector quantity.

2. E
Wtotal = Fs + mgh = (60 N )(20 m ) + (4 kg )(10 m / s 2 )(2 m ) = 1280 J

3. C
The potential energy is equal to the work done against gravity:
( )
W = mgh = (20 kg ) 10 m / s 2 (3 m ) = 600 J

4. D
As the ball falls, the potential energy decreases and the kinetic energy increases, but the
sum of the two remains constant.

5. D
Conservation of energy:
U top = K bottom
1 2
mgh = mv
2
( )
v = 2 gh = 2 10 m / s 2 (20 m ) = 20 m / s
6. C
( )
Both do W = mgh = (60 kg ) 10 m / s 2 (1 m ) = 600 J of work, but Drew does it in less time,
and therefore operates at a higher power level.

7. B

P= = = =
(
W Fh (mg )h (500 kg ) 10 m / s 2 (30 m ))= 1250 Watts
t t t 120 s

Free Response Question Solution

(a) 4 points
The magnitude (absolute value) of the force acting on the object is equal to the slope of
the U vs x graph:
30 J − 10 J
F= = 5N
− 2 m − (− 6 m )

(b) 2 points
F 5N
a= = = 2.5 m / s 2
m 2 kg

94
Chapter 6 Work and Energy

(c) 2 points
The object cannot reach – 8m, since this displacement corresponds to a potential energy
of 40 J, which is greater than the total energy of the object, 30 J.

(d) 2 points
Since the total energy is constant, the maximum kinetic energy occurs when the
minimum potential energy first occurs at x = 0.

(e) 5 points
The change in speed between x = 3 m and x = 5 m can be found from the change in
kinetic energy between these two points, which is equal to the change in potential energy
between the same two points. At x = 3 m the kinetic energy is 28 J and at x = 5 m the
kinetic energy is 10 J. Thus,

2K 3 2(28 J )
v3 = = = 7.5 m / s
m 2kg
2K 5 2(10 J )
v5 = = = 3.2 m / s
m 2kg
∆v = 7.5 m / s − 3.2 m / s = 4.3 m / s

95
Chapter 7 Momentum and Impulse

Chapter 7

IMPULSE AND MOMENTUM

PREVIEW

The momentum of an object is the product of its mass and velocity. If you want to change
the momentum of an object, you must apply an impulse, which is the product of force and
the time during which the force acts. If there are no external forces acting on a system of
objects, the momentum is said to be conserved, that is, the total momentum of the system
before some event (like a collision) is equal to the total momentum after that event. In
this chapter, we will discuss examples of both one- and two-dimensional collisions.

The content contained in all sections of chapter 7 of the textbook is included on the AP
Physics B exam.

QUICK REFERENCE

Important Terms

impulse
The product of the average force acting on an object and the time during which it
acts. Impulse is a vector quantity, and can also be calculated by finding the area
under a force versus time curve.
linear momentum
The product of the mass of an object and its velocity. Momentum is a vector
quantity, and thus the total linear momentum of a system of objects is the vector
sum of the individual momenta of the objects in the system.
internal forces
The forces which act between the objects of a system
external forces
The forces which act on the objects of a system from outside the system, that is by
an agent which is not a part of the system of objects which are being studied.
inelastic collision
A collision between two or more objects in which momentum is conserved but
kinetic energy is not conserved, such as two railroad cars which collide and lock
together.
elastic collision
A collision between two or more objects in which both momentum and kinetic
energy are conserved, such as in the collision between two steel balls.
center of mass
The point at which the total mass of a system of masses can be considered to be
concentrated.

96
Chapter 7 Momentum and Impulse

Equations and Symbols

p = mv where

J = F∆t = ∆p = mv f − mv 0 p = momentum
m = mass
m1 x1 + m2 x 2
xcm = v = velocity
m1 + m2 J = impulse
F = force
∆t = time interval during which a force
acts
x cm = position of the center of mass of a
system of particles
x 1 = position of a mass relative to a
chosen origin

Ten Homework Problems


Chapter 7 Problems 5, 9, 17, 22, 28, 29, 32, 39, 41, 55

DISCUSSION OF SELECTED SECTIONS

7.1 The Impulse – Momentum Theorem


The momentum p of an object is the product of the mass m of the object and its velocity
v:

p = mv

The momentum of a moving mass is a vector which has a direction that is the same as
the velocity of the mass. Thus, the momentum of an object can be broken down into its
components:
py
p
p = px + py
px

where p x = pcosθ and p y = psinθ .

The magnitude of the momentum vector can be found by the Pythagorean theorem:

p = px + p y
2 2

97
Chapter 7 Momentum and Impulse

Newton’s second law states that an unbalanced (net) force acting on a mass will
accelerate the mass in the direction of the force. Another way of saying this is that a net
force acting on a mass will cause the mass to change its momentum. We can rearrange
the equation for Newton’s second law to emphasize the change in momentum:

 ∆v 
Fnet = ma = m 
 ∆t 

Rearranging this equation by dividing both sides by ∆t gives

F∆t = m∆v = mv f − mv 0

The left side of the equation (F∆t) is called the impulse, and the right side is the change
in momentum. This equation reflects the impulse-momentum theorem, and in words can
be stated “a force acting on a mass during a time causes the mass to change its
momentum”. The force F in this equation is the average force acting over the time
interval.
Example 1
A 2-kg block slides along a floor of negligible friction with a speed of 20 m/s when it
collides with a 3-kg block, which is initially at rest. The graph below represents the force
exerted on the 3-kg block by the 2-kg block as a function of time.

Find
(a) the initial momentum of the 2-kg block,
(b) the impulse exerted on the 3-kg block,and
(c) the momentum of the 2-kg block immediately after the collision.

98
Chapter 7 Momentum and Impulse

Solution: (a) The initial momentum of the 2-kg block is


 m
p = mv = (2kg ) 20  = 40
kgm
to the right.
 s s

(b) When the two blocks collide, they exert equal and opposite forces on each other for
the same time interval, and thus exert equal and opposite impulses (F∆t) on each other.
The impulse exerted on the 3-kg block can be found by finding the area under the force
vs. time graph:

Impulse = area under F vs. t graph = 20 N s.

(c) Since the 2-kg block is experiences – 20 N s of impulse, which acts in the opposite
direction of the motion of the block, the impulse causes the block to lose momentum.
This loss (change) in momentum is equal to the impulse exerted on the block. Thus, by
the impulse-momentum theorem,

Impulse = change in momentum


F∆t = mv f − mvi

mv f = mvi + (F∆t ) = 40 + (− 20 Ns ) = 20
kgm kgm
s s
kg m
Note that a N s is equivalent to .
s
Example 2
One of the photographs in chapter 7 of your textbook shows a tennis ball striking a tennis
racquet, applying an impulse to it. Suppose the 0.1 kg ball strikes the racquet with a
velocity of 60 m/s at an angle of 30° from a line which is perpendicular to the face of the
racquet and rebounds with a speed of 60 m/s at 30° above the perpendicular line, as
shown below. The ball is in contact with the strings of the racquet for 12 milliseconds.
y

+x

30˚

Find
(a) the magnitude and direction of the average impulse exerted on the ball by the strings
of the racquet, and
(b) the magnitude of the average acceleration of the ball while it is in contact with the
strings.

99
Chapter 7 Momentum and Impulse

Solution:
(a) The strings of the racquet exert a force and thus an impulse which is perpendicular to
the face of the racquet, that is, along the +x – axis in the figure above. Therefore the
change in momentum of the ball is also only along the +x – axis:
F∆t = ∆p x = m∆v x = m( v fx − v 0 x )
where v0 x = −v0 cos 30 and v0 x = +v f cos 30
So the impulse is
m m
(0.1 kg )[60 cos 30 − (−60 cos 30)] = 10.4 N s
s s
(b) Since the force is applied in the +x – direction, the average acceleration is must also
be directed along the +x – axis, that is, there is no acceleration along the y-axis.
m m
[60 cos 30 − (−60 cos 30)]
∆v s s m
ax = = = 8700 2
∆t 0.012 s s

7.2 The Principle of Conservation of Linear Momentum


We’ve seen that if you want to change the momentum of an object or a system of
objects, Newton’s second law says that you have to apply an unbalanced force. This
implies that if there are no unbalanced forces acting on a system, the total momentum of
the system must remain constant. This is another way of stating Newton’s first law, the
law of inertia, discussed in chapter 4. If the total momentum of a system remains constant
during a process, such as an explosion or collision, we say that the momentum is
conserved. The principle of conservation of linear momentum states that the total
linear momentum of an isolated system remains constant (is conserved). An isolated
system is one for which the vector sum of the external forces acting on the system is zero.

Typically, the AP Physics B exam includes the following types of problems which use
the principle of conservation of linear momentum: recoil in one and two dimensions,
inelastic collisions in one and two dimensions, and elastic collisions in one and two
dimensions. Remember, if a momentum vector is conserved, its components are also
conserved.

Example 3
A toy cannon (m 1 = 2 kg) is mounted to two horizontal rails on which it can slide with
negligible friction. The cannon fires a ball (m 2 = 0.025 kg) at a speed v f2 = 40 m/s at an
angle of 60° above the horizontal.
(a) Is the total momentum of the system conserved during the firing of the cannon?
Explain.
(b) What is the recoil velocity of the cannon after it is fired?

100
Chapter 7 Momentum and Impulse

Solution: Let’s sketch the cannon and ball before and after they are fired:
Before After

pf2y

p2f2

pf2x
p1x
60˚ 60˚

(a) The total momentum is not conserved in this case, since the horizontal rail provides an
external force acting on the cannon in the vertical direction. Since there is negligible
friction acting horizontally on the cannon, we can say that momentum is conserved in the
horizontal direction, but not in the vertical direction.

(b) Since momentum is conserved in the horizontal direction, we can set the momentum
of the system (cannon and ball) before the collision, which is zero, equal to the
momentum of the system after the collision.
p 0 = p f 1x + p f 2 x
0 = m1 v f 1x + m2 v f 2 x
0 = m1v f 1x + m2 v f 2 cos 60
Solving for the horizontal recoil velocity of the cannon, we get
m
− m2 v f 2 cos 60 − (0.025 kg )(40 s ) cos 60 m
v f 1x = = = − 0.25
m1 2 kg s

7.3 Collisions in One Dimension


Example 4
A lump of clay (m 1 = 0.2 kg) moving horizontally with a speed v o1 = 16 m/s strikes and
sticks to a wood block (m 2 = 3 kg) which is initially at rest on the edge of a horizontal
table of height h = 1.5 m. Neglecting friction, find
(a) the horizontal distance x from the edge of the table at which the clay and block strike
the floor, and
(b) the total momentum of the clay and block just before they strike the floor.

101
Chapter 7 Momentum and Impulse

Solution: The sketches below represent the system before, during, and after the collision.
Before During After

(a) Before we can find the horizontal distance the clay and block travel we need to find
their speed v f as they leave the edge of the table. Momentum is conserved in this inelastic
collision:
po = p f
m1 v o1 = (m1 + m2 ) v f
m
(0.2 kg )(16 )
m1vo1 s =1m
vf = =
(m1 + m2 ) (0.2 kg + 3.0 kg ) s
Now the clay and block have become a projectile which is launched horizontally. We can
find the time of flight by using the height:
1
h = gt 2
2
2h 2(1.5 m)
t= = = 0.55 s
g m
10 2
s
m
Then the horizontal distance traveled is x = v f t = (1 )(0.55 s ) = 0.55 m
s
(b) The momentum of the clay and block just before it strikes the ground can be found by
finding the horizontal and vertical components of the momentum:
m kg m
p x = mv x = (3.2 kg )(1 ) = 3.2
s s
m kg m
p y = mv y = m( gt ) = (3.2 kg )(10 2 )(0.55 s ) = 17.6
s s
By the Pythagorean theorem, the magnitude of the momentum can be found by
kg m 2 kg m 2 kg m
p= p x + p y = (3.2 ) + (17.6 ) = 17.9
2 2

s s s
The angle at which the momentum is directed is
 py  −1 17.6 
θ = tan −1   = tan  = 80° below the horizontal.
 px   3.2 

102
Chapter 7 Momentum and Impulse

Example 5
A white pool ball (m 1 = 0.3 kg) moving at a speed of v o1 = +3 m/s collides head-on with
a red pool ball (m 2 = 0.4 kg) initially moving at a speed of v o2 = - 2 m/s. Neglecting
friction and assuming the collision is perfectly elastic, what is the velocity of each ball
after the collision?

Solution: The sketches below represent the system before and after the collision.
Before After

vo1 vo2
vf1 vf2

white red
white red

Since momentum is conserved in this collision, we can set the total momentum of the
system before the collision equal to the total momentum after the collision:

p 01 + p 02 = p f 1 + p f 2
m1v 01 + m2 v 02 = m1v f 1 + m2 v f 2

Solving for v f1 we get

m1v01 + m2 v02 − m2 v f 2
vf1 =
m1

But here we have two unknowns, v f1 and v f2 , and only one equation. We can generate
another equation containing the same variables if we remember that both momentum and
kinetic energy are conserved in an elastic collision.
KE 01 + KE 02 = KE f 1 + KE f 2
1 1 1 1
m1v01 + m2 v02 = m1v f 1 + m2 v f 2
2 2 2 2

2 2 2 2
Substituting the equation solved above for v f1 into the equation for conservation of
kinetic energy, we get one equation with only one unknown, namely v f2 . Substituting the
known values into this equation and solving for v f2 gives v f2 = +2.3 m/s. Solving for v f1
in the equation above gives v f1 = - 2.7 m/s.

It can be shown that if we solve the equations for conservation of momentum and
conservation of kinetic energy simultaneously, it always turns out that the relative speeds
of the two masses remains the same (except for a negative sign) before and after a

103
Chapter 7 Momentum and Impulse

perfectly elastic collision regardless of the masses of the two objects. That is,
v 01 − v 02 = −( v f 1 − v f 2 )

7.4 Collisions in Two Dimensions


Example 6
The diagram below shows a collision between a white pool ball (m 1 = 0.3 kg) moving at
a speed v 01 = 5 m/s in the +x direction and a blue pool ball (m 2 = 0.6 kg) which is initially
at rest. The collision is not head-on, so the balls bounce off of each other at the angles
shown. Find the final speed of each ball after the collision.

Before After

+y +y vf2

v01

65˚
+x +x
blue 25˚
white
vf1

Solution: The components of the momenta before and after the collision are conserved.

Writing the x – components of the momentum before and after the collision:

p 01x + p 02 x = p f 1x + p f 2 x
m1 v 01x + m2 v 02 x = m1 v f 1x + m2 v f 2 x
m
(0.3 kg )(5 ) = (0.3 kg )(v f 1 cos 25 + (0.6 kg )(v f 2 cos 65)
s
Here we have two unknowns, v f1 and v f2 , and only one equation so far. When we write
the conservation of momentum equation for the y-components of the momentum of each
ball, we see that the total momentum in the y direction is zero before the collision, and
thus must be zero after the collision.

p 01 y + p 02 y = p f 1 y + p f 2 y
0 = m1 v f 1 y + m2 v f 2 y
0 = (0.3 kg )(v f 1 sin 25) + (0.6 kg )(v f 2 sin 65)

104
Chapter 7 Momentum and Impulse

Solving these two equations simultaneously for the unknown speeds gives v f1 = 1.8 m/s
and v f2 = 4 m/s. In any two-dimensional elastic collision in which one mass is at rest, the
angle between the two objects after the collision will be 90°.

7.5 Center of Mass


The center of mass of a system of particles can be thought of as the point at which all of
the mass of the system can be considered to be concentrated. The equation given to you
in your textbook used for calculating the center of mass of two objects on the x – axis is

m1 x1 + m2 x 2
xcm =
(m1 + m2 )

This equation is typically not needed on the AP Physics B exam, but the concept of
center of mass is important in many applications on the AP exam. For a conservation law
to be true, it must hold true in any inertial reference frame, including the reference frame
of the center of mass. If we multiply both sides of the center of mass equation above by
the total mass (m 1 + m 2 ), we get

(m1 + m2 ) xcm = m1 x1 + m2 x 2

If the two objects are moving, each x is changing with respect to time, we can replace
∆x
each x with ∆x, and divide both sides by ∆t. The ratio is velocity, so our center of
∆t
mass equation becomes

(m1 + m2 )vcm = m1v1 + m2 v 2

In words, this equation says that the momentum of the center of mass is equal to the sum
of the momenta of each of the particles, which is conservation of momentum. If we take
the rate of change of this equation, we get

(m1 + m2 )a cm = m1 a1 + m2 a 2

which is an expression of Newton’s second law, that is, the net force acting on the center
of mass of a system of particles is equal to the sum of the forces acting on the individual
particles. Here we have written the scalar form of these equations, since we are assuming
all velocities, accelerations, and forces to be directed along the x – axis. As your textbook
shows, often we can solve problems from the reference frame of the center of mass.

Example 7
Two masses m and 2m are connected by a spring and rest on a horizontal table on which
friction can be neglected. The two masses are pulled apart to a length L and released from
rest. Describe
(a) the location of the center of mass before the masses are released, and
(b) the motion of the center of mass after the masses are released from rest.

105
Chapter 7 Momentum and Impulse

Solution:
(a) The center of mass will be located closer to the greater mass. In fact, the center of
1 2
mass will be located at a distance of L from 2m and L from m.
3 3

(b) Before the masses are released, the center of mass is at rest. After the masses are
released, there are no external forces acting on the center of mass, and therefore the
momentum of the center of mass does not change, remaining at rest while the two masses
oscillate about it. It also follows that the momentum of block m is equal and opposite to
block 2m at all times during the oscillation of the masses.

CHAPTER 7 REVIEW QUESTIONS


For each multiple choice question below, choose the BEST answer.

1. A 0.2-kg hockey puck is sliding along Questions 2 – 3: A net force is applied to


the ice with an initial speed of 12 m/s a block of mass 4 kg according to the
when a player strikes it with his stick, Force vs. time graph below.
causing it to reverse its direction and
giving it a speed of 23 m/s. The impulse
the stick applies to the puck is most
nearly
(A) - 2 N s
(B) - 6 N s
(C) - 7 N s
(D) - 70 N s
(E) - 120 N s

2. The impulse given to the mass


between 1 and 5 seconds is most nearly
(A) 20 N s
(B) 16 N s
(C) 12 N s
(D) 10 N s
(E) 4 N s

106
Chapter 7 Momentum and Impulse

5. A block of mass m slides with a speed


3. If the mass starts from rest at t = 1 s, v o on a frictionless surface and collides
the speed of the mass at t = 5 s is most with another mass M which is initially at
nearly rest. The two blocks stick together and
m v
(A) 20 move with a speed of o . In terms of
s 3
m m, mass M is most nearly
(B) 16
s m
(A)
m 4
(C) 12
s m
(B)
m 3
(D) 8
s m
(C)
m 2
(E) 4
s (D) 2m
(E) 3m

4. An astronaut floating at rest in space


throws a wrench in one direction and
subsequently recoils back with a velocity
in the opposite direction. Which of the
following statements is/are true? 6. The vector diagram above represents
I. The velocity of the wrench is equal the momenta of two objects after they
and opposite to the velocity of the collide. One of the objects is initially at
astronaut. rest. Which of the following vectors may
II. The momentum of the wrench is represent the initial momentum of the
equal and opposite to the momentum other object before the collision?
of the astronaut.
III. The impulse applied to the wrench is (A)
equal and opposite to the impulse
applied to the astronaut.
(A) I only (B)
(B) II only
(C) I and II only
(D) II and III only (C)
(E) I, II, and III

(D)

(E) zero

107
Chapter 7 Momentum and Impulse

9. The total momentum of the two balls


+y after the collision is most nearly
4 kg
kg m
(A) 1
s
+x
kg m
(B) 2
3 kg s
kg m
7. Two objects of mass 4 kg and 3 kg (C) 3
approach each other at a right angle as s
shown above. The 4-kg mass moves kg m
(D) 4
along the +x – axis with an initial speed s
of 5 m/s, and the 3-kg mass moves in the (E) zero
+y – direction with a speed of 5 m/s. The
two masses collide at the origin and stick 10. A bullet moving with an initial speed
together. Measured from the +x – axis, of v 0 strikes and embeds itself in a block
the angle of the resulting momentum of of wood which is suspended by a string,
the two objects after the collision is most causing the bullet and block to rise to a
nearly maximum height h. Which of the
(A) 30° above the +x – axis following statements is true of the
(B) 37° above the +x – axis collision?
(C) 45° above the +x – axis (A) The initial kinetic energy of the
(D) 53° above the +x – axis bullet before the collision is equal to
(E) 60° above the +x – axis the kinetic energy of the bullet and
block immediately after the collision.
+y (B) The initial kinetic energy of the
bullet before the collision is equal to
the potential energy of the bullet and
+x block when they reach the maximum
height h.
Questions 8 – 9: A 0.2-kg billiard ball (C) The initial momentum of the bullet
approaches an identical ball at rest with before the collision is equal to the
a speed of 10 m/s along the +x - axis, as momentum of the bullet and block at
shown above. The collision between the the instant they reach the maximum
balls is perfectly elastic, and after the height h.
collision the incident ball moves at an (D) The initial momentum of the bullet
angle of 50° below the x – axis. before the collision is equal to the
momentum of the bullet immediately
8. The angle at which the target ball after the collision.
moves after the collision above the +x – (E) The kinetic energy of the bullet and
axis is most nearly block immediately after the collision
is equal to the potential energy of the
(A) 10°
bullet and block at the instant they
(B) 40°
reach the maximum height h.
(C) 50°
(D) 90°
(E) 140°

108
Chapter 7 Momentum and Impulse

Free Response Question


Directions: Show all work in working the following question. The question is worth 15
points, and the suggested time for answering the question is about 15 minutes. The parts
within a question may not have equal weight.

vo

1. (15 points)
A block of mass m is moving on a horizontal frictionless surface with a speed v o as it
approaches a block of mass 2m which is at rest and has an ideal spring attached to one
side. When the two blocks collide, the spring is completely compressed and the two
blocks momentarily move at the same speed, and then separate again, each continuing to
move.
(a) Briefly explain why the two blocks have the same speed when the spring is
completely compressed.
(b) Determine the speed v f of the two blocks while the spring is completely compressed.
(c) Determine the kinetic energy of the two blocks as they move together with the same
speed.
(d) When the spring expands, the blocks are again separated, and the spring returns its
compressed potential energy to kinetic energy in the two blocks. On the axes below,
sketch a graph of kinetic energy vs. time from the time block m approaches block 2m
until the two blocks are separated after the collision.

KE

(e) Write the equations that could be used to solve for the speed of each block after they
have separated. It is not necessary to solve these equations for the two speeds.

109
Chapter 7 Momentum and Impulse

ANSWERS AND EXPLANATIONS TO CHAPTER 7 REVIEW QUESTIONS

Multiple Choice

1. A
F∆t = m(v f − v0 ) = (0.2 kg )(−23
m m
− 12 ) = −7 N s
s s
2. B
The impulse is equal to the area under the graph from 1 s to 5 s = 16 N s.

3. D
m
Impulse = 16 N s = (4 kg )(v f − 0) gives v f = 4 .
s
4. D
Conservation of momentum indicates that the two momenta are equal and opposite, and
since they both experience the same force during the same time interval, the impulses
must also be equal and opposite. Since the two masses are different, their velocities
would not be the same.

5. D
Conservation of momentum for the inelastic collision gives
v 
mv0 = (m + M ) 0  , implying that M = 2m.
3
6. B
Conservation of momentum states that the momentum vector before the collision must
equal the vector sum of the momenta after the collision. Adding the two vectors tip-to-tail
gives a resultant which points in the direction of the vector arrow in answer (B).

7. B
After the collision, there is one mass of 7 kg moving upward and to the right.
Measuring the angle θ from the +x – axis, the conservation of momentum equations in
the horizontal and vertical directions are
p 0 x = p fx
(4 kg )(5 kg ) = (7 kg )v f cosθ and
p 0 y = p fy
(3 kg )(5 kg ) = (7 kg )v f sin θ
Dividing the y-component equation by the x-component equation yields
15
= tan θ which implies that θ = 37° from the +x – axis.
20

8. B
In a perfectly elastic collision, the angle between the two momentum vectors after the
collision must add to 90°. So, 90° – 50° = 40° below the +x – axis.

110
Chapter 7 Momentum and Impulse

9. B
Conservation of momentum states that the total momentum of the system is equal to the
total momentum after the collision. Since the total momentum before the collision is
(0.2 kg)(10 m/s) = 2 kg m/s, the total momentum after the collision is also 2 kg m/s.

10. E
During the inelastic collision between the bullet and the block, kinetic energy is lost. But
the kinetic energy of the bullet and block immediately after the collision is transformed
into potential energy at their maximum height.

Free Response Question Solution

1. We begin by drawing the situation before, during, and after the collision:
Before During After

(a) 2 points
When the spring is completely compressed, the two blocks are at rest relative to each
other and must have the same speed. At this point, it is as if they are stuck together
immediately after an inelastic collision.

(b) 3 points
Conservation of momentum:
mv0 = (m + 2m)v f
v0
vf =
3
(c) 3 points
2
1 v  1
K= (3m) 0  = mv0
2

2 3 6

(d) 3 points
KE

The dip in the graph indicates the time during which the spring is compressed. After the
blocks separate, all of the kinetic energy is restored to the system.

111
Chapter 7 Momentum and Impulse

(e) 4 points
Since the kinetic energy is the same before the collision and after the blocks have
separated, it is as if the blocks have undergone and elastic collision, where both
momentum and kinetic energy are conserved. Thus, two equations that could be solved
for the speeds after the blocks separate are
1 1 1
mv0 = mv f 1 + mv f 2 and mv0 = mv f 1 + mv f 2
2 2 2

2 2 2

112
Chapter 9 Rotational Dynamics

Chapter 9

ROTATIONAL DYNAMICS

PREVIEW

A force acting at a perpendicular distance from a rotation point, such as pushing a


doorknob and causing the door to rotate on its hinges, produces a torque. If the sum of
the torques acting on an object is zero, we say that the object is in equilibrium. The
weight of an object can produce a torque, and we say that the point at which an object’s
weight can be considered to be concentrated is its center of gravity.

The content of sections 1, 2, and 3, and 7 (Example 9) in chapter 9 of the textbook is


included on the AP Physics B exam.

QUICK REFERENCE

Important Terms
center of gravity
the point on a rigid body at which its weight can be considered to act when
calculating the torque due to its weight
equilibrium
a rigid body is in equilibrium if the net force and net torque acting on the body are
both zero.
lever arm
the distance between the line of action of a force acting on a body and the axis of
rotation.
torque
the vector product of a force acting on a body and the lever arm at which it acts

Equations and Symbols

τ = F where
W1 x1 + W2 x 2
xcg = τ = torque
W1 + W2
F = force
 = lever arm which is perpendicular to
the applied force
x cg = position of the center of gravity of
a system of particles
W = weight
x = position of an object relative to some
chosen origin

114
Chapter 9 Rotational Dynamics

Ten Homework Problems


Chapter 9 Problems 2, 3, 4, 6, 7, 10, 14, 16, 20, 63

DISCUSSION OF SELECTED SECTIONS

9.1 – 9.3 Torque, Equilibrium, and Center of Gravity

Torque is the result of a force acting at a distance from a rotational axis, and may cause a
rotation about the axis. Torque is the vector product of the displacement vector r (as in
radius) from the rotational axis and the force F:
T = r×F
The unit for torque is the newton-meter. By the right-hand rule, place your fingers in the
direction of r, and cross them into the direction of the force F, and the torque vector T
points in the direction of your thumb. To open a hinged door, you apply a force to the
doorknob which is mounted a certain distance from the hinges, and create a torque which
causes the door to rotate. When you use a wrench to tighten a bolt, you apply a force at
the end of the wrench to get the most torque to turn the bolt.
If the force is applied to the wrench so that there is an angle θ between the displacement
vector r and the force vector F, then the magnitude of the torque becomes
Τ = rFsinθ

For a torque to be produced there must a component of the force which is perpendicular
to the radius vector. A force and a radius which are parallel to each other will not produce
a torque, since the sine of the angle between them is zero.

F
Fsinθ
θ

For a system in static equilibrium, the sum of the forces must equal zero, and the sum of
the torques must also equal zero. This is illustrated in the next example.

115
Chapter 9 Rotational Dynamics

Example 1
Two children sit on a see-saw which is 3 m long and pivoted on an axis at its center. The
first child has a mass m 1 of 25 kg and sits at the left end of the see-saw, while the second
child has a mass m 2 of 50 kg and sits somewhere on the see-saw to the right of the axis.
At what distance r 2 from the axis should the second child sit to keep the see-saw
horizontal?
3m

r2
m1 m2

For the see-saw to remain horizontal, the torque on the left must equal the torque on the
right. The forces acting on the see-saw on either side are just the weight mg of each child.
So,

Torque on the left = Torque on the right


r1 F1 = r2 F2
m1 gr1 = m2 gr2

Dividing out the g’s and solving for r 2 we get

m1 r1 (25 kg )(1.5m)
r2 = = = 0.75 m to the right of the axis.
m2 50 kg

Could you have guessed this answer before we worked it out? Since the child on the right
is twice as heavy as the child on the left, he should sit half as far from the axis on the
right side to balance the torque the lighter child is producing on the left side.

Example 2
A mechanic applies a force of 400 N to the end of a 15-cm wrench at an angle of 30˚
from the horizontal, as shown below. If the bolt does not turn, what is the reaction torque
the bolt must be providing?
F=400 N
30˚

0.15 m

116
Chapter 9 Rotational Dynamics

Solution
The reaction torque provided by the bolt is equal to the torque the mechanic is applying
to the bolt.
T = rF sin θ = (0.15 m )(400 N )sin 30 = 30 Nm

If the vector sum of the torques is not zero, the object will rotate with an angular
acceleration. The AP Physics B exam typically only deals with equilibrium, that is, the
sum of the torques is zero.

CHAPTER 9 REVIEW QUESTIONS


For each of the multiple choice questions below, choose the best answer.
Unless otherwise noted, use g = 10 m/s2 and neglect air resistance.

1. Torque
(A) is the vector product of displacement
and force. F3
(B) is a scalar and has no direction F4
associated with it.
(C) is always equal to force.
(D) is always greater for shorter lever F2
arms.
(E) must always equal zero.

A B C D E
F5
F1
3 4
3. The figure above shows a flat object
lying on a table of negligible friction.
2. Two blocks of mass 3 kg and 4 kg Five forces are separately applied to the
hung from the ends of a rod of object as shown. Which of the five
negligible mass which is marked in forces will NOT cause the object ot
seven equal parts as shown. At which rotate about the center of the object?
of the points indicated should a string (A) F 1
be attached if the rod is to remain (B) F 2
horizontal when suspended from the (C) F 3
string? (D) F 4
(A) A (E) F 5
(B) B
(C) C
(D) D
(E) E

117
Chapter 9 Rotational Dynamics

L ℓ

θ
m1 C m2
m
4. Two masses are mounted on either 5. The figure above shows a mass m
end of a bar of negligible mass. The bar hanging from the end of a massless rod
is marked off in quarters. The center of which is pivoted on a fulcrum as shown.
mass of the system C is labeled as A string is tied at an angle θ from the
shown. The ratio of m 2 to m 1 is horizontal at the right end of the rod to
4 keep the rod from rotating. Which of the
(A)
1 following is a correct equation for
3 finding the tension F T in the string?
(B)
1 (A) mgL = FT 
2 (B) mgL = FT  cos θ
(C)
1 (C) mgL = FT  sin θ
(D)
1 (D) mg = FT
2 (E) mg = FT sin θ
1
(E)
3

118
Chapter 9 Rotational Dynamics

Free Response Question

Directions: Show all work in working the following question. The question is worth 10 points,
and the suggested time for answering the question is about 10 minutes. The parts within a
question may not have equal weight.

1. (10 points)
In the laboratory, you are asked to determine the mass of a meter stick without using a scale of
any kind. In addition to the meter stick, you may use any or all of the following equipment:

____ a set of known masses ____ four weight hangers ____ tape

____ a fulcrum upon which the ____ string ____ stopwatch


meter stick can be mounted
and pivoted

(a) Briefly list the steps in your procedure that will lead you to the mass of the meter
stick. Include definitions of any parameters that you will measure.
(b) On the list of equipment before part (a) place check marks beside each additional
piece of equipment you will need to do this experiment.

(c) Show the calculations you would perform to find the mass of the meter stick.

119
Chapter 9 Rotational Dynamics

ANSWERS AND EXPLANATIONS TO CHAPTER 9 REVIEW QUESTIONS

Multiple Choice

1. A
Torque is defined as the vector (cross) product of displacement (lever arm) and force.

2. D
For the sum of the torques to be zero, m3 gr3 = m4 gr4 , or (3 kg )(4 units ) = (4 kg )(3 units ) .

3. B
The line of action of F 2 passes through the center of rotation, and thus cannot produce a torque
about the center (r = 0).

4. B
The center of mass is 3 times farther from the lighter mass than the heavier mass, so the heavier
mass m 2 must be 3 times larger than the lighter mass m 1 .

5. C
The free-body diagram for the system is shown below.
L ℓ

θ
FN Fsinθ FT
mg

The sum of the torques about the fulcrum (which applies a normal force to the rod) must equal
zero. Thus, the hanging weight acting at L must balance the downward component of the tension
acting at ℓ: mgL = FT  sin θ .

Free Response Question Solution

(a) 4 points
Mount the meter stick on the fulcrum so that it is pivoted on a point other than the center of the
stick. Hang two unequal weights on either side of the meter stick at such distances that the meter
stick remains horizontal. Record the value of each mass and the distance from the fulcrum to
each mass. Let m 1 be the mass on the left and ℓ 1 be the distance from the fulcrum to m 1 .
Likewise, let m 1 be the mass on the left and ℓ 1 be the distance from the fulcrum to m 1 . Let C be
the center of the meter stick. Repeat the procedure using different masses and distances.

120
Chapter 9 Rotational Dynamics

ℓ1 ℓ2

C
m1
m2

(b) 3 points
We can check the set of known masses, the fulcrum, and two of the weight hangers.

(c) 3 points
The free-body diagram for the stick would look like this:

ℓM

ℓ1 ℓ1

Mg m2g
m1g
Fulcrum

where M is the unknown mass of the meter stick, and ℓ M is the distance from the fulcrum to the
center of mass of the meter stick. For equilibrium, the sum of the torques is zero, that is, the
torque caused by the masses on the left of the fulcrum must equal the caused by the masses on
the right of the fulcrum:

m1 g 1 + Mg M = m2 g 2
m2 g 2 − m1 g 1
M =
g M
Substitute the measured values from part (a) to find a value for the mass of the meter stick.

121
Chapter 10 Simple Harmonic Motion

Chapter 10

SIMPLE HARMONIC MOTION

PREVIEW

An object such as a pendulum or a mass on a spring is oscillating or vibrating if it is


moving in a repeated path at regular time intervals. We call this type of motion harmonic
motion. For an object to continue oscillating there must be a restoring force continually
trying to restore it to its equilibrium position. For, the force exerted by an ideal spring
obeys Hooke’s law. As an object in simple harmonic motion oscillates, its energy is
repeatedly converted from potential energy to kinetic energy, and vice – versa.

The content contained in sections 1, 2, 3, and 4 of chapter 10 of the textbook is included


on the AP Physics B exam.

QUICK REFERENCE

Important Terms
amplitude
maximum displacement from equilibrium position; the distance from the midpoint
of a wave to its crest or trough.
equilibrium position
the position about which an object in harmonic motion oscillates; the center of
vibration
frequency
the number of vibrations per unit of time
Hooke’s law
law that states that the restoring force applied by a spring is proportional to the
displacement of the spring and opposite in direction
ideal spring
any spring that obeys Hooke’s law and does not dissipate energy within the
spring.
mechanical resonance
condition in which natural oscillation frequency equals frequency of a driving
force
period
the time for one complete cycle of oscillation
periodic motion
motion that repeats itself at regular intervals of time
restoring force
the force acting on an oscillating object which is proportional to the displacement
and always points toward the equilibrium position.

122
Chapter 10 Simple Harmonic Motion

simple harmonic motion


regular, repeated, friction-free motion in which the restoring force has the
mathematical form F = - kx.

Equations and Symbols

Fs = − kx where
1 2
PE elastic = kx F s = the restoring force of the spring
2 k = spring constant
x = A cos ωt x = displacement from equilibrium
2π position
ω= = 2πf PE elastic = elastic (spring) potential
T
energy (denoted U s on the AP
m
TS = 2π exam)
k A = amplitude
l ω = angular frequency
TP = 2π T = period
g
f = frequency
1 m = mass
T=
f T P = period of a pendulum
T S = period of a mass on a spring
g = acceleration due to gravity

Ten Homework Problems


Chapter 10 Problems 4, 6, 8, 12, 17, 27, 28, 31, 40, 41

DISCUSSION OF SELECTED SECTIONS

10.1 The Ideal Spring and Simple Harmonic Motion

An object is in harmonic motion if it follows a repeated path at regular time intervals, that
is, it is oscillating. Two common examples of harmonic motion often studied in physics
are a mass on a spring and a pendulum.

123
Chapter 10 Simple Harmonic Motion

As a mass on a spring vibrates, it has both a period and a frequency. The period of
vibration is the time it takes for one complete cycle of motion, that is the time it takes for
the object to return to its original position. The frequency is the number of cycles per unit
time, such as cycles per second, or hertz. The lowest point in the swing of a pendulum is
called the equilibrium position, and the maximum displacement from equilibrium is
called the amplitude.

Since the mass on a spring vibrates about the equilibrium position, there must be a force
that is trying to restore it back toward the center of the oscillation. This force is called the
restoring force, and it is greatest at the amplitude and zero as the mass passes through the
equilibrium position. Newton’s second law tells us that if there is a net force, there must
be an acceleration, and if the force is maximum at the amplitude, the acceleration must
maximum at the amplitude as well. The velocity, however, is zero at the amplitude and
maximum as it passes through the equilibrium position.

Equilibrium

The period and frequency of a mass vibrating on a spring depend on the stiffness in the
spring. For a stiffer spring, it takes more force to stretch the spring to a particular length.
The amount of force needed per unit length is called the spring constant k, measured in
newtons per meter. The relationship between force, stretched length, and k for an ideal
(or linear) spring is called Hooke’s law:

F spring = - kx

where x is the stretched length of the spring. For an ideal spring, the stretch is
proportional to the force, but in the opposite direction. If we pull with twice the force, the
spring will stretch twice as far.
The graph below represents the magnitude of the force F vs stretched length x for a
spring:

124
Chapter 10 Simple Harmonic Motion

The slope of the line is the change in force (rise) divided by the change in length (run).
Since this ratio is also equal to the spring constant k, the higher slope of the graph the
higher the spring constant, which is an indication of the stiffness of the spring.

We can find the spring constant k for this spring by taking the ratio of the force to the
stretch for a particular interval. In other words, we can find the slope of the F vs. x graph
for each spring. The slope of the line and the spring constant for spring is 50 N/m.

As on any other F vs. x graph, the work done in stretching or compressing the spring to a
certain displacement can be found by finding the area under the graph. The total work
done in stretching this spring is

W = Area =
1
(60 N )(1.2 m ) = 36 J
2
10.2 Simple Harmonic Motion and the Reference Circle
In the equation for the spring force above, x is the displacement from equilibrium
position at any time. Because of the oscillatory nature of the vibrating mass, we can
express the displacement x from equilibrium position at any time t as

x = A cos(ω t ) or x = A sin (ω t )

where A is the amplitude of oscillation and ω (the lower case Greek letter omega) is
called the angular frequency, and is measured in radians per second. The quantity ωt is
called the phase, and is an angle in radians. The mass on the spring makes one full
oscillation (2π radians) in one period T, so the angular frequency can be found by


ω= = 2π f , where f is the frequency in hertz.
T

Another relationship between the angular frequency of a mass oscillating on a spring and
the spring constant k is

k
ω= .
m

We see from this equation that the higher the spring constant k, the stiffer the spring, and
the greater the angular frequency of oscillation. A smaller mass will also increase the
angular frequency for a particular spring.

If we set the two equations above for ω equal to each other and solve for the period T of
oscillation, we get

m
T = 2π .
k

125
Chapter 10 Simple Harmonic Motion

Example 1
A mass of 0.5 kg oscillates on the end of a spring on a horizontal surface with negligible
friction according to the equation x = A cos(ω t ) . The graph of F vs. x for this motion is
shown below.

The last data point corresponds to the maximum displacement of the mass.
Determine the
(a) angular frequency ω of the oscillation,
(b) frequency f of oscillation,
(c) amplitude of oscillation,
(d) displacement from equilibrium position (x = 0) at a time of 2 s.

Solution:
(a) We know that the spring constant k = 50 N/m from when we looked at this graph
earlier. So,
k 50 N / m rad
ω= = = 10
m 0.5 kg s
ω 10 rad / s
(b) f = = = 1.6 Hz
2π 2π
(c) The amplitude corresponds to the last displacement on the graph, A = 1.2 m.
(d) x = A cos(ωt ) = (1.2 m ) cos[(10 rad / s )(2 s )] = 0.5 m

10.3 Energy and Simple Harmonic Motion


As an object vibrates in harmonic motion, energy is transferred between potential energy
and kinetic energy. Consider a mass sitting on a surface of negligible friction and
attached to a linear spring. If we stretch a spring from its equilibrium (unstretched)
position to a certain displacement, we do work on the mass against the spring force. By
the work-energy theorem, the work done is equal to the stored potential energy in the
spring. If we release the mass and allow it to begin moving back toward the equilibrium
position, the potential energy begins changing into kinetic energy. As the mass passes
through the equilibrium position, all of the potential energy has been converted into

126
Chapter 10 Simple Harmonic Motion

kinetic energy, and the speed of the mass is maximum. The kinetic energy in turn begins
changing into potential energy, until all of the kinetic energy is converted into potential
energy at maximum compression.

The compressed spring then accelerates the mass back through the equilibrium to the
original starting position, and the entire process repeats itself. If we neglect friction on the
surface and in the spring, the total energy of the system remains constant, that is,

Total Energy = Potential Energy + Kinetic Energy = a constant

Thus, whatever potential energy is lost must be gained by kinetic energy, and vice-versa.
As long as no energy is lost to the surroundings, the mass on the spring continues to
oscillate in mechanical resonance. As the spring oscillates, we can calculate the total
mechanical energy at any time:
Etotal = K + U
1 2 1
where K = mv and U = kx 2
2 2
Example 2
A spring of constant k = 100 N/m hangs at its natural length from a fixed stand. A mass
of 3 kg is hung on the end of the spring, and slowly let down until the spring and mass
hang at their new equilibrium position.

(a) Find the value of the quantity x in the figure above.

127
Chapter 10 Simple Harmonic Motion

The spring is now pulled down an additional distance x and released from rest.

(b) What is the potential energy in the spring at this distance?


(c) What is the speed of the mass as it passes the equilibrium position?
(d) How high above the point of release will the mass rise?
(e) What is the period of oscillation for the mass?

Solution:
(a) As it hangs in equilibrium, the upward spring force must be equal and opposite to the
downward weight of the block. Fs

Fs = mg
kx = mg

x= =
(
mg (3 kg ) 10 m / s 2)= 0.3 m
k 100 N / m

mg

(b) The potential energy in the spring is related to the displacement from equilibrium
position by the equation
U = kx 2 = (100 N / m )(0.3m ) = 4.5 J
1 1 2

2 2
(c) Since energy is conserved during the oscillation of the mass, the kinetic energy of the
mass as it passes through the equilibrium position is equal to the potential energy at the
amplitude. Thus,

1 2
K =U = mv
2
2U 2(4.5 J )
v= = = 1.7 m / s
m 3 kg
(d) Since the amplitude of the oscillation is 0.3 m, it will rise to 0.3 m above the
equilibrium position.
m 3 kg
(e) T = 2π = 2π = 1.1 s
k 100 N / m

10.4 The Pendulum


A pendulum is a mass on the end of a string which oscillates in harmonic motion. All of
the concepts for the period, frequency, amplitude, and energy for a pendulum are the
same as for the mass on a spring. The equation for the period of a pendulum is
L
T = 2π
g

128
Chapter 10 Simple Harmonic Motion

where L is the length of the pendulum, and g is the acceleration due to gravity at the
location of the pendulum. A longer length will have a longer period, while a stronger
gravitational field will shorten the period of a pendulum.

Example 3
A pendulum of mass 0.4 kg and length 0.6 m is pulled back and released from and angle
of 10˚ to the vertical.

(a) What is the potential energy of the mass at the instant it is released. Choose potential
energy to be zero at the bottom of the swing.
(b) What is the speed of the mass as it passes its lowest point?

This same pendulum is taken to another planet where its period is 1.0 second.
(c) What is the acceleration due to gravity on this planet?

Solution
(a) First we must find the height above the lowest point in the swing at the instant the
pendulum is released.

Recall from chapter 1 of this study guide L


10˚
that h = L − L cos θ .
Then L
U = mg (L − L cos θ )
( )
h
U = (0.4 kg ) 10 m / s 2 (0.6 m − 0.6 m cos10°) = 0.4 J

(b) Conservation of energy:


1
U max = K max = mv 2
2
2U 2(0.4 J )
v= = = 1.4 m / s
m 0.4 kg
(c)
L
T = 2π
g
4π 2 L 4π 2 (0.6 m ) m
g= = = 23.7 2
T 2
(1.0 s ) 2
s

129
Chapter 10 Simple Harmonic Motion

CHAPTER 10 REVIEW QUESTIONS


For each of the multiple choice questions below, choose the best answer.
Unless otherwise noted, use g = 10 m/s2 and neglect air resistance.

1. According to Hooke’s law for an ideal 3. The spring constant k is equal to


spring, doubling the stretch distance will (A) 5 N/m
(A) double the velocity of the mass. (B) 10 N/m
(B) double the force that the spring (C) 20 N/m
exerts on the mass. (D) 25 N/m
(C) quadruple the force the spring exerts (E) 50 N/m
on the mass.
(D) double the period.
(E) double the frequency.
L
Questions 2 – 3: Consider the force vs θ
displacement graph shown for an ideal
spring.

Questions 4 – 6:
A pendulum of length L swings with an
amplitude θ and a frequency f as
shown above.

4. If the amplitude is increased and the


pendulum is released from a greater
angle,
(A) the period will decrease.
(B) the period will increase.
(C) the period will not change.
(D) the frequency will increase.
(E) the frequency will decrease.
2. The work done in stretching the spring
from 0.1 m to 0.5 m is 5. If the mass and the length of the
(A) 1 J pendulum are both doubled, the
(B) 4 J frequency of vibration will be
(C) 6 J (A) f
(D) 12 J (B) 2f
(E) 24 J (C) 4f
(D) ½ f
(E) ¼ f

130
Chapter 10 Simple Harmonic Motion

6. Which of the following statements is 8. The period of vibration for this mass
true about the swinging pendulum? on a spring is most nearly
(A) 1 s
I. The greatest restoring force and the (B) 2 s
greatest velocity occur at the same (C) 3 s
point. (D) 6 s
II. The greatest restoring force and the (E) 9 s
greatest acceleration occur at the same
point. 9. The total distance traveled by the
III. The greatest acceleration and the mass during one full oscillation is
greatest velocity occur at the same (A) 3 cm
point. (B) 6 cm
(C) 12 cm
(A) I only (D) 18 cm
(B) I and II only (E) 24 cm
(C) II only
(D) I and III only
(E) I, II, and III P

Questions 7 – 9:
The equation which describes the motion
of a mass oscillating on an ideal spring is Equilibrium
x = 6 cos 3t
where x is in centimeters and t is in 10. A mass vibrates on an ideal spring as
seconds. shown above. The total energy of the
spring is 100 J. What is the kinetic
7. The amplitude of the harmonic motion energy of the mass at point P, halfway
is between the equilibrium point and the
(A) 3 cm amplitude?
(B) 6 cm (A) 25 J
(C) 9 cm (B) 50 J
(D) 18 cm (C) 75 J
(E) 30 cm (D) 100 J
(E) 200 J

131
Chapter 10 Simple Harmonic Motion

Free Response Question

Directions: Show all work in working the following question. The question is worth 15
points, and the suggested time for answering the question is about 15 minutes. The parts
within a question may not have equal weight.

1. (15 points)

ℓ x
B C D

A small 0.10 kg block starts from rest at point A, which is at a height of 1.0 m. The
surface between points A and B and between points C and D is frictionless, but is rough
between points B and C, having a coefficient of friction of 0.10. After traveling the
distance ℓ = 1.0 m, the small block strikes a larger block of mass 0.30 kg, and sticks to it,
compressing the spring to a maximum distance x = 0.50 m. Determine

(a) the speed of the 0.10 kg block at point B.


(b) the acceleration of the 0.10 kg block between points B and C.
(c) the speed of the block at point C.
(d) the speed of the combined small and large block immediately after they collide.
(e) the spring constant of the spring.

132
Chapter 10 Simple Harmonic Motion

ANSWERS AND EXPLANATIONS TO CHAPTER 10 REVIEW QUESTIONS

Multiple Choice

1. B
Since the force is proportional to the stretch distance, the force would double if the
stretch distance doubled.

2. C
The work done by the spring from 0.1 m to 0.5 m is equal to the area under the graph
between those two points.
A = (20 N )(0.4 m ) + (5 N )(0.4 m ) = 6 J
1
2
3. E
25 N N
k = slope = = 50
0.5 m m
4. C
The period of a pendulum does not depend on the amplitude of swing for small swings.

5. D
1 g 1 1 1
f = . Thus , f ∝ , and f ∝
2π L L 2 4L

6. C
Newton’s second law states that the acceleration is proportional to the force. The
restoring force and the acceleration are the greatest at the amplitude of swing.

7. B
The amplitude is the the constant which appears in front of the cosine of the angle.

8. B
2π 2(3)
T= ≈ = 2s
ω 3

9. E
The total distance moved during one full oscillation would be four times the amplitude of
the motion: 4(6 cm) = 24 cm.

10. B
At point B, the mass is halfway between the equilibrium position, where the kinetic
energy is 100 J, and the amplitude, where the kinetic energy is zero. Thus, the kinetic
energy at point B is 50 J.

133
Chapter 10 Simple Harmonic Motion

Free Response Question Solution

(a) 3 points
U A = KB
1
mgh =
2
mv B
2
( )
v B = 2 gh = 2 10 m / s 2 (1.0 m ) = 4.5 m / s

(b) 3 points
The frictional force and acceleration between points B and C is
f = µN = µmg = ma
µmg
a= =−
m
f
m
( )
= − µg = −(0.10 ) 10 m / s 2 = −1.0 m / s 2

(c) 2 points
v C = v B + 2 a
2 2

v C = v B + 2 a =
2
(4.5 m / s )2 + 2(− 1.0 m / s 2 )(1.0 m ) = 4.3 m / s
(d) 3 points
Conservation of momentum:

mvC = (m + M )vC

vC =
mvC
=
(0.10 kg )(4.3 m / s ) = 1.1 m / s
(m + M ) (0.40 kg )
(e) 4 points
The kinetic energy of the two blocks just after the collision is equal to the potential
energy in the spring.

KC = U D

(m + M )vC ′ = 1 kx 2
2
1
2 2
(m + M )vC ′ (0.40 kg )(1.1 m / s )2
2

k= = = 1.9 N / m
x2 (0.5 m )2

134
Chapter 16 Waves and Sound

Chapter 16

WAVES AND SOUND

PREVIEW

A wave is a disturbance which causes a transfer of energy. Mechanical waves need a


medium in which to travel, but electromagnetic waves do not. Waves can be transverse or
longitudinal, depending on the direction of the vibration of the wave. Sound is a
longitudinal wave. A wave is characterized by its frequency, period, wavelength,
amplitude, and speed. Waves can be reflected, refracted, diffracted, and two waves in the
same medium will interfere. An apparent change in frequency of a wave because of
relative motion is called the Doppler effect.

The content contained in sections 1, 2, 3, 5, 6, 9, and 12 of chapter 16 of the textbook is


included on the AP Physics B exam.

QUICK REFERENCE

Important Terms

amplitude
maximum displacement from equilibrium position; the distance from the midpoint
of a wave to its crest or trough.
crest of a wave
the highest point on a wave
decibel
the unit for the loudness of a sound; one-tenth of a Bel
Doppler effect
apparent change in frequency of a wave due to motion between the source of the
wave and the detector of the wave
frequency
the number of vibrations of a wave per unit of time
longitudinal wave
wave in which the vibration of the medium is parallel to the direction of motion of
the wave
loudness
the quality of a sound wave which is measured by its amplitude
mechanical wave
a wave which uses a material medium through which to transfer energy
period
the time for one complete cycle or revolution
periodic wave
a wave that repeats itself at regular intervals of time

179
Chapter 16 Waves and Sound

pitch
the perceived sound characteristic equivalent to frequency
rarefaction
and expansion of the medium in a longitudinal wave
transverse wave
a wave in which the vibration is perpendicular to the velocity of the wave
trough of a wave
the low point of wave motion
wavelength
the distance between successive identical parts of a wave

Equations and Symbols


1 where
f =
T
λ f = frequency
v= = fλ T = period
T λ = wavelength
  f o = frequency measured by the observer
 
1
fo = fs   f s = frequency produced by the source
 vs  v s = speed of the source of the wave
1± 
 v  v = speed of the wave

Ten Homework Problems


Chapter 16 Problems 4, 6, 8, 9, 13, 14, 34, 35, 82, 95

DISCUSSION OF SELECTED SECTIONS

16.1 and 16.2 The Nature of Waves and Periodic Waves

A mechanical wave is a traveling disturbance in a medium which transfers energy from


one place to another. A medium is the substance through which a wave moves, such as
water for a water wave, or air for a sound wave. An electromagnetic wave is a vibration
of an electric and magnetic field which travels through space at an extremely high speed,
and does not need a medium through which to travel. Light, radio waves, and
microwaves are all examples of electromagnetic waves. We will return to
electromagnetic waves later.

There are two types of mechanical waves. Transverse waves vibrate in a direction which
is perpendicular to the direction of motion of the wave. For example if you hold the end
of a spring and vibrate your hand up and down, you create a transverse wave in the
spring.

180
Chapter 16 Waves and Sound

If you gather the spring up into a bunch and let it go, you create a longitudinal wave, in
which the spring vibrates in a direction which is parallel to the direction of motion of the
wave.

Sound is a common example of a longitudinal wave, since the air through which a sound
wave moves is repeatedly compressed and expanded.

Since an object vibrating with simple harmonic motion can create a wave in a medium, it
is not surprising that many of the terms we discussed in chapter 10 can also be applied to
waves. A wave has a period, the time it takes for a wave to vibrate once, a frequency, the
number of waves that pass a given point per second, and an amplitude, the maximum
displacement of a wave, or its height. The length of one complete vibration of a wave is
called the wavelength, and is denoted by the Greek letter lambda, λ. The figure below
illustrates these quantities.
λ crest

trough

The crest is the highest point on the wave and the trough is the lowest point on the wave.
The wavelength can be measured from one crest to the next crest, or from one trough to
the next trough.

The speed of a wave can be found by the equation v = fλ , where v is the speed, f is the
frequency, and λ is the wavelength. Since frequency is the reciprocal of period, we can
λ
also write this equation as v = . The speed of all types of waves can be found using this
T
equation.

16.3 The Speed of a Wave on a String


The speed of a wave passing through a tight string with tension F T is proportional to the
square root of the tension in the string. As the tension of the string is increased, the speed
of the wave increases. However, when the tension in the string changes, the frequency of
the wave is not affected if it is produced by an outside source such as a tuning fork or
vibrating machine. Therefore when the tension and thus the speed increases, the
wavelength must also increase by the equation v = fλ.

low speed, short wavelength high speed, long wavelength

181
Chapter 16 Waves and Sound

Example 1

A string is attached to a tuning fork of frequency 256 Hz, and a wave travels along the
string with a speed of 200 m/s.
(a) Determine the wavelength of the wave in the string.
(b) If the tension in the same string is increased to four times its initial value, find
i. the speed of the wave
ii. the wavelength of the wave.

Solution
v 200 m / s
(a) λ = = = 0.8 m
f 250 Hz
(b) i. Since v ∝ FT , 2v ∝ 4 FT

So four times the tension produces twice the speed, or 400 m/s.
v 400 m / s
ii. λ = = = 1.6 m
f 250 Hz
We will say more about speed and tension in a string in chapter 17.

16.5 and 16.6 The Nature of Sound and The Speed of Sound

Sound is a mechanical longitudinal wave, and therefore must have a medium through
which to travel. Sound generally travels at about 340 m/s in air, but travels at
considerably higher speeds in more dense media such as water or steel. The
characteristics of sound which are produced and how we detect and perceive these
characteristics are summarized in the table below.

Sound produced as: Sound detected as:


Frequency Pitch
Amplitude Loudness or volume
Harmonics Quality or tone

The third characteristic, harmonics, is the combination of several simultaneous


frequencies that give a sound its special tone. For example, we can tell the difference
between a trumpet and a clarinet because our ear detects the special harmonics, even if
they are playing the same pitch at the same loudness. For the same reason, we can tell the
difference between two voices.

182
Chapter 16 Waves and Sound

16.9 The Doppler Effect

When a sound source is moving toward you, you hear a slightly higher pitch than if the
sound source is at rest relative to you. By the same token, when a sound source is moving
away from you, you hear a slightly lower pitch. This phenomena is called the Doppler
effect. For example, if a train is traveling toward you while blowing its horn at a certain
pitch, the waves will appear to be arriving at your ear more frequently, increasing the
pitch you perceive.

high low
pitch pitch

Listener

If the train blows its whistle while accelerating away from you, the waves will appear to
be arriving at your ear less frequently, decreasing the pitch you perceive. Remember, the
Doppler effect describes the apparent change in frequency (pitch), not amplitude
(loudness). Also, it doesn’t matter whether the sound source is moving or the observer is
moving, only that they have relative motion between the two of them.

Of course, the frequency of the whistle is not actually changing, but you perceive it to
change due to the relative motion between you and the train. The Doppler effect also
occurs when a light source is moving toward or away from us. The light spectrum of a
star, for example, is shifted toward the red (low frequency) end if the star is moving away
from us, and toward the blue (high frequency) end of the spectrum if it were to move
toward us.

The equations for calculating the apparent shift in frequency due to relative motion
between the source and observer are typically not needed on the AP Physics B exam, but
the concepts involved in the Doppler effect are included in the questions on the exam.

183
Chapter 16 Waves and Sound

CHAPTER 16 REVIEW QUESTIONS


For each of the multiple choice questions below, choose the best answer.

The following diagram of a wave relates 5. As a wave passes from a spring to


to questions 1 – 3. another spring with a greater tension,
6m (A) the speed of the wave decreases
(B) the frequency of the wave increases.
(C) the frequency of the wave decreases.
2m (D) the amplitude of the wave increases
(E) the speed of the wave increases.
t = 1.5s
Questions 6 – 7:
1. The wavelength of the wave is A wave source of constant frequency
(A) 0.5 m sends a wave through a tight string of
(B) 1.0 m uniform density with a speed v o and
(C) 2.0 m wavelength λ o . The tension is then
(D) 4.0 m relaxed to half its initial tension.
(E) 6.0 m
6. The speed of the wave is now
2. The amplitude of the wave is (A) vo
(A) 0.5 m (B) 2vo
(B) 1.0 m
(C) 2.0 m (C) 4vo
(D) 4.0 m (D) 2vo
(E) 6.0 m 1
(E) vo
3. The frequency of the wave is 2
(A) 2.0 Hz
(B) 3.0 Hz 7. The wavelength of the wave is now
(C) 1.5 Hz (A) λo
(D) 0.5 Hz (B) 2λo
(E) 1.0 Hz
(C) 4 λo
4. A girl on the beach watching water (D) 2λ o
waves sees 4 waves pass by in 2 1
seconds, each with a wavelength of (E) λo
2
0.5 m. The speed of the waves is
(A) 0.25 m/s
8. The Doppler effect produces apparent
(B) 0.5 m/s
changes in
(C) 1.0 m/s
(A) loudness
(D) 2.0 m/s
(B) frequency
(E) 4.0 m/s
(C) amplitude
(D) velocity
(E) acceleration

184
Chapter 16 Waves and Sound

9. A car’s horn emits a constant frequency as it accelerates away from a stationary listener.
Which of the following quantities actually changes for the listener?
I. frequency of the sound
II. pitch of the sound
III. amplitude of the sound
IV. loudness of the sound

(A) I only
(B) I and II only
(C) III and IV only
(D) II and IV only
(E) I, II, III, and IV

Free Response Question

Directions: Show all work in working the following question. The question is worth 10 points,
and the suggested time for answering the question is about 10 minutes. The parts within a
question may not have equal weight.

1. (10 points)

A tuning fork of frequency 300 Hz is activated and sends a sound wave toward a classroom wall,
and the echo is detected at the location of the tuning fork 0.06 s later.
(a) Determine the wavelength of the sound wave.
(b) Determine the distance from the tuning fork to the wall.

185
Chapter 16 Waves and Sound

The same tuning fork is mounted vertically on a ring stand as shown below.
A string of length 2 m is attached to the tuning fork and a
mass m is hung on the end of the string. The tuning fork is
activated, and a wave passes through the string (the
size of the amplitude of the wave is exaggerated for clarity).
Assume that the tension in the string does not affect the
frequency of vibration of the tuning fork.

(c) If the speed of the wave is 600 m/s when the mass m
is hung on the end of the string, how many full wavelengths
will occupy the string?

(d) If the mass m is replaced with a mass of 4m, how many


wavelengths (or what fraction of a wavelength) will occupy
the string?

ANSWERS AND EXPLANATIONS TO CHAPTER 16 REVIEW QUESTIONS

Multiple Choice

1. C
In the diagram of the wave shown, three wavelengths occupy a space of 6 m. Thus, one
wavelength is 2.0 m.

2. B
The amplitude is the distance from the base line of the wave to the crest, or half the distance
from the trough to the crest, 1.0 m.

3. A
It takes a time of 1.5 s for three waves to pass by, so it takes 0.5 s for one wave to pass by (the
period). The frequency of the wave is the inverse of the period, or 2.0 Hz.

4. C
Four waves in 2 s implies 2 waves in 1 s, or a frequency of 2 Hz. Then the speed of the wave is
v = fλ = (2 Hz )(0.5m ) = 1.0 m / s

186
Chapter 16 Waves and Sound

5. E
Higher tension (tightness) in the spring will allow the wave to move with a higher speed.

6. E
vo
v ∝ FT , so halving the tension force F T gives a speed of .
2
7. E
λo
Since wavelength and speed are proportional, the new tension will result in a wavelength of .
2

8. B
The Doppler effect describes apparent changes in frequency, although the loudness of a wave
may also be changing.

9. D
The frequency and amplitude of the wave do not actually change, but the perception of these
(pitch and loudness) will change according to the listener.

Free Response Question Solution

(a) 2 points
At room temperature, the speed of a sound wave v = 343 m/s. Thus,
v 343 m / s
λ= = = 1.1 m
f 300 Hz
(b) 3 points
If the time for the sound wave to make a round trip is 0.06 s, then the distance from the tuning
fork to the wall corresponds to 0.03 s.
d = vt = (343 m / s )(0.03 s ) = 10.3 m

(c) 3 points
The wavelength of the wave in the string is
v 600 m / s
λ= = = 2.0 m
f 300 Hz
Since the string is 2.0 m long, one wavelength will just fit in the length of the string.

(d) 2 points
Four times the mass hung on the end of the string will produce 4 times the tension in the string,
doubling the speed and wavelength in the string. The new wavelength will be 4 m long, so only
half the wavelength will fit into the string with 4m hanging on it.

187
Chapter 17

THE PRINCIPLE OF LINEAR SUPERPOSITION AND


INTERFERENCE PHENOMENA

PREVIEW

When two waves meet in the same medium they combine to form a new wave by the principle of
superposition. The result of superposition is called interference. If the two waves are in phase at
a point, they will interfere constructively. At a point where the two waves are out of phase they
will interfere destructively. Places where maximum constructive and destructive interference are
occurring are called antinodes and nodes, respectively. Interference can be caused by point
sources, or by a wave passing through a single – slit opening or a double – slit opening.
Interference in a vibrating string, open pipe, or closed pipe produces resonance conditions called
harmonics and overtones.

The content contained in sections 1, 2, 3, 5, 6, and 8 of chapter 17 of the textbook is included on


the AP Physics B exam.

QUICK REFERENCE

Important Terms

antinode
a point of constructive interference and maximum reinforcement in a standing
wave
beat
variations in the loudness of sounds due to the slight difference in frequency of
interfering waves
closed pipe resonator
a pipe closed at one end and a sound source at the other, causing the sound to
resonate
constructive interference
addition of two or more waves which are in phase, resulting in a wave of
increased amplitude
destructive interference
addition of two or more waves which are out of phase resulting in a wave of
decreased amplitude
diffraction
the spreading of a wave beyond the edge of a barrier or through an opening
harmonics
the combination of several simultaneous frequencies that produce a distinct
standing wave pattern

188
in phase
term applied to two or more waves whose crests and troughs arrive at a place at
the same time in such a way as to produce constructive interference
interference of waves
displacements of two or more waves in the same medium at the same time
producing either larger or smaller waves
node
the point of no displacement in a standing wave
out of phase
term applied to two or more waves for which the crest of one wave arrives at a
point at the same time as the trough of a second wave arrives, producing
destructive interference
overtone
a resonance condition (standing wave) which is above the fundamental
principle of superposition
the displacement due to two or more interfering waves is equal to the sum of the
displacement of the individual waves
standing wave
wave with stationary nodes produced by two identical waves traveling in opposite
directions in the same medium at the same time

Equations and Symbols


λ
sin θ =
D
 v 
f n = n  n = 1,2,3,4,... (vibrating string and open tube)
 2L 
 v 
f n = n  n = 1,3,5,... (closed tube)
 4L 
where
θ = the angle of spread of a wave after passing through a single slit opening
λ = wavelength
f n = the nth frequency in a series of resonant frequencies
v = speed of the wave
L = length
D = width of an opening through which a wave will diffract

189
Ten Homework Problems
Chapter 17 Problems 1, 2, 3, 10, 24, 26, 29, 33, 49, 52

DISCUSSION OF SELECTED SECTIONS

17.1 The Principle of Linear Superposition


When two or more waves pass through each other in the same medium at the same time they
interfere, as shown in the diagrams on the opening pages of chapter 17 in your textbook. Two
waves meeting on the same side of the spring will interfere constructively, producing an
antinode, and two waves meeting on opposite sides of the spring will interfere destructively and
cancel the waves momentarily, producing a node. Either way, the principle of superposition
states that any two waves which meet at a point may be added (or subtracted, if they meet on
opposite sides). We say that two waves which interfere constructively are in phase, and two
waves that interfere destructively are out of phase.

17.2 Constructive and Destructive Interference of Sound Waves


Have you ever sat in a room or auditorium in a place where it was difficult to hear the speaker?
Like other waves, sound waves interfere constructively and destructively as they are produced
and reflected around a room. For two sound waves which are in phase (crest on crest), the path
difference between the waves is either zero or a whole number of wavelengths apart (Fig. A):

Fig. A Waves in phase Fig. B Waves out of phase

For two sound waves which are out of phase (crest on trough), the path difference between the
waves is a half-number of wavelengths apart (Fig. B). Example 1 in your textbook is a good
illustration of whether waves arrive at a particular point in phase or out of phase.

190
17.3 Diffraction
As a wave passes through an opening, the middle of the wave is unhindered, but the outside
edges drag on the sides of the opening, momentarily slowing those points on the wave down.
Thus, the sides of the wave lag behind the center of the wave, and a circular wave pattern is
produced as the wave exits the opening. The bending of a wave through an opening or around a
barrier is called diffraction.

Waves Intensity

1st min
θ
D Central max
θ
1st min

The relationship between D, λ, and θ is


λ
sin θ =
D

Notice that if the wavelength is small compared to the size of the opening, the diffraction angle is
small, and we may not notice much diffraction, that is, the wave will pass through the opening
unhindered.

Example 1
Sound is passed through a tall narrow doorway (width D = 1.0 m) in a wall on a stage in an
auditorium. A singer stands behind the doorway and sings a high-C pitch (f = 512 Hz).

(a) What is the wavelength of the high-C pitch?

191
Your friend really enjoys listening to this particular singer, but you do not. Your friend sits
directly in from of the doorway at a distance L = 15 m.

Waves Intensity

y
D

How far (y) away from your friend should you sit in order to hear the least intense sound waves
from the singer?

Solution
v 343 m / s
(a) λ = = = 0.67 m
f 512 Hz

λ y
(b) sin θ = , and sin θ = . So,
D L
y λ
=
L D
λL (0.67 m )(15 m )
y= = =10 m
D 1.0 m

Another type of standing wave is produced when waves are passed through two openings, called
a double-slit, and an interference pattern results. The semi-circular wave patterns which emerge
from the slits interfere with each other, creating nodes and antinodes. We will return to double-
slit diffraction patterns in chapter 27.

17.5 Transverse Standing Waves


The term standing wave is an oxymoron, since waves must move and never stand still. But
waves can appear to stand still when two identical waves traveling in opposite directions in the
same medium at the same time create a series of nodes and antinodes. Consider a rope tied to a
wall. If we send a wave down the rope toward the wall, the wave reflects off the wall on the
opposite side from which it was sent, according to the law of reflection.

192
If we continue to send regular waves down the rope and they continue to reflect off the wall, the
incident and reflected waves will reinforce each other in some places and cancel each other in
other places. The result is a series of antinodes (loops) where constructive interference is
occurring, and nodes (points of no displacement between the loops) where destructive
interference is occurring, and we call the pattern produced a standing wave.

A N A N A

The pattern above, called a harmonic or overtone, shows three antinodes (loops), and occupies
3
of a wavelength.
2

Example 2
A string is attached to a vibrating machine which has a frequency of 120 Hz. The other end of
the string is passed over a pulley of negligible mass and friction and is attached to a weight
hanger which holds a mass m = 0.5 kg.

(a) Determine the tension in the string.


(b) The speed of the wave in the string is related to the tension by the equation
FT
v= , where F T is the tension in the string and μ is the linear density of the string. If the
µ
linear density of this string is 0.05 kg/m, determine the speed of the wave in the string.

193
(c) Determine the wavelength of the wave in the string.
(d) Determine the length of the string from the point of attachment on the vibrating machine to
the pulley.
(e) Would you need to increase or decrease the mass on the hanger to produce a lower number of
loops? Explain.

Solution
( )
(a) FT = mg = (0.5kg ) 10 m / s 2 = 5 N
FT 5N
(b) v = = = 100 m / s
µ 0.05 kg / m
v 100 m / s
(c) λ = = = 0.83 m
f 120 Hz
(d) L = 3λ = 3(0.83 m ) = 2.5 m
(e) A lower number of loops would imply a longer wavelength, which would require a higher
speed, which would require a higher tension in the string, which would require increasing the
mass on the hanger.

17.6 Longitudinal Standing Waves


An open pipe is one in which both ends of the pipe are open. If we send a sound wave into the
pipe and let it reflect off of the air at other end, it will return to the original end of the pipe. If we
continually send sound waves down into the pipe to match the rate at which they are being
reflected off the closed end, we will set up a resonance condition, that is, a standing wave. as
shown in the figure below:

A N A A N A N A

1st Harmonic 2nd Harmonic


Fundamental Overtone 1st Overtone

A standing wave pattern is set up in the open pipe, having a series of nodes and antinodes. Note
that there is an antinode at either end of the pipe. When a standing sound wave fits inside the
pipe this way, we hear a louder pitch than when the sound does not fit inside the pipe. We say
that the sound is resonating inside the pipe.

Note that in the first harmonic of the open pipe, ½ of a wavelength just fits inside the length of
the pipe. In the 2nd harmonic, one full wavelength just fits inside the pipe.

194
1st Harmonic
Fundamental Overtone
3rd Harmonic
1st Overtone
5th Harmonic
2nd Overtone

A closed pipe is one in which one end of the pipe (bottom) is closed and the other end is open
(top). If we send a sound wave into the pipe and let it reflect off of the closed end, it will return
to the top of the pipe, as shown in the figures above. This results in an antinode at the top open
end of the pipe, and a node at the bottom closed end. Again, when a standing sound wave fits
inside the pipe this way, we hear a louder pitch (resonance) than when the sound does not fit
inside the pipe.

Note that in the 1st harmonic of the closed pipe, ¼ of a wavelength just fits inside the length of
the pipe, ¾ wavelength for the 3rd harmonic, and 5/4 of a wavelength for the 5th harmonic. The
harmonics are named after each quarter-wavelength that fits in the pipe as the sound resonates.

If we know the frequency of the sound waves and the length of the pipe, we can find their
wavelength and then their speed by v = fλ. The relationships among all these quantities in a
resonating pipe are illustrated in the review questions that follow.

195
CHAPTER 17 REVIEW QUESTIONS
For each of the multiple choice questions below, choose the best answer.

Questions 1 – 2:

P
2. Which of the following diagrams best
represents the shape of the rope just after
Q
the two waves have completely passed
1. Two waves approach each other in the through the region between points P and
same rope at the same time, Q?
as shown. When the two waves are
exactly between points P and Q, the (A) P
shape of the rope will be
Q
(A)
(B)

(B) P

Q
(C) Q
P

(C)
(D) Q
P

(D)

(E)

(E) 3. In general, the higher the frequency


of the interfering waves,
(A) the higher the wavelength
(B) the higher the speed of the waves
(C) the greater the number of nodes and
antinodes.
(D) the smaller the number of nodes and
antinodes.
(E) the higher the amplitude of the
waves.

196
4. Diffraction 7. Which of the following is true of a
(A) is the bending of a wave when it sound which is resonating in a pipe
changes media which is closed at one end?
(B) is the reflection of a wave off of a (A) Nodes are formed at both ends of the
barrier pipe
(C) is the combining of two identical (B) Antinodes are formed at both ends of
waves moving in the same medium the pipe
(D) is the bending of a wave around an (C) An antinode is formed at the closed
obstacle or through an opening end of the pipe and a node is formed
(E) only produces constructive at the open end.
interference (D) An antinode is formed at the open
end of the pipe and a node is formed
5. Wave A having a wavelength of 2 cm at the closed end.
passes through an opening which is 1 cm (E) A sound wave cannot resonate in a
wide. Wave B has a wavelength of 4 cm pipe which is closed at only one end.
and passes through the same opening at
a different time. The angle of spread θ A 8. A N A
for wave A compared to the angle of
spread θ B for wave B can be described
by
(A) θ A > θ B
(B) θ B > θ A
(C) θ A = θ B 1st Harmonic
Fundamental Overtone
(D) θ A + θ B = 90
(E) θ A - θ B = 90
The figure above represents the 1st
harmonic for an open pipe. If the length
of the pipe is 40 cm, the wavelength of
1.5 m
the sound wave resonating inside the
pipe is
(A) 10 cm
(B) 20 cm
(C) 40 cm
(D) 80 cm
6. A standing wave is produced in a (E) 160 cm
vibrating string as shown. If the length
of the string is 1.5 m and the frequency
of the vibrating motor is 60 Hz, the
speed of the wave is
(A) 15 m/s
(B) 20 m/s
(C) 40 m/s
(D) 60 m/s
(E) 90 m/s

197
Free Response Question

Directions: Show all work in working the following question. The question is worth 10 points,
and the suggested time for answering the question is about 10 minutes. The parts within a
question may not have equal weight.

1. (10 points)
A N A N A

2nd Harmonic
1st Overtone

A sound wave resonates inside an open pipe filled with air at room temperature, as shown above.
The length of the pipe is 33 cm.

(a) Determine the wavelength of the resonating sound wave.


(b) Determine the frequency of the tuning fork.
(c) Determine the next higher frequency that will resonate in a pipe of this length.

1st Harmonic
Fundamental Overtone

(d) If the open pipe is replaced with a pipe which is closed at one end, what would have to be the
length of the closed pipe for the original tuning fork to resonate at its fundamental frequency?

198
ANSWERS AND EXPLANATIONS TO CHAPTER 17 REVIEW QUESTIONS

Multiple Choice

1. B
The two waves are in phase (crest on crest), and so they will constructively interfere and produce
a larger wave between points P and Q.

2. A
After passing through the region between points P and Q, the waves will simply have switched
places, and since they are identical, the rope appears as it did before the waves met.

3. C
More frequency results in more waves interfering with each other, producing more nodes and
antinodes.

4. D
A wave diffracts when it must go around an obstacle or through an opening.

5. B
The longer wavelength passing through the same opening will have a greater angle of spread.

6. D
The wavelength is the length of two loops and is 1 m. The speed of the wave is v = f λ= (60
Hz)(1 m) = 60 m/s.

7. D
The wave is reflected off the closed end, creating a node at the closed end and an antinode at the
open end.

8. D
The wave pattern in the 40-cm pipe represents half of a wavelength. Thus, the wavelength is
2(40 cm) = 80 cm.

199
Free Response Question Solution

(a) 2 points
One wavelength just fits inside the pipe for the 2nd harmonic, so L = λ = 33 cm.

(b) 3 points
v 343 m / s
f = = = 1029 Hz
λ 0.33m

(c) 3 points
A N A N A
The next higher frequency would correspond
to two wavelengths in the pipe. Thus, L = 2λ,
and λ = ½ L = 16.5 cm. The speed of sound
is 343 m/s, so the next higher frequency is

v 343 m / s
f = = = 2058 Hz
λ 0.165 m

This is simply twice the original frequency.

(d) 2 points
The first resonance condition in a closed pipe requires the length of the pipe to be one-quarter the
wavelength, or

λ = (33 cm ) = 8.25 cm
1 1
L=
4 4

200
Chapter 17 The Principle of Linear Siperposition and Interference Phenomena

201
Chapter 18 Electric Forces and Electric Fields

Chapter 18

ELECTRIC FORCES AND ELECTRIC FIELDS

PREVIEW

Electric charge is the fundamental quantity that underlies all electrical phenomena. There
are two types of charges, positive and negative, and like charges repel each other, and
unlike charges attract each other. A conductor is a material through which charge can
easily flow due to a large number of free electrons, whereas an insulator does not allow
charge to flow freely through it. The force between charges can be found by applying
Coulomb’s law. The electric field around a charge is the force per unit charge exerted on
another charge in its vicinity.

The content contained in sections 1 – 8, and 11 of chapter 18 of the textbook is included


on the AP Physics B exam.

QUICK REFERENCE

Important Terms
charging by conduction
transfer of charge by actual contact between two objects
charging by induction
transfer of charge by bringing a charged object near a conductor, then grounding
the conductor
conservation of charge
law that states that the total charge in a system must remain
constant during any process
coulomb
the unit for electric charge
Coulomb’s law
the electric force between two charges is proportional to the product of
the charges and inversely proportional to the square of the distance between them
electric charge
the fundamental quantity which underlies all electrical phenomena
electric field
the space around a charge in which another charge will experience a force;
electric field lines always point from positive charge to negative charge
electron
the smallest negatively charged particle
electrostatics
the study of electric charge, field, and potential at rest

201
Chapter 18 Electric Forces and Electric Fields

elementary charge
the smallest existing charge; the charge on one electron or one
proton (1.6 x 10-19 C)
parallel plate capacitor
capacitor consisting of two oppositely charged parallel plates of equal area, and
storing an electric field between the plates
neutral
having no net charge
test charge
the very small charge used to test the strength of an electric field

Equations and Symbols

kq1 q 2 1 q1 q 2 where
F= =
r 2
4πε 0 r 2
F = electric force
F kq 1 q k = electric constant = 9x109 Nm2 / C2
E= = 2 =
q0 r 4πε 0 r 2 ε 0 = permittivity constant
= 8.85 x 10-12 C2 / Nm2
q (or Q) = charge
r = distance between charges
E = electric field

Ten Homework Problems


Chapter 18 Problems 11, 14, 18, 20, 23, 26, 34, 35, 42, 65

DISCUSSION OF SELECTED SECTIONS

18.2 - 18.3 Charged Objects and the Electric Force, Conductors and
Insulators
Charge is the fundamental quantity that underlies all electrical phenomena. The symbol
for charge is q, and the SI unit for charge is the Coulomb (C). The fundamental carrier of
negative charge is the electron, with a charge of – 1.6 x 10-19 C. The proton, found in the
nucleus of any atom, carries exactly the same charge as the electron, but is positive. The
neutron, also found in the nucleus of the atom, has no charge. When charge is transferred,
only electrons move from one atom to another. Thus, the transfer of charge is really just
the transfer of electrons. We say that an object with a surplus of electrons is negatively
charged, and an object having a deficiency of electrons is positively charged. Charge is
conserved during any process, and so any charge lost by one object must be gained by
another object.

202
Chapter 18 Electric Forces and Electric Fields

The Law of Charges

The law of charges states that like charges repel each other and unlike charges attract
each other. This law is fundamental to understanding all electrical phenomena.

Example 1
Consider four charges, A, B, C, and D, which exist in a region of space. Charge A attracts
B, but B repels C. Charge C repels D, and D is positively charged. What is the sign of
charge A?

Solution
If D is positive and it repels C, C must also be positive. Since C repels B, B must also be
positive. A attracts B, so A must be negatively charged.

Charge is one of the four quantities in physics that is conserved during any process.

Example 2
Consider two charged spheres of equal size carrying a charge of +6 C and –4 C,
respectively. The spheres are brought in contact with one another for a time sufficient to
allow them to reach an equilibrium charge. They are then separated. What is the final
charge on each sphere?

+6 -4

Solution
When the two spheres come in contact with each other, charge will be transferred, but
the total amount of charge is conserved. The total charge on the two spheres is +6 C + -4
C = +2 C, and this is the magnitude of the equilibrium charge. When they are separated,
they divide the charge evenly, each keeping a charge of +1 C.

Conductors, like metals, have electrons which are loosely bound to the outskirts of their
atoms, and can therefore easily move from one atom to another. An insulator, like wood
or glass, does not have many loosely bound electrons, and therefore cannot pass charge
easily.

18.4 Charging by Contact and by Induction


We can give an object a net charge two ways: conduction (contact) and induction. In
order to charge an object by conduction, we must touch the object with a charged object.
giving the two objects the same charge sign.

Charging by induction gives us an object charged oppositely to the original charged


object. For example, as shown in your textbook, if we bring a negatively charged rod near
a conducting (metal) sphere, and then ground the metal sphere, negative charges on the
sphere escape to the ground, leaving the sphere with a net positive charge.

203
Chapter 18 Electric Forces and Electric Fields

Example 3
Show how we can begin with a positively charged rod and charge a metal sphere
negatively.

Take a moment to draw the charges on each of the objects in the sequence of diagrams
below.

++++++++ ++++++++

I II III

Solution
++++++++ ++++++++
-
- -
- + -
+ -
- + - -
+

ground

I II III

In figure I a positively charged rod is brought near a neutral metal sphere, separating the
charges in the sphere. When the sphere is grounded, the positive charges escape into the
ground (actually, electrons come up from the ground). When the rod and grounding wire
are removed, the sphere is left with a net negative charge.

18.5 Coulomb’s Law

The force between any two charges follows the same basic form as Newton’s law of
universal gravitation, that is, the electric force is proportional to the magnitude of the
charges and inversely proportional to the square of the distance between the charges.

204
Chapter 18 Electric Forces and Electric Fields

The equation for Coulomb’s law is


Kq1 q 2
FE =
r2

where F E is the electric force, q 1 and q 2 are the charges, r is the distance between their
centers, and K is a constant which equals 9 x 109 Nm2/C2.

-q1 +q2

1
Sometimes the constant K is written as K = , where ε o = 8.85 x 10-12 C2 / Nm2.
4πε o
Example 4

+2 μC -4 μC

Two point charges q 1 = +2 μC and q 2 = - 4 μC are separated by a distance r, as shown


above.
(a) If the force between the charges is 2 N, what is the value of r?
(b) Where could you place a third charge q 3 = +1 μC on the horizontal axis so that there
would be no net force acting on q 3 ? Find an equation which could be solved for x, where
x is the distance from the +2 μC charge to q 3 . It is not necessary to solve this equation.

Solution
(a)
Kq1 q 2
FE =
r2
 Nm 2 
 9 x10 9 
Kq1 q 2  C2 
r= = = 0.19 m
FE 2N

205
Chapter 18 Electric Forces and Electric Fields

(b) For the force on the third charge to be zero, it would have to be placed to the left of
the +2 μC charge. Let x be the distance from the +2 μC charge to q 3 . Then the - 4 μC
charge would be (x + r) from q 3.
x r

q3
+2 μC -4 μC

Kq1 q3 Kq 2 q3
F13 + F23 = + =0
x2 ( x + r )2
This equation can be solved for x.

18.6 The Electric Field

An electric field is the condition of space around a charge (or distribution of charges) in
which another charge will experience a force. Electric field lines always point in the
direction that a positive charge would experience a force. For example, if we take a
charge Q to be the source of an electric field E, and we bring a very small positive “test”
charge q nearby to test the strength and direction of the electric field, then q will
experience a force which is directed radially away from Q.

q F
Q

The electric field is given by the equation

F
E= ,
q
where electric field E is measured in Newtons per coulomb, and F is the force acting on
the charge q which is experiencing the force in the electric field. Electric field is a vector
which points in the same direction as the force acting on a positive charge in the electric
field. The test charge q would experience a force radially outward anywhere around the
source charge Q, so we would draw the electric field lines around the positive charge Q
like this:

Electric field lines in a region can also represent the path a positive charge would follow
in that region.

206
Chapter 18 Electric Forces and Electric Fields

Remember, electrons (negative charges) are moved when charge is transferred, but
electric field lines are drawn in the direction a positive charge would move.

The electric field due to a point charge Q at a distance r away from the center of the
charge can also be written using Coulomb’s law:

 KQq 
 2 
E= =
F r  KQ
= 2
q q r
where K is the electric constant, Q is the source of the electric field, and q is the small
charge which feels the force in the electric field due to Q.

18.7 Electric Field Lines


Drawing the electric field lines around a charge or group of charges helps us to imagine
the behavior of a small charge place in the region of the electric field. The diagrams
below illustrate the electric field lines in the region of a positive charge and a negative
charge. Your textbook has several more diagrams showing the electric field lines around
pairs of opposite charges and pairs of like charges.

Positive charge
Negative charge

The above electric fields are not uniform but vary with the square of the distance from the
source charge. We can produce a uniform electric field by charging two metal plates
oppositely and creating a capacitor. A capacitor can store charge and electric field for
later use. We will discuss capacitors further in chapter 20.

+++++++++++++++++++

---------------------------

207
Chapter 18 Electric Forces and Electric Fields

18.8 The Electric Field Inside a Conductor: Shielding


When charge is placed on a conductor, all of the charge moves to the outside of the
conductor. Consider a metal sphere. If we place positive charges totaling Q on the sphere,
they all go to the outside and distribute themselves in such a way to get as far from each
other as possible.
+ +
+ Q
+
+

+ R + r

+ +

+ +
+

Inside the metal sphere (r < R) , the electric field is zero, since all the charge is on the
outside of the sphere. Outside the sphere (r > R), the electric field behaves as if the sphere
KQ
is a point charge centered at the center of the sphere, that is, Eoutside = 2 .
r
We can graph electric field E vs. distance from the center r for the charged conducting
sphere:

r
0
R

208
Chapter 18 Electric Forces and Electric Fields

CHAPTER 18 REVIEW QUESTIONS


For each of the multiple choice questions below, choose the best answer.

1. When charge is transferred from one 4. Two charges q 1 and q 2 are separated
object to another, which of the following by a distance r and apply a force F to
are actually transferred? each other. If both charges are doubled,
(A) electrons and the distance between them is halved,
(B) protons the new force between them is
(C) neutrons (A) ¼ F
(D) quarks (B) ½ F
(E) photons (C) 4F
(D) 8F
2. Two conducting spheres of equal size (E) 16F
have a charge of – 3 C and +1 C,
respectively. A conducting wire is 5. Two uncharged spheres A and B are
connected from the first sphere to the near each other. A negatively charged
second. What is the new charge on each rod is brought near one of the spheres as
sphere? shown. The far right side of sphere B is
(A) – 4 C (A) uncharged
(B) + 4 C (B) neutral
(C) – 1 C (C) positive
(D) + 1 C (D) negative A B
(E) zero (E) equally positive and negative.

3. According to Coulomb’s law, if the


electric force between two charges is
positive, which of the following must be
true?
(A) One charge is positive and the other
charge is negative.
(B) The force between the charges is
repulsive.
(C) The force between the charges is
attractive
(D) The two charges must be equal in
magnitude.
(E) The force must be directed toward
the larger charge.

209
Chapter 18 Electric Forces and Electric Fields

9. Which of the particles would not


experience a force while between the
plates?
(A) I and II only
(B) II and III only
A B (C) I only
(D) III only
6. Two charges A and B are near each (E) I, II, and III
other, producing the
electric field lines shown. What are the
two charges A and B, respectively?
(A) positive, positive
(B) negative, negative
(C) positive, negative
(D) negative, positive
(E) neutral, neutral

7. A force of 40 N acts on a charge of


0.25 C in a region of space. The electric
field at the point of the charge is
(A) 10 N/C
(B) 100 N/C
(C) 160 N/C
(D) 40 N/C
(E) 0.00625 N/C

Questions 8 - 9:
Two charged parallel plates are oriented
as shown.
The following particles are placed
between the plates, one at a time:
I. electron
II. proton E
III. neutron

8. Which of the particles would move to


the right between the plates?
(A) I and II only
(B) I and III only
(C) II and III only
(D) II only
(E) I only

210
Chapter 18 Electric Forces and Electric Fields

+ +
+ Q
+
+

+ R + r
Q
+ +

+ +
+

10. An amount of positive charge Q is placed on a conducting sphere. A positive point


charge Q is placed at the exact center of the sphere and remains there. Which of the
following graphs best represents the of electric field E vs distance r from the center?

(A) (D)
E E

r r
R R

(B) E (E) E

r r
R R

(C) E

r
R

211
Chapter 18 Electric Forces and Electric Fields

Free Response Question


Directions: Show all work in working the following question. The question is worth 15
points, and the suggested time for answering the question is about 15 minutes. The parts
within a question may not have equal weight.

1. (15 points)
y

+Q

x
a 2a
P

+Q

Two charges each with charge +Q are located on the y – axis, each a distance a on either
side of the origin. Point P is on the x – axis a distance 2a from the origin.

(a) In terms of the given quantities, determine the magnitude and direction of the electric
field at
i. the origin
ii. point P
iii. a distance x on the x –axis a great distance from the origin (x >> 2a).

(b) On the axes below, sketch a graph of electric field E x vs. distance x on the +x – axis.

Ex

a 2a

212
Chapter 18 Electric Forces and Electric Fields

A small ball of mass m and charge +q is hung from a thread which is attached to the
ceiling directly above the mark at a distance a from the origin. Charge +q is repelled
away from the origin and comes to rest at a point of equilibrium at a distance 2a from the
origin on the
x – axis.
y

+Q
m,
a +q
a
x
a 2a
P

+Q

(c) On the diagram below, draw a free-body diagram of the forces acting on the ball when
it is in equilibrium at point P.

(d) Determine an expression for the tension F T in the string in terms of the given
quantities and fundamental constants.

213
Chapter 18 Electric Forces and Electric Fields

ANSWERS AND EXPLANATIONS TO CHAPTER 18 REVIEW QUESTIONS

Multiple Choice

1. A
When charge is transferred, electrons move from one object to another.

2. C
Conservation of charge: - 3 + 1 = - 2, which is divided evenly between the two charges,
so each sphere gets – 1 C.

3. B
In the equation for electric force, two positive or two negative charges multiplied by each
other yields a positive force, indicating repulsion.

4. E
K (2q1 )(2q 2 )
F= = 16 F
1 2
( r)
2

5. D
The far right side of sphere B is negative, since the negative charges in the sphere are
pushed as far away as possible by the negative charges on the rod.

6. D
Electric field lines begin on positive charges and end on negative charges, thus A is
negative and B is positive.

7. C
F 40 N N
E= = = 160
q 0.25C C

8. D
Only the positively charged proton would move to the right, toward the negatively
charged plate.

9. D
Since the neutron has no charge, it would not experience a force in an electric field.

10. B
KQ 2 KQ
The electric field on the inside is Einside = 2
and on the outside is E outside = 2 . In
r r
both cases, the electric field follows the inverse square law.

214
Chapter 18 Electric Forces and Electric Fields

Free Response Question Solution

(a)
i. 1 point
The electric field at the origin is zero, since a positive test charge placed at the origin
would experience no net force.

ii. 4 points
The net electric field E x at point P is equal to the sum of the x-components of the electric
field vectors from each of the two charges, since the y-components cancel.

r = a 2 + (2a )
+Q 2

a
θ θ
Ex
a 2a
P

+Q

 KQ  2a 
E x = E1x + E 2 x = 2 E cos θ = 2  2  
 r  r 
Substituting for r:
 KQ  2a 
= 2 KQa
E x = 2 2 
 a + (2a )  a + (2a )  a 2 + (2a )2 2
[ ]
2  2 2  3

iii. 2 points
If we go out to a point very far away on the x – axis where x >> 2a, the two charges seem
very close together such that they behave as one point charge of magnitude +2Q. Then
the electric field a distance x away is
K (2Q )
E=
x2

215
Chapter 18 Electric Forces and Electric Fields

(b) 2 points

Ex

a 2a

(c) 3 points

FT FTy

φ FE

FTx
mg

(d) 3 points
Since the system is in equilibrium, ΣF = 0.
 
FTx = FE = qE = q  2 KQa 
and
 2
[ 2 2 
]
3
 a + (2a ) 
FTy = mg
Then
1
 
2
2
[ ]  2 KQa 2
1
FT = FTx + FTy
2 2 2
=  + (mg ) 
 3 


[
 a 2 + (2a )2 ]
2
 

216
Chapter 20 Electric Circuits

Chapter 20

ELECTRIC CIRCUITS

PREVIEW

Conventional current is the flow of positive charges though a closed circuit. The current
through a resistance and the voltage which produces it are related by Ohm’s law. Power
is the rate at which energy is consumed in a circuit through the resistors. Resistors in a
circuit may be connected in series or in parallel. If a capacitor is placed in a circuit with
a resistor, the current in the circuit becomes time-dependent as the capacitor charges or
discharges.

The content contained in sections 1 – 4, 6 – 9, 11 – 13, and 15 of chapter 20 of the


textbook is included on the AP Physics B exam.

QUICK REFERENCE

Important Terms

ammeter
device used to measure electrical current
ampere
unit of electrical current equal to one coulomb per second
battery
device that converts chemical energy into electrical energy , creating a potential
difference (voltage)
capacitive time constant
the product of the resistance and the capacitance in a circuit; at a time equal to the
product RC, the capacitor has reached 63% of its maximum charge
capacitor
two oppositely charged conductors used to store charge and energy in an electric
field between them
direct current
electric current whose flow of charges is in one direction only
electric circuit
a continuous closed path in which electric charges can flow
electric current
flow of charged particles; conventionally, the flow of positive charges
electric power
the rate at which work is done or energy is dissipated through a resistor
electrical resistance
the ratio of the voltage across a device to the current running through it

230
Chapter 20 Electric Circuits

electron flow
the movement of electrons through a conductor; electron flow is equal and
opposite to conventional current flow
emf
electromotive force; another name for voltage
equivalent resistance
the single resistance that could replace the individual resistances
in a circuit and produce the same result
ohm
the SI unit for resistance equal to one volt per ampere
Ohm’s law
the ratio of voltage to current in a circuit is a constant called resistance
parallel circuit
an electric circuit which has two or more paths for the current to follow,
allowing each branch to function independently of the others
resistivity
the constant which relates the resistance of a resistor to its length and cross-
sectional area
resistor
device designed to have a specific resistance
schematic diagram
a diagram using special symbols to represent a circuit
series circuit
an electric circuit in which devices are arranged so that charge flows
through each equally.
terminal voltage
the actual voltage across the positive and negative terminals of a battery, which is
usually lowered by the internal resistance of the battery
voltage
the potential difference between the positive and negative sides of a circuit
element
watt
the SI unit for power equal to one joule of energy per second

231
Chapter 20 Electric Circuits

Equations and Symbols


∆q
I= where
∆t
V
R= I = current
I Δq = amount of charge passing a given
ρL
R= point
A Δt = time interval
P = IV R = resistance
Rs = R1 + R2 + R3 + ... V = voltage
P = power
1 1 1 1
= + + + ... R S = total resistance in series
R p R1 R2 R3 R P = total resistance in parallel
C P = C1 + C 2 + C 3 + ... C P = total capacitance in parallel
C S = total capacitance in series
1 1 1 1
= + + + ... q = charge
C s C1 C 2 C 3 U E = electrical energy stored in a
q = CV capacitor
τ = capacitive time constant
1 1 1 Q2
UE = CV 2 = QV =
2 2 2 C
τ = RC

Ten Homework Problems


Chapter 20 Problems 11, 26, 44, 45, 54, 65, 68, 94, 97, 106

DISCUSSION OF SELECTED SECTIONS

20.1 and 20.2 Electromotive Force and Current, and Ohm’s Law

When we connect a battery, wires, and a light bulb in a circuit shown, the bulb lights up.
But what is actually happening in the circuit?

I
I
V
+ -

+ -

232
Chapter 20 Electric Circuits

Recall from an earlier chapter that the battery has a potential difference, or voltage,
across its ends. One end of the battery is positive, and the other end is negative. We say
that the movement of positive charge from the positive end of the battery through the
circuit to the negative end of the battery is called conventional current, or simply current.
Current is the amount of charge moving through a cross-sectional area of a conductor per
second, and the unit for current is the coulomb/second, or ampere. We use the symbol I
for current. Area
∆q
+q I=
∆t

Conventional current is defined as the flow of positive charge on the AP Physics B exam,
although electrons are actually moving in the wire.

In a circuit such as the one shown above, the current is directly proportional to the
voltage. The ratio of voltage to current is a constant defined as the resistance, and is
measured in ohms (Ω), or volts/amp. The relationship between voltage, current, and
resistance is called Ohm’s law:

V
R=
I

This relationship typically holds true for the purposes of the AP Physics B exam.

20.3 Resistance and Resistivity


As charge moves through the circuit, it encounters resistance, or opposition to the flow of
current. Resistance is the electrical equivalent of friction. In our circuit above, the wires
and the light bulb would be considered resistances, although usually the resistance of the
wires is neglected. The resistance of a resistor is proportional to the length L of the
resistor and inversely proportional to the cross-sectional area A of the resistor by the
equation

ρ A
ρL
R=
A
L

where the constant ρ is called the resistivity of the resistor and has units of ohm-meters
(Ω m). The resistivity of a material is a characteristic of the material itself rather than a
particular sample of the material.

233
Chapter 20 Electric Circuits

Example 1
A copper wire has a cross-sectional area of 5.0 x 10-7 m2 and a length of 10.0 m. An
aluminum wire of exactly the same dimensions is welded to the end of the copper wire.
the ends of this long copper-aluminum wire are connected to a 3.0-volt battery. Neglect
the resistance of any other wires in the figure.

Cu Al

V
Figure not drawn to scale

Determine
(a) the total resistance of the circuit.
(b) the total current in the wire.

Solution
(a) The total resistance is equal to the sum of the copper and aluminum resistors. We
can find the value of the resistivity of copper and aluminum in the table in this chapter of
the textbook.
ρ Cu L (1.72 x10 −8 Ωm )(10 m )
RCu = = = 0.34 Ω
A 5 x10 −7 m 2
ρ L (2.82 x10 −8 Ωm )(10 m )
R Al = Al = = 0.56 Ω
A 5 x10 −7 m 2
Rtotal = RCu + R Al = 0.34 Ω + 0.56 Ω = 0.90 Ω

(b) According to Ohm’s law,


V 3.0 V
I= = = 3.33 A
Rtotal 0.90 Ω

20.4 Electric Power


In an earlier chapter we defined power as the rate at which work is done, or the rate at
which energy is transferred. When current flows through a resistor, heat is produced, and
the amount of heat produced in joules per second is equal to the power in the resistor. The
heating in the resistor is called joule heating.The equation that relates power to the
current, voltage, and resistance in a circuit is

V2
P = IV = I 2 R =
R

234
Chapter 20 Electric Circuits

The unit for power is the joule/second, or watt.

Example 2
A simple circuit consists of a 12-volt battery and a 6 Ω resistor. Let’s draw a schematic
diagram of the circuit, and include an ammeter which measures the current through the
resistor, and a voltmeter which measures the voltage across the resistor, indicate the
reading on the ammeter and voltmeter, and find the power dissipated in the resistor.

R=6Ω

I
A

+ -
12V

Note that the ammeter is placed in line (series) with the resistor, and the voltmeter is
placed around (parallel) the resistor.

The ammeter reads the current, which we can calculate using Ohm’s law:
V 12V
I= = = 2A
R 6Ω
The voltmeter will read 12 V, since the potential difference across the resistor must be
equal to the potential difference across the battery. As we will see later, if there were
more than one resistor in the circuit, there would not necessarily be 12 volts across each.

The power can be found by

P = IV = (2 A)(12 V) = 24 watts

20.6 Series Wiring


Two or more resistors of any value placed in a circuit in such a way that the same current
passes through each of them is called a series circuit. Consider the series circuit below
which includes a voltage source ε (which stands for emf, an older term for voltage) and
three resistors R 1 , R 2 , and R 3 .

235
Chapter 20 Electric Circuits

I1

R1
ε R2 I2

R3

I3

The rules for dealing with series circuits are as follows:

1. The total resistance in a series circuit is the sum of the individual resistances:

R total = R 1 + R 2 + R 3

2. The total current in the circuit is


V
I total = total .
Rtotal
This current must pass through each of the resistors, so each resistor also gets I total , that
is, I total = I 1 = I 2 = I 3.

3. The voltage divides proportionally among the resistances according to Ohm’s law:

V1 = I 1 R1 ; V2 = I 2 R2 ; V3 = I 3 R3 ;

Example 3
Consider three resistors of 2 Ω, 6 Ω, and 10 Ω connected in series with a 9 volt battery.
Draw a schematic diagram of the circuit which includes an ammeter to measure the
current through the 2 Ω resistor, and a voltmeter to measure the voltage across the 10 Ω
resistor, and indicate the reading on the ammeter and voltmeter.

236
Chapter 20 Electric Circuits

I1

ε=9V 6Ω I2

10Ω

I3

The current through each resistor is equal to the total current in the circuit, so the
ammeter will read the total current regardless of where it is placed, as long as it is placed
in series with the resistances.
V 9V
I total = total = = 0.5 A
Rtotal 2Ω + 6Ω + 10Ω

The voltmeter will read the voltage across the 10 Ω resistor, which is NOT 9 volts. The 9
volts provided by the battery is divided proportionally among the resistances. The voltage
across the 10 Ω resistor is

V 10 = I10 R 10 = (0.5 A)(10 ) = 5 V.

20.7 Parallel Wiring


Two or more resistors of any value placed in a circuit in such a way that each resistor has
the same potential difference across it is called a parallel circuit. Consider the parallel
circuit below which includes a voltage source ε and three resistors R 1 , R 2 , and R 3 .

237
Chapter 20 Electric Circuits

ε R1 R2 R3

The rules for dealing with parallel circuits are as follows:

1. The total resistance in a parallel circuit is given by the equation

1 1 1 1
= + +
Rtotal R1 R2 R3

2. The voltage across each resistance is the same:

V total = V 1 = V 2 = V 3

3. The current divides in an inverse proportion to the resistance:


V V V
I1 = 1 ; I 2 = 2 ; I 3 = 3
R1 R2 R3

where V 1 , V 2 , and V 3 are equal to each other.

Example 4
Three resistors of resistance 2 Ω, 3 Ω, and 12 Ω are connected in parallel to a battery of
voltage 24 V.
Draw a schematic diagram of the circuit which includes an ammeter to measure only the
current through the 2 Ω resistor, a voltmeter to measure the voltage across the 12 Ω
resistor, and find the total resistance in the circuit, along with the readings on the
ammeter and voltmeter.

Itot

24 V 2Ω 3Ω 12Ω
I1 I2 I1 V

238
Chapter 20 Electric Circuits

The total resistance in the circuit is found by

1 1 1 1 1 1 1 6 4 1 11
= + + = + + = + + =
Rtotal R1 R2 R3 2Ω 3Ω 12Ω 12Ω 12Ω 12Ω 12Ω

1
Notice that this fraction is not the total resistance, but . Thus, the total resistance in
Rtotal
12
this circuit must be Ω.
11

Since the ammeter is placed in series only with the 2 Ω resistance, it will measure the
current passing only through the 2Ω resistance. Recognizing that the voltage is the same
(24 V) across each resistance, we have that

V2 24V
I2 = = = 12 A
R2 2Ω

Each resistance is connected across the 24 V battery, so the voltage across the 10 Ω
resistance is 24 V.

Note that the equation for the total resistance in parallel comes from the fact that the total
current is the sum of the individual currents in the circuit, and each resistor gets the same
V V V
voltage: total = 1 + 2 + ... , where all the V ’s are equal.
Rtotal R1 R2

20.8 Circuits Wired Partially in Series and Partially in Parallel


Example 5
Consider the circuit below, which is a combination of series and parallel:
R1=20Ω

ε=13V R3=40Ω

R2=30Ω

239
Chapter 20 Electric Circuits

Find
(a) the total resistance,
(b) the total current in the circuit,
(c) the voltage across each resistor, and
(d) the current through each resistor.

Solution
We see that R 1 is in parallel with R 2 , and R 3 is in series with the parallel combination of
R 1 and R 2 .

(a) Before we can find the total resistance of the circuit, we need to find the equivalent
resistance of the parallel combination of R 1 and R 2 :

1 1 1 1 1 3 2 5
= + = + = + =
R12 R1 R2 20Ω 30Ω 60Ω 60Ω 60Ω

which implies that the combination of R 1 and R 2 has an equivalent resistance of


60
Ω = 12 Ω.
5

Then the total resistance of the circuit is R total = R 12 + R 3 = 12 Ω + 40 Ω = 52 Ω.

Vtotal 13V
(b) The total current in the circuit is I total = = = 0.25 A
Rtotal 52Ω
(c) The voltage provided by the battery is divided proportionally among the parallel
combination of R 1 and R 2 (with R 1 and R 2 having the same voltage across them), and R 3 .
Since R 3 has the total current passing through it, we can calculate the voltage across R 3 :

V 3 = I 3 R 3 = (0.25 A)(40 Ω) = 10 V.

This implies that the voltage across R 1 and R 2 is the remainder of the 13 V provided by
the battery. Thus, the voltage across R 1 and R 2 is 13 V – 10 V = 3 V.

(d) The current through R 3 is the total current in the circuit, 0.25 A. Since we know the
voltage and resistance of the other resistances, we can use Ohm’s law to find the current
through each.

V1 3V
I1 = = = 0.15 A
R1 20Ω

V2 3V
I2 = = = 0.10 A
R2 30Ω

Notice that I 1 and I 2 add up to the total current in the circuit, 0.25 A.

240
Chapter 20 Electric Circuits

20.9 and 20.10 Internal Resistance and Kirchhoff’s Rules


Consider the multi-loop circuit shown below:

R1= 8Ω R2= 12Ω R3= 20Ω

b
ε1= 6 V ε 2= 3 V

In this circuit, we have three resistors and two batteries, one on either side of junction b.
We can’t say that all the resistors are in series with each other, nor parallel, because of
the placement of the batteries. If we want to find the current in and voltage across each
resistor, we must use Kirchhoff’s rules.

Kirchhoff’s Rules for Multi-loop Circuits:

1. The total current entering a junction (like a or b in the figure above) must also leave
that junction. This is sometimes called the junction rule, and is a statement of
conservation of charge.

2. The sum of the potential rises and drops (voltages) around a loop of a circuit must be
zero. This is sometimes called the loop rule, and is a statement of conservation of energy.
a. If we pass a battery from negative to positive, we say that there is a rise in
potential, +ε.
b. If we pass a battery from positive to negative, we say that there is a drop in
potential, - ε.
c. If we pass a resistor against the direction of our arbitrarily chosen current, we
say there is a rise in potential across the resistor, + IR.
d. If we pass a resistor in the direction of our arbitrarily chosen current, we say
there is a drop in potential across the resistor, - IR.

241
Chapter 20 Electric Circuits

Example 6
Find the current through each resistor in the multiloop circuit above.

Solution
First, let’s choose directions for the each of the currents through the resistors. If we
happen to choose the wrong direction for a particular current, the value of the current will
simply come out negative.
a

I1 R2= 12Ω I2

R1= 8Ω R3= 20Ω


loop1 loop2

I3

b
ε1= 6 V ε 2= 3 V
If we apply the junction rule to junction a, then I 1 and I 2 are entering the junction, and I 3
is exiting the junction. Then

I1 + I2 = I 3 (Equation 1)

Let’s apply the loop rule beginning at junction b. Writing the potential rises and drops by
going clockwise around loop 1 gives

+ε 1 - I 1 R 1 + I 2 R 2 = 0 (Equation 2)

Note that we encountered a rise in potential (+ε) as we passed the battery, and a drop (-
I 1 R 1 ) across the first resistor, and a rise (+I 2 R 2 ) across the second resistor.

Similarly, if we write the potential rises and drops around loop 2, going clockwise
beginning at b, we get

- I 2 R 2 – I 3 R 3 - ε 2 = 0 (Equation 3)

We can solve the three equations above for the values of the three currents by substituting
the values of the resistors and the emfs of the batteries:

Equation 2:
6 V − I 1 (8 Ω ) + I 2 (12 Ω ) = 0

242
Chapter 20 Electric Circuits

Equation 3:
− I 2 (12 Ω ) − I 3 (20 Ω ) − 3V = 0

Solving Equations 1, 2, and 3 simultaneously gives


I 1 = - 6.3 A, I 2 = 9.2 A, and I 3 = 2.9 A.

Note that I 1 is negative, which simply means we chose the wrong direction for the current
through R 1 .

20.12 Capacitors in Series and Parallel


In the last chapter we discussed capacitors, two oppositely charged conductors used to
store charge. There are times when we want to know the equivalent capacitance of two or
more capacitors which are connected in a certain way. Consider two capacitors which are
connected to a battery in the figure below:

+
+ +

ε C1
- -
C2

We say that the capacitors are connected in parallel. Note that the positive plates of the
capacitors are connected to each other and the negative plates are connected to each
other. Since the positive plates are also connected to the positive terminal of the battery,
and the negative plates are connected to the negative terminal of the battery, the voltage
across each capacitor is the same as the battery voltage. In other words, capacitors in
parallel have the same voltage across them. However, the total charge will be divided
proportionally among them:

qtotal = q1 + q 2
C total Vtotal = C1V1 + C 2V2
But the voltages are all the same :
C total = C1 + C 2 (parallel)

Thus, to find the total capacitance in parallel, we simply add the capacitors.

243
Chapter 20 Electric Circuits

If we connect two capacitors in series as shown in the figure below, each capacitor will
get the same charge, but will divide the voltage proportionally.

+
C1
+ -

ε -

+
C2
-

We can find the total (or equivalent) capacitance by summing the voltages:

Vtotal = V1 + V2
qtotal q q
= 1 + 2
C total C1 C1
But, since all the charges are same,
1 1 1
= + (series)
C total C1 C 2

Example 7

8 μF
C1
24 V
4 μF
6 μF 2 μF

(a) Find the equivalent capacitance of the capacitors above.


(b) Determine the total charge in the circuit.
(c) Determine the charge on one plate of C 1 .
(d) Determine the electrical energy stored in C 1 .

244
Chapter 20 Electric Circuits

Solution
(a) We can simply add the 6 µF and the 2 µF capacitors, since they are in parallel, 6 µF +
2µF = 8 µF. But since the two 8 µF capacitors are in series, we must add their inverses to
find their equivalent capacitance:

1 1 1 2
= + =
C88 8µF 8µF 8µF
C88 = 4 µF
Now the two 4 µF capacitors are in parallel with each other, and can be added:
CT = 4 µF + 4 µF = 8µF
(b) qtot = CT ε = (8µF )(24 V ) = 192 µC
(c) C 1 is in parallel with the battery and thus has 24 V across it.
q tot = C1ε = (4 µF )(24 V ) = 96 µC
C1V 2 1 = C1ε 2 = (4µF )(24 V ) = 1152µJ
1 1 1
(d) U =
2

2 2 2

20.13 RC Circuits
A resistance-capacitance (RC) circuit is simply a circuit containing a battery, a resistor,
and a capacitor in series with one another. An RC circuit can store charge, and release it
at a later time. The capacitive time constant τ c is equal to the product of R and C (which
has units of time), and gives an indication of how long it takes for a capacitor to charge or
discharge. Typically, when connected to a battery, a capacitor is fully charged in a time
of approximately 5RC.
a
R
I

b
+

ε C
-

When the switch is moved to position a, current begins it flow from the battery, and the
ε
capacitor begins to fill up with charge. Initially, the current is by Ohm’s law, but then
R
decreases as time goes until the capacitor is full of charge and will not allow any more
charge to flow out of the battery.

245
Chapter 20 Electric Circuits

This leads us to a couple of rules we can follow when dealing with capacitors in an RC
circuit:
1. An empty capacitor does not resist the flow of current, and thus acts like a wire.
2. A capacitor which is full of charge will not allow current to flow, and thus acts like a
broken wire.

If we move the switch to position b, the battery is taken out of the circuit, and the
capacitor begins to drain its charge through the resistor, creating a current in the opposite
direction to the current flowing when the battery was connected.
a
R
I

b
+

ε C
-

Eventually, the current will die out because of the heat energy lost through the resistor.

Example 8
a

I

b
+
12V
6μF
-

A 12-volt battery is connected in series to a 3 Ω resistor and an initially uncharged


capacitor. The switch is connected to position a in the circuit shown above.
(a) Determine the current in the circuit immediately after the battery is connected to the
resistor and capacitor,
(b) Determine the current in the circuit a long time later.
(c) Determine the maximum charge on the capacitor.

The switch is then moved to position b.


(d) Determine the total amount of energy dissipated through the resistor.

246
Chapter 20 Electric Circuits

Solution
(a) Immediately after the circuit is connected, the capacitor is still empty and thus acts
like a wire.

V 12V
The current, then, is I = = = 4A
R 3Ω

(b) Then the current begins to decrease as the capacitor fills up with charge, and after a
long time, the capacitor is full of charge, and the current stops flowing completely. Thus,
I = 0 a long time later.
(c) When the capacitor is full of charge, the voltage across it is equal and opposite to the
battery emf.
q = Cε = (6 µF )(12 V ) = 72 µC

(d) The total energy dissipated through the resistor is equal to the maximum electrical
energy U E stored in the capacitor:
U = CV 2 = Cε 2 = (6 µF )(12 V ) = 432 µJ .
1 1 1 2

2 2 2

CHAPTER 20 REVIEW QUESTIONS


For each of the multiple choice questions below, choose the best answer.

Questions 1 – 3: 2. The current in the 2 Ω resistor is


(A) 6 A
Two resistors of 2 Ω and 4 Ω are placed (B) 4 A
in series with a 12-V battery. (C) 3 A
(D) 2 A
1. Which of the following statements is (E) 1 A
true?
(A) The 2Ω will get more current than 3. The voltage across the 4 Ω resistor is
the 4 Ω resistor since it has less (A) 2 V
resistance. (B) 4 V
(B) The 4 Ω will get more current than (C) 6 V
the 2 Ω resistor since it has more (D) 8 V
resistance. (E) 12 V
(C) The 2 Ω and 4 Ω resistors will get
the same voltage.
(D) The 2Ω resistor will get more
voltage than the 4Ω resistor.
(E) The 4Ω resistor will get more
voltage than the 2Ω resistor.

247
Chapter 20 Electric Circuits

Questions 4 – 6 6. What is the correct reading on


ammeter 2?
(A) zero
(B) 1 A
A1 A2
(C) 2 A
(D) 3 A
(E) 6 A

V1

6V


A3

4V

V2

Two resistors of 3Ω and 6Ω are placed 7. What is the total current flowing in
in parallel with a 6-V battery. Three the circuit shown above?
ammeters and two voltmeters are placed (A) 4 A
in the circuit as shown. (B) 3 A
(C) 2 A
4. Which of the following statements is (D) 4/3 A
true of the voltmeters? (E) 2/3 A
(A) Voltmeter 1 and voltmeter 2 will
read the same voltage.
(B) Voltmeter 1 will read 6 V.
(C) Voltmeter 2 will read 6 V. 2Ω
(D) Voltmeter 2 will read the correct C
voltage across the 3Ω resistor.
(E) Both voltmeters will read the correct
10V
voltage across the 6Ω resistor.

5. Which ammeter will read the highest
amount of current?
(A) Ammeter 1 8. A resistance-capacitance circuit is
(B) Ammeter 2 connected as shown above.
(C) Ammeter 3 After a long time, the current in the
(D) All three will read the same current. 2Ω resistor is
(E) All three will read zero current. (A) zero
(B) 10 A
(C) 8 A
(D) 2 A
(E) 1 A

248
Chapter 20 Electric Circuits

2A

10. The aluminum cylinder above has a


6Ω length L and a cross-sectional area A.
20V Which of the following would double
9. The internal resistance of the battery the resistance of the aluminum?
in the portion of the circuit above is (A) double both the length and the area
(A) 12 Ω (B) double only the length
(B) 10 Ω (C) double only the area
(C) 8 Ω (D) reduce the length by half
(D) 7 Ω (E) reduce the area by one quarter
(E) 4 Ω

Free Response Question


Directions: Show all work in working the following question. The question is worth 15 points,
and the suggested time for answering the question is about 15 minutes. The parts within a
question may not have equal weight.

1. (15 points)

4Ω 2Ω

ε=6V 6Ω
3 μF

The switch in the circuit above is initially open and the capacitor is uncharged. The switch is
then closed.
(a) Determine the total current in the circuit immediately after the switch is closed.

(b) Determine the current in the circuit after the switch has been closed for a long time.

249
Chapter 20 Electric Circuits

(c) On the axes below, sketch a graph of current I vs time t from the time the switch is closed at t
= 0 to the time when the capacitor is completely full of charge.
Be sure to label any important points on the axes.

t
0

(d) Determine the voltage across the capacitor after it is fully charged.

(e) After a long time, the 2 Ω resistor fails, breaking the circuit at the location of the 2 Ω resistor,
but before the parallel combination of the 6 Ω resistor and the capacitor. Determine the
magnitude and direction of the initial current in the 6 Ω resistor immediately after the 2 Ω
resistor fails.

250
Chapter 20 Electric Circuits

ANSWERS AND EXPLANATIONS TO CHAPTER 20 REVIEW QUESTIONS


Multiple Choice

1. E
Since both resistors get the same current, the 4 Ω resistor needs more voltage than the 2
Ω resistor, since the 4 Ω has more resistance.

2. D
The total resistance in the series circuit is the sum of the resistors, or 6Ω.
The current in the 2 Ω resistor is the same as the total current in the circuit, which is I = V/R =
12V/6Ω = 2 A.

3. D
Since the 4 Ω resistor represents 2/3 of the total resistance, it will get 2/3 of the total voltage, or 8
V.

4. B
The voltage across the 6 Ω resistor is 6 V since it is in parallel with the battery. Voltmeter 2 is
incorrectly placed in the circuit and will read zero volts.

5. A
Ammeter 1 is in a position to measure the total current in the circuit before it splits up to go
through the 3Ω and 6Ω resistors.

6. B
Ammeter 2 measures only the current through the 6Ω resistor: I = 6V/6Ω = 1 A.

7. B
1 1 1 3
The total resistance can be found by = + = , which implies that R t = 4/3 Ω
Rt 2Ω 4Ω 4Ω
V 4V
So, the total current in the circuit is I = = = 3A .
R 4

3
8. E
After a long time, the capacitor is full of charge and does not allow any more current to flow
through it. Thus, the current from the battery will only flow through the resistors and the current
will be I = 10V/10Ω = 1 A.

9. E
20V
By Ohm’s law, 2 A = , so the internal resistance R = 4 Ω.
(6Ω + R )
10. B
ρL ρ (2 L )
R= , so R = = 2R
A A

251
Chapter 20 Electric Circuits

Free Response Question Solution

(a) 3 points
Immediately after the switch is closed the capacitor is empty and acts like a wire (short) around
the 6 Ω resistor. Thus, the current in the circuit is
V 6V
Io = = =1A
Rtot 6Ω

(b) 2 points
After a long time, the capacitor is full, so no current flows through it, and the current flows
through the 6 Ω resistor.
V 6V
If = = = 0.5 A
Rtot 12Ω

(c) 4 points

1A

0.5A

t
0

(d) 3 points
When the capacitor is fully charged, it has the same voltage across it as the 6 Ω resistor, since
they are in parallel.
Vc = V6 = I f R6 = (0.5 A)(6Ω ) = 3V I

(e) 3 points
The capacitor acts as the source of voltage for 6Ω
3 μF
the current through the 6 Ω resistor.
3V
I= = 0.5 A counterclockwise
6Ω

252

You might also like